You are on page 1of 204

Marlia Brasil Xavier REITORA

Prof. M. Sc. Rubens Vilhena Fonseca COORDENADOR GERAL DOS CURSOS DE MATEMTICA

MATERIAL DIDTICO

COLABORAO
Maria da Glria Costa Lima Cleyton Isamu Muto

EDITORAO ELETRONICA
Odivaldo Teixeira Lopes

ARTE FINAL DA CAPA


Odivaldo Teixeira Lopes

REALIZAO

BELM PAR BRASIL - 2011 -

SUMRIO
Captulo 1:...............................................................................................................................................9 NMEROS INTEIROS NOES FUNDAMENTAIS ...................................................................................9
1.1 NMEROS INTEIROS ....................................................................................................................................... 10 1.2 PROPRIEDADES DOS INTEIROS ........................................................................................................................ 11 1.3 VALOR ABSOLUTO DE UM INTEIRO ................................................................................................................ 12 1.4 REPRESENTAO DOS INTEIROS EM OUTRAS BASES ...................................................................................... 14 1.5 FATORIAL E PRINCPIO FUNDAMENTAL DA CONTAGEM .................................................................................. 15 1.6 PRINCPIO FUNDAMENTAL DA CONTAGEM - PFC ............................................................................................ 16 1.7 NMERO BINOMIAL ........................................................................................................................................ 17 1.8 NMEROS BINOMIAIS COMPLEMENTARES...................................................................................................... 18 1.9 NMEROS BINOMIAIS CONSECUTIVOS ............................................................................................................ 18 1. 10 PISO, TETO E NINT DE UM NMERO REAL. ................................................................................................ 20 1.11 O PRINCPIO DA CASA DOS POMBOS (PRINCPIO DAS GAVETAS DE DIRICHLET) .............................................. 24 1.12 CAOS FATORIAL: !N. ..................................................................................................................................... 25 1.13 LEFT FATORIAL: L!N ..................................................................................................................................... 26 EXERCCIOS............................................................................................................................................................ 27

Captulo 2:.............................................................................................................................................30 INDUO MATEMTICA ........................................................................................................................30


2.1 ELEMENTO MNIMO DE UM CONJUNTO DE INTEIROS ...................................................................................... 30 2.2 PRINCPIO DA BOA ORDENAO .................................................................................................................... 31 2.3 PRINCPIO DE INDUO FINITA. ...................................................................................................................... 32 2.4 INDUO MATEMTICA ................................................................................................................................ 33 2.5. EXEMPLOS DE DEMONSTRAO POR INDUO MATEMTICA .......................................................................... 35 2.6 . OUTRAS FORMAS DA INDUO MATEMTICA ................................................................................................ 37 EXERCCIOS............................................................................................................................................................ 42

Captulo 3:.............................................................................................................................................43 SOMATRIOS E PRODUTRIOS .............................................................................................................43


3.1 . SOMATRIOS ................................................................................................................................................. 43 3.2. PROPRIEDADES DOS SOMATRIOS................................................................................................................... 44 3.3. PRODUTRIOS................................................................................................................................................. 45 3.4. PROPRIEDADES DOS PRODUTRIOS ................................................................................................................. 46

Captulo 4 ..............................................................................................................................................48 DIVISIBILIDADE .....................................................................................................................................48


4.1. RELAO DE DIVISIBILIDADE EM Z ................................................................................................................. 48 4.2. CONJUNTO DOS DIVISORES DE UM INTEIRO ..................................................................................................... 50 4.3. DIVISORES COMUNS DE DOIS INTEIROS ........................................................................................................... 50 4.4. TEOREMA DA DIVISO ................................................................................................................................... 51 4.5. PARIDADE DE UM INTEIRO .............................................................................................................................. 54 EXERCCIOS............................................................................................................................................................ 56

Captulo 5 ..............................................................................................................................................58 MXIMO DIVISOR COMUM ...................................................................................................................58


5.1. MXIMO DIVISOR COMUM DE DOIS INTEIROS .................................................................................................. 58 5.2. EXISTNCIA E UNICIDADE DO MDC. ................................................................................................................ 59 5.3. INTEIROS RELATIVAMENTE PRIMOS (COPRIMOS OU PRIMOS ENTRE SI) ............................................................ 61 5.4. CARACTERIZAO DO MDC DE DOIS INTEIROS ................................................................................................ 64 5.5. MDC DE VRIOS INTEIROS .............................................................................................................................. 64 EXERCCIOS............................................................................................................................................................ 65

Captulo 6 ............................................................................................................................................. 67 ALGORITMO DE EUCLIDES MNIMO MLTIPLO COMUM................................................................. 67


6.1. ALGORITMO DE EUCLIDES .............................................................................................................................. 67 6.2 . MLTIPLOS COMUNS DE DOIS INTEIROS ......................................................................................................... 74 6.3. MNIMO MLTIPLO COMUM DE DOIS INTEIROS ................................................................................................ 75 6.5. MMC DE VRIOS INTEIROS .............................................................................................................................. 76 EXERCCIOS ............................................................................................................................................................ 78

Captulo 7 ............................................................................................................................................. 79 NMEROS PRIMOS ................................................................................................................................ 79


7.1. INTRODUO .................................................................................................................................................. 79 7.2. NMEROS PRIMOS (DO LAT. PRIMUS, PRINCIPAL. PRIME EM INGLS) .............................................................. 81 7. 3. TEOREMA FUNDAMENTAL DA ARITMTICA. ................................................................................................... 82 7.4. A SEQNCIA DOS NMEROS PRIMOS .............................................................................................................. 84 7.5. O CRIVO DE ERATSTENES. .............................................................................................................................. 86 7.6. SEQNCIA DE INTEIROS CONSECUTIVOS COMPOSTOS .................................................................................... 94 7.7 . CONJECTURAS ................................................................................................................................................ 96 7.8. FRMULAS QUE GERAM ALGUNS NMEROS PRIMOS........................................................................................ 98 7.9. DECOMPOSIO DO FATORIAL EM FATORES PRIMOS ..................................................................................... 101 7.10. MTODO DA FATORAO DE FERMAT .......................................................................................................... 105 7. 11 ALGORITMO DE FERMAT ............................................................................................................................ 105 EXERCCIOS .......................................................................................................................................................... 107

Captulo 8: .......................................................................................................................................... 110 EQUAES DIOFANTINAS LINEARES ................................................................................................. 110


3.1. GENERALIDADES ........................................................................................................................................... 111 3.2. CONDIO DE EXISTNCIA DE SOLUO ........................................................................................................ 112 3.3. SOLUES DA EQUAO AX + BY = C. ........................................................................................................... 113 EXERCCIOS .......................................................................................................................................................... 115

Captulo 9 ........................................................................................................................................... 117 CONGRUNCIAS .................................................................................................................................. 117


9.1. CONGRUNCIAS ............................................................................................................................................ 117 9.2. CARACTERIZAO DE INTEIROS CONGRUENTES ............................................................................................ 117 9.3. PROPRIEDADES DAS CONGRUNCIAS ............................................................................................................. 118 9.4. SISTEMAS COMPLETOS DE RESTOS................................................................................................................ 121 9.5 ARITMTICA MDULO M .............................................................................................................................. 122 9.6. ADIO E MULTIPLICAO EM m .............................................................................................................. 124 9.7. SUBTRAO EM 9.8. DIVISO EM

m ...................................................................................................................................... 130

m ............................................................................................................................................ 131 9.9. POTENCIAO EM m .................................................................................................................................. 135


EXERCCIOS .......................................................................................................................................................... 139

Captulo 10 ......................................................................................................................................... 141 TEOREMAS DE FERMAT, WILSON E EULER .......................................................................... 141


10.1. PEQUENO TEOREMA DE FERMAT ....................................................................................................... 141 EXERCCIOS .......................................................................................................................................................... 145 10.2. TEOREMA DE WILSON .................................................................................................................................. 146
EXERCCIOS ............................................................................................................................................................................. 149

10.3. TEOREMA DE EULER .................................................................................................................................... 150 10.4. FUNO TOTIENT (N) ................................................................................................................................. 151 10.5 CLCULO DE (N) ...................................................................................................................................... 152 10.6. RESOLUO DE CONGRUNCIAS LINEARES PELO TEOREMA DE EULER ......................................................... 155 10. 7. RESOLUO DA EQUAO (N).................................................................................................................. 156

10.8 VALNCIA DA FUNO TOTIENTE: N (m) . ............................................................................................ 159 EXERCCIOS.......................................................................................................................................................... 160 10.9. TEOREMA CHINS DO RESTO (TCR) .............................................................................................................. 161 10.10. POTENCIAO: UMA APLICAO DO TEOREMA DE EULER ......................................................................... 165 10.11 POTENCIAO: UMA APLICAO DO TEOREMA CHINS DO RESTO (TCR) .................................................. 165

Captulo 11 ..........................................................................................................................................171 CIFRA DE CSAR ..................................................................................................................................171


11.1. FUNES POLINOMIAIS DE CODIFICAO .................................................................................................... 174

Captulo 12 ..........................................................................................................................................179 CIFRA DE VIGENRE ...........................................................................................................................179 Captulo 13 ..........................................................................................................................................182 CIFRA DE HILL.....................................................................................................................................182 Captulo 14 ..........................................................................................................................................190 RSA .......................................................................................................................................................190
14. 1. PR-CODIFICAO ...................................................................................................................................... 190 14.2 CODIFICANDO E DECODIFICANDO ............................................................................................................... 191 14. 3. ASSINATURA DIGITAL UTILIZANDO A CRIPTOGRAFIA RSA .......................................................................... 195

Captulo 15 ..........................................................................................................................................201 PARTILHA DE SENHAS .........................................................................................................................201

CAPTULO 1 NMEROS INTEIROS NOES FUNDAMENTAIS

Captulo 1:

NMEROS INTEIROS NOES FUNDAMENTAIS


INTRODUO

Teoria dos Nmeros nasceu cerca de 600 anos antes de Cristo quando Pitgoras e os seus discpulos comearam a estudar as propriedades dos nmeros inteiros. Os pitagricos rendiam verdadeiro culto mstico ao conceito de nmero, considerando-o como essncia das coisas. Acreditavam que tudo no universo estava relacionado com nmeros inteiros ou razes de nmeros inteiros (em linguagem atual, nmeros racionais). Alis, na antiguidade a designao nmero aplicava-se s aos inteiros maiores do que um. http://nonio.fc.ul.pt/analise1/cap1/hnum.htm O conceito de nmero tomou forma num longo desenvolvimento histrico. A origem e formulao deste conceito ocorreu simultaneamente com o despontar, entenda-se nascimento, e desenvolvimento da Matemtica. As atividades prticas do homem, por um lado, e as exigncias internas da Matemtica por outro determinaram o desenvolvimento do conceito de nmero. A necessidade de contar objetos levou ao aparecimento do conceito de nmero Natural. Todas as naes que desenvolveram formas de escrita introduziram o conceito de nmero Natural e desenvolveram um sistema de contagem. O desenvolvimento subsequente do conceito de nmero prosseguiu principalmente devido ao prprio desenvolvimento da Matemtica. Os nmeros negativos aparecem pela primeira vez na China antiga. Os chineses estavam acostumados a calcular com duas colees de barras - vermelha para os nmeros positivos e preta para os nmeros negativos.No entanto, no aceitavam a idia de um nmero negativo poder ser soluo de uma equao. Os Matemticos indianos descobriram os nmeros negativos quando tentavam formular um algoritmo para a resoluo de equaes quadrticas. So exemplo disso as contribuies de Bramaghupta, pois a aritmtica sistematizada dos nmeros negativos encontra-se pela primeira vez na sua obra. As regras sobre grandezas eram j conhecidas atravs dos teoremas gregos sobre subtrao, como por exemplo (a - b)(c - d) = ac + bd - ad - bc, mas os hindus converteram-nas em regras numricas sobre nmeros negativos e positivos. Diofanto (Sc. III) operou facilmente com os nmeros negativos. Eles apareciam constantemente em clculos intermdios em muitos problemas do seu "Aritmetika", no entanto havia certos problemas para o qual as solues eram valores inteiros negativos como por exemplo: 4x + 20 = 4 ou 3x 18 = 5x2 Nestas situaes Diofanto limitava-se a classificar o problema de absurdo. Nos sculos XVI e XVII, muitos matemticos europeus no apreciavam os nmeros negativos e, se esses

CAPTULO 1 NMEROS INTEIROS NOES FUNDAMENTAIS

nmeros apareciam nos seus clculos, eles consideravam-nos falsos ou impossveis. Exemplo deste fato seria Michael Stifel (1487- 1567) que se recusou a admitir nmeros negativos como razes de uma equao, chamando-lhes de "numeri absurdi". Cardano usou os nmeros negativos embora chamando-os de "numeri ficti". A situao mudou a partir do (Sc.XVIII) quando foi descoberta uma interpretao geomtrica dos nmeros positivos e negativos como sendo segmentos de direes opostas. http://www.somatematica.com.br/historia.php

1.1 Nmeros Inteiros


Os nmeros inteiros ou apenas os inteiros so: ..., -3, -2, -1, 0, 1, 2, 3,... cujo conjunto representa-se pela letra Z, isto : Z = {..., -3, -2, -1, 0, 1, 2, 3,...} Neste conjunto Z destacam-se os seguintes subconjuntos: 1) Conjunto Z* dos inteiros no nulos ( 0 ): Z* = {x Z | x
0} { 1, 2, 3,...}

2) Conjunto Z dos inteiros no negativos ( 0 ):

{x Z | x 0} = {0, 1, 2, 3,...}

3) Conjunto Z dos inteiros no positivos ( 0 ):

{x Z | x 0} = {0, -1, -2, -3,...}

4) Conjunto Z* dos inteiros positivos (> 0):


Z* {x Z | x 0} = {1, 2, 3,...}

5) Conjunto Z* dos inteiros negativos (< 0):


Z* {x Z | x 0} = {-1, -2, -3,...}

Os inteiros positivos so tambm denominados inteiros naturais e por isso o conjunto dos inteiros positivos habitualmente designado pela letra N (N = Z* ).

10

CAPTULO 1 NMEROS INTEIROS NOES FUNDAMENTAIS

1.2 Propriedades dos Inteiros


O conjunto Z dos inteiros munido das operaes de adio (+) e multiplicao ( . ) possui as propriedades fundamentais que a seguir enumeramos, onde a, b e c so inteiros quaisquer, isto , elementos de Z: 1) a + b = b + a e ab = ba; e (ab) c = a (bc);

2) (a + b) + c = a + (b + c) 3) 0 + a = a e 1.a = a; 4) a = (-1) a e

a a = a + (-a) = 0;

5) a (b + c) = ab + ac; 6) 0.a = 0, e se ab = 0, ento a = 0 ou b = 0. Tambm existe uma relao de ordem entre os inteiros, representada pelo sinal < (menor que), que possui as seguintes propriedades: 7) Se a 0 , ento a > 0 ou a < 0; 8) Se a < b e b < c, ento a < c; 9) Se a < b, ento a + c < b + c; 10) Se a < b e 0 < c, ento ac < bc; 11) Se a < b e c < 0, ento bc < ac. Destas propriedades podem ser deduzidas muitas outras propriedades dos inteiros. Exemplo 1.1: Demonstrar: -(a + b) = (-a) + (-b). Com efeito, temos sucessivamente: -(a + b) = (-1) (a + b) = = (-1) a + (-1) b = = (-a) + (-b) (Propriedade 4) (Propriedade 5) (Propriedade 4)
x2 .

Exemplo 1.2: Demonstrar que , se x 0 , ento 0 Com efeito: 1) Se x 0 , ento 2) Se x 0 , ento 3) Se 0


x 0 ou 0 0.x x.x 0 x2 x , ento 0.x x.x 0 x2 x

(Propriedade 7) (Propriedade 11) (Propriedade 6) (Propriedade 10) (Propriedade 6)

11

CAPTULO 1 NMEROS INTEIROS NOES FUNDAMENTAIS

Nota: Com o mesmo significado de a < b, escreve-se b > a. Indica-se, de modo abreviado, que a < b ou a = b por a b . Por exemplo, temos 2 3 , porque 2 < 3, e 2 2 , porque 2 - 2. Com o mesmo significado de a b , escreve-se b a . Em lugar de a b e b c tambm se escreve a b c .

1.3 Valor absoluto de um Inteiro


Definio 1.1: Chama-se valor absoluto de um inteiro a, o inteiro que se indica por | a | , e tal que:

|a|

a, se a 0 a, se a < 0

Assim, por exemplo:


| 3| 3

| 5|

( 5)

Consoante a definio de | a | , para todo inteiro a, temos:


|a| 0, |a| a , | a | | a | , a | a |

O valor absoluto | a | de um inteiro a tambm pode ser definido pelas igualdades:


|a| a , | a | = mx [-a, a]

onde a denota a raiz quadrada no negativa de a e mx [-a, a] indica o maior dos dois inteiros a e a.

Assim, por exemplo:


| 4| ( 4) 16 4

| 6 | = mx [-6, 6] = 6

Teorema 1.1: Se a e b so dois inteiros, ento:


| ab | | a | .| b |

12

CAPTULO 1 NMEROS INTEIROS NOES FUNDAMENTAIS

Demonstrao: Com efeito:


| ab | (ab) ab a. b | a | . | b |

Teorema 1.2: Se a e b so dois inteiros, ento:


|a b| |a | |b|

Demonstrao: Com efeito, pela definio de | a | , temos:


| a | a | a |, | b| b | b|

Somando ordenadamente estas desigualdades, obtemos:


(| a | | b |) a b | a | | b |

o que implica:
|a b| |a | |b|

* Usou se o fato de que x

a.

Corolrio 1.1: Se a e b so dois inteiros, ento:


| a b| |a | | b|

Demonstrao: Com efeito:


|a b | | a ( b) | | a | | b| |a | | b|

13

CAPTULO 1 NMEROS INTEIROS NOES FUNDAMENTAIS

1.4 Representao dos Inteiros em outras Bases


Teorema 1.3: Dado um inteiro qualquer b representao da forma: 2, todo inteiro positivo n admite uma nica

n ambm am 1bm 1 a2b2 a1b a0


onde os ai so tais que 0 ai < b , i = 0, 1, ... , m

Demonstrao: Assim, dado um inteiro qualquer b 2 , todo inteiro positivo n pode ser representado por um polinmio inteiro em b do grau m (porque am 0 ), ordenado segundo as potencias decrescentes de b, e cujos coeficientes ai so inteiros que satisfaam as condies:

0 ai

b(i 0,1, 2,, m) , sendo am

Este polinmio representa-se, de modo abreviado, pela notao:

n (amam 1 a2a1a0 )b
em que os coeficientes ai so indicados pela ordem respectiva, figurando o inteiro b como um ndice. O inteiro b chama-se base e costume dizer que n est escrito no sistema de base b.

Exemplos:
a) Escrever 105 no sistema binrio 105 = 1.26 + 1.25 + 0.24 + 1.23 + 0.22 + 0.2 + 1 = (1101001)2 Por outro lado, (100111)2 = 1.25 + 0.24 + 0.23 + 1.22 + 1.2 + 1 = 39 b) Escrever 31415 no sistema de base 8 Temos, sucessivamente: 31415 3926 490 61 7 Portanto 31415 = 7.84 + 5.83 + 2.82 + 6.8 + 7 = 8.3926 = 8.490 = 8.61 = 8.7 = 8.0 = (75267)8 + + + + + 7 6 2 5 7

14

CAPTULO 1 NMEROS INTEIROS NOES FUNDAMENTAIS

c) Escrever (3531)6 no sistema de base 10 Temos, (3531)6 = 3.63 + 5.62 + 3.6 + 1 = 847 d) Escrever (6165)7 no sistema de base 12 Temos, (6165)7 = 6.73 + 1.72 + 6.7 + 5 = 2154 Vamos escrever 2154 (base 10) na base 12: 2154 179 14 1 = 12.179 = 12.14 = 12.1 = 12.0 +6 + 11 +2 +1

No sistema de base 12 hbito designar 10 e 11 por a e b, respectivamente, de modo que os algarismos deste sistema so: 0, 1, 2, 3, 4, 5, 6, 7, 8, 9, a, b. Portanto, 2154 = 1.123 + 2.122 + b.12 + 6 = (12b6)12 Assim, no sistema de numerao decimal, dado um inteiro n, temos que, n = am. 10m + am-1. 10m-1 + ... + a1. 10 + a0, 0 ak 10 a representao no sistema decimal do inteiro positivo n. Podemos tambm dizer que todo inteiro positivo n pode ser expresso sob a forma: n = 10k + a0 Onde a0 o algarismo das unidades de n

1.5 Fatorial e Princpio Fundamental da Contagem


Foi a necessidade de calcular o nmero de possibilidades existentes nos chamados jogos de azar que levou ao desenvolvimento da Anlise Combinatria, parte da Matemtica que estuda os mtodos de contagem. Esses estudos foram iniciados j no sculo XVI, pelo matemtico italiano Niccollo Fontana (1500-1557), conhecido como Tartaglia. Depois vieram os franceses Pierre de Fermat (1601-1665) e Blaise Pascal (1623-1662). A Anlise Combinatria visa desenvolver mtodos que permitam contar - de uma forma indireta - o nmero de elementos de um conjunto, estando esses elementos agrupados sob certas condies. Definio 1.2: Chama-se fatorial de um inteiro no negativo n ( n 0 ), o inteiro que se indica por n!, e tal que:
1, se n = 0 ou n = 1 n(n 1)(n 2)...3.2.1 se n 2

n!

Assim, por exemplo:

15

CAPTULO 1 NMEROS INTEIROS NOES FUNDAMENTAIS

7! = 7.6.5.4.3.2.1 = 5040 Observe-se que n! = n.(n-1)!.

Exemplo 1.4: Escrever, usando o smbolo de fatorial, o produto dos n primeiros inteiros positivos pares e o produto dos n primeiros inteiros positivos mpares. Os n primeiros inteiros positivos pares so: 2,4,6, ..., 2n 2, 2n Isto : 2.1,2.2,2.3, ..., 2 . (n 1), 2n Portanto: 2,4,6, ..., 2n 2, 2n = 2n (1.2.3... (n -1).n) = 2n . n! Os n primeiros inteiros positivos mpares so: 1,3,5, ..., 2n 3, 2n - 1 Portanto:
1.3.5...(2n 3).(2n 1) 1.2.3.4...(2n 2).(2n 1).2n 2.4.6...(2n 2).2n (2n )! 2n.n !

Exemplo 1.5: Calcular a soma: 1.1! + 2.2 ! + 3.3! + ... + n.n! Tomemos a igualdade: k.k! = (k + 1)! k! e nela faamos sucessivamente k = 1, 2, 3,..., n, o que d: 1.1! = 2! 1 2.2! = 3! 2! 3.3! = 4! 3! n.n! = (n + 1)! n! Somando ordenadamente todas essas n igualdades e simplificando, obtemos: 1.1! + 2.2! + 3.3! +...+ n.n! = (n + 1)! 1

1.6 Princpio fundamental da contagem - PFC 16

CAPTULO 1 NMEROS INTEIROS NOES FUNDAMENTAIS

Se determinado acontecimento ocorre em n etapas diferentes, e se a primeira etapa pode ocorrer de k1 maneiras diferentes, a segunda de k2 maneiras diferentes, e assim sucessivamente , ento o nmero total T de maneiras de ocorrer o acontecimento dado por: T = k1. k2 . k3 . ... . kn

1.7 Nmero Binomial


Definio 1.3: Sejam n > 0 e k dois inteiros tais que 0 k n . Chama-se nmero binomial de numerador n e classe k, o inteiro que se indica por

n k

, e tal que:

n k

n! k!(n k)!

Obviamente, tambm podemos escrever:

n k

n(n 1)...(k 1) (n k)!

n(n 1)...(n k 1) k!

Em particular, para k = 0 ou k = n, temos:

n 0
Assim, por exemplo:

n n

8 3

8! 8.7.6.5.4.3.2.1 8.7.6 56 3!5! 3.2.1.5.4.3.2.1 3.2.1 7 7.6.5 7.6.5 35 4 (7 4)! 3.2.1

17

CAPTULO 1 NMEROS INTEIROS NOES FUNDAMENTAIS

1.8 Nmeros Binomiais Complementares


Definio 1.4: Chamam-se nmeros binomiais complementares dois nmeros binomiais que tm o mesmo numerador e cuja soma das suas classes respectivas igual ao numerador comum. Assim, por exemplo,

20 7

20 13

so nmeros binomiais complementares, pois, tm o

mesmo numerador 20 e 7 + 13 = 20. Teorema 1.4: Dois nmeros binomiais complementares so iguais.

Demonstrao: Sejam

n k

n h

dois nmeros binomiais complementares. Ento, k + h = n e k = n h.

Portanto:

n k

n n h

n! n! (n h)!(n (n h))! (n h)!h!

n h

1.9 Nmeros Binomiais Consecutivos


Definio 1.5: Chamam-se nmeros binomiais consecutivos dois nmeros binomiais que tm o mesmo numerador e cujas classes respectivas so inteiros consecutivos. Assim, por exemplo,

18 9

18 10

so nmeros binomiais consecutivos, pois, tm o mesmo

numerador 18 e as suas classes respectivas so os inteiros consecutivos 9 e 10. Teorema 1.5: Entre dois nmeros binomiais consecutivos subsiste a relao de Stifel:

n k 1

n k

, com 1 k n ,

n k 1

n k

n 1 k

18

CAPTULO 1 NMEROS INTEIROS NOES FUNDAMENTAIS

Demonstrao: Com efeito:

n k 1

n k

n! n! (k 1)!(n k 1)! k!(n k)!

n! n! (k 1)!(n k 1)(n k)! k(k 1)!(n k)! n! 1 1 (k 1)!(n k)! n k 1 k

n! n 1 (k 1)!(n k)! k(n k 1) n 1 (n 1)! k!(n 1 k)!


Assim, por exemplo:

18 9 13 8
Corolrio 1.2:

18 10 12 8

19 10 12 7 k 1 k 1

n k

n 1 k 1

n 2 k 1

...

k k 1

Demonstrao: Com efeito, mudando na relao de Stifel n sucessivamente por n 1, obtemos: n 2, n 3,..., k,

n k n 1 k n 2 k n 1 k

n 1 k 1 n 2 k 1 n 3 k 1 k k 1

n 1 k n 2 k n 3 k k k

...........................................

19

CAPTULO 1 NMEROS INTEIROS NOES FUNDAMENTAIS

Alm disso, evidente:

k k

k 1 k 1

Somando ordenadamente todas essas igualdades e suprimindo os termos comuns aos dois membros acha-se a relao desejada. Substituindo, nesta relao, cada nmero binomial pelo seu complementar, obtemos:

n n k n k n 1 k

n 1 n k n 2 k 1 ...

n 2 n k 1 n k 1

...

k 1

k 1 0

Corolrio 1.3:

n k 1 0

Demonstrao: Consoante a relao de Stifel, temos:


n k n 1 k 1 n 2 k 2 n k 1 1 n 1 k 1 n 2 k 2 n 3 k 3 n k 0 n 1 k n 2 k 1 n 3 k 2 n k 1

...........................................

Alm disso, temos:


n k 0 n k 1 0

Somando ordenadamente todas essas igualdades e suprimindo os termos comuns aos dois membros acha-se a relao desejada.

1. 10 Piso, Teto e Nint de um nmero real.


fcil perceber que qualquer nmero real est entre dois nmeros inteiros, um inteiro menor que o dado nmero real e um inteiro maior que esse nmero real. Por exemplo, o nmero real 3 , est entre os inteiros -5 e 5 , est entre os inteiros 2 e 3 ( 2 5 3 ); o nmero real 2 3 4( 5 4 ), etc.. Veremos a seguir que o inteiro esquerda ser chamado de Piso 2 (floor) e o inteiro direita ser chamado de Teto(ceiling).

20

CAPTULO 1 NMEROS INTEIROS NOES FUNDAMENTAIS

Definio 1.6: Chamam-se partes inteiras de um nmero real r, os inteiros n e n+1 que verificam s condies:

n 1

A todo nmero real r podemos associar dois nmeros inteiros chamados piso e teto. Keneth Iverson introduziu esses nomes, assim como a notao que ser usada, no incio da dcada de 1960. Definio 1.7: Chama-se piso de um nmero real r, ao maior nmero inteiro menor ou igual a r. Definio 1.8: Chama-se teto de um nmero real r, ao menor nmero inteiro maior ou igual a r. Definio 1.9: Chama-se nint de um nmero real r, o valor inteiro mais prximo de r. Para evitar ambigidades, no caso de valores de r iguais metade de um inteiro, convenciona-se arredondar o valor de nint sempre para o inteiro par. Notao: Usaremos as seguintes notaes:
r = piso de r r = teto de r r

= nint de r

Assim, o piso e o teto de um nmero real r so os inteiros definidos pelas desigualdades:


r 1 r r r r 1

Em linguagem da Teoria dos Conjuntos:


r max{n | n r} e r min{n | n r}

Observe que r r r se, e somente se, r um nmero inteiro, e que todo nmero real r pode ser escrito sob a forma:
r r k , onde 0 k r r 1

e
r r 1 k , onde 0 k r r 1 1

O nmero real k chama-se parte no-inteira de r.

21

CAPTULO 1 NMEROS INTEIROS NOES FUNDAMENTAIS

Exemplos: piso e teto de r. a)


2 1 e 2 2

d)

1 3

0 e

1 3

b)

3 e

e)

1 2

1 e

1 2

c)

3 2

2 e

3 2

f)

7 e

Exemplos: nint de r.

a) [2,3] = 2 e [2,7] = 3

d) [3,5] = 4 e [4,5] = 4

b)

1 3

23 6

e)

1 2

1,5

c) [ ] = 3

e [e] = 3

f) [-3, 4] = -3 e [-3, 7] = -4

22

CAPTULO 1 NMEROS INTEIROS NOES FUNDAMENTAIS

Abaixo, esto ilustrados os grficos das funes piso, teto e nint, respectivamente.

f ( x)

f ( x)

23

CAPTULO 1 NMEROS INTEIROS NOES FUNDAMENTAIS

f ( x)

1.11 O Princpio da Casa dos Pombos (Princpio das Gavetas de Dirichlet)


O princpio da Casa dos Pombos a afirmao de que se n pombos devem ser postos em m casas, sendo n > m ento pelo menos uma casa ir conter mais de um pombo. tambm conhecido como Princpio das Gavetas de Dirichlet, acredita-se que o primeiro relato deste principio foi feito pr Dirichlet em 1834, com o nome de Schubfachprinzip ("Princpio das Gavetas"). O princpio da casa do pombo um exemplo de um argumento de calcular que pode ser aplicado em muitos problemas formais, inclundo aqueles que envolvem um conjunto infinito. Exemplo: Quantas pessoas so necessrias para se ter certeza que haver pelo menos duas delas faam aniversrio no mesmo ms? Resposta: 13 pessoas. Pelo princpio da casa dos pombos se houver mais pessoas (13) do que meses (12) certo que pelos menos duas pessoas tero nascido no mesmo ms. Embora o princpio da casa dos pombos seja uma observao trivial, pode ser usado para demonstrar resultados possivelmente inesperados . Por exemplo, em toda grande cidade, digamos com mais de 1 milho de habitantes existem pessoas com o mesmo nmero de fios de cabelo. Demonstrao: Tipicamente uma pessoa tem cerca de 150 mil fios de cabelo. razoavel supor que ningum tem mais de 1.000.000 de fios de cabelo em sua cabea. Se h mais habitantes do que o nmero mximo de fios de cabelo, necessariamente pelo menos duas pessoas tero exatamente o mesmo nmero de fios de cabelo.

24

CAPTULO 1 NMEROS INTEIROS NOES FUNDAMENTAIS

1.12 Caos Fatorial: !n.


Suponha que queremos calcular todos os anagramas da palavra ESCOLA, de modo que nenhuma letra ocupe o seu lugar original, ou primitivo. Um deles seria SEOCAL, uma vez que nenhuma letra ocupa seu lugar inicial. Esse tipo de permutao chamada de catica ou desordenada e o caos fatorial n ( tambm chamado de subfatorial ou derangements em Ingls), simbolizado por !n , usado para calcular o nmero dessas permutaes caticas. Lembre-se que o fatorial calcula o total de permutaes de um conjunto.

Definio 1.11.: Chama-se caos fatorial de um inteiro no negativo n ( n 0 ), o inteiro que se indica por !n, e tal que:

!n n!
Para n 1 , temos:

1 1 1 1 ( 1)n ... 0! 1! 2! 3! n! 1 1 ( 1)n ... 2! 3! n!


n

n!
k

( 1)n 0 k!

!n n!
Pode-se provar que !n
n! . e

n!
k

( 1)n 2 k!

M. Hassani deu outras formas para o caos fatorial:


!n n! 1 ,n 1 e

!n
Os 10 primeiros valores !n, so: n 0 1 2 3 4 5 6 7 8 9 10

e e

n!

en! , n 1

n! 1 0 1 2 9 44 265 1854 14833 133496 1334961

25

CAPTULO 1 NMEROS INTEIROS NOES FUNDAMENTAIS

Voltando ao problema comentando no incio, podemos afirmar que o nmero de permutaes caticas da palavra ESCOLA !6 = 265.

Exemplos:
a) !6 6!
1 1 1 2! 3! 4! 9 1 1 !6 6! 24 5! 6! 6.5.4!.9 6.5! !6 4! 5! !6 265 1 1 5! 6! 9 6! 4! 6! 270 6! 1 1 2 6 1 1 5! 6! 6!
6 1

1 24

1 1 5! 6!

b) !6

6! e 6! 1 e

720 2,718... 721 2,718...

264,87...

265

c) !6

265, 241...

265

d) !6
!6

2,718...
2221,92

1 .720 2,718...
1956,96

2,718... .720
265

3,086... .720

2, 718... .720

2221 1956

1.13 Left Fatorial: L!n

Dura Kurepa , em 1971 publicou a o conceito de L!n, o left factorial, definido como
n 1

L !n

0! 1! 2! ... ( n 1)!
k 0

k!

Um famoso problema em aberto na Teoria dos Nmeros, uma conjectura feita por Kurepa de que o MDC (n!, L!n) = 2 para todo n maior que 1. Abaixo colocamos os 10 primeiros valores do left fatorial. Por definio, L!0 = 0.
n 0 1 2 L!n 0 1 2

26

CAPTULO 1 NMEROS INTEIROS NOES FUNDAMENTAIS

3 4 5 6 7 8 9 10

4 10 34 154 874 5914 46234 409114

O left fatorial sempre par para qualquer inteiro maior que 1. Se dividirmos o left fatorial por 2, obtemos alguns valores primos. Veja L !n n 2 3 2 4 5 5 17 8 2957 9 23117 10 204557 L !n Uma questo em aberto saber se existem infinitos primos da forma . 2

EXERCCIOS
1) Sem usar P.A., calcule a soma dos n primeiros inteiros positivos. 2) Calcular o inteiro positivo n, sabendo que 3n+2 . 2n+3 = 2592. 8) Decompor o inteiro 565 numa soma de cinco inteiros mpares consecutivos. 9) Achar todas as solues inteiras e positivas da equao (x + 1)(y + 2) = 2xy. 10) Determinar todos os inteiros positivos de dois algarismos que sejam igual ao qudruplo da soma dos seus algarismos. 11) Achar o menor e o maior inteiro positivo de n algarismos. 12) Resolva a equao: (x + 2)! = 72.x! 13) Resolver a equao:

3) Calcule o inteiro positivo n, sabendo-se que: 3n + 3n+1 + 3n+2 + 3n+3 = 1080. 4) Com uma calculadora, achar os valores de n < 10 para os quais n! + 1 um quadrado perfeito. 5) Sendo m e n inteiros positivos, dizer se verdadeiro ou falso: a) (mn)! = m!. n! b) (m + n)! = m! + n! 6) Demonstrar: (n 1)! [(n + 1)! n!] = (n!)2 7) Sendo n > 2, demonstrar: (n2)! > (n!)2.

7 x
2

7 x 2x 2

14) Demonstrar :

n k

n k 1 n k 1 k

27

CAPTULO 1 NMEROS INTEIROS NOES FUNDAMENTAIS 15) Achar todas as solues inteiras e positivas da equao: x2 y2 = 88.; 16) Verificar se o quadrado de um inteiro pode terminar em 2, 3, 7 ou 8. 17) Hilbert escreveu os inteiros de 1 at 1000 (inclusive), em ordem decrescente. Sem usar P.A, determine qual foi o 3330 inteiro escrito? 18) Calcular o nmero de algarismos necessrios para ser escrever os nmeros positivos de 1, 2. 3, 4, ......, n algarismos. 19) O produto de um inteiro positivo de trs algarismos por 7 termina direita por 638. Achar esse inteiro. 20) Determinar quantos algarismos se emprega para numerar todas as pginas de um livro de 2748 pginas. 21) Dois homens estavam conversando num bar quando um virou para o outro e disse: Tenho trs filhas a soma de suas idades igual ao nmero da casa em frente e o produto 36. Posso determinar as idades de suas filhas apenas com esses dados? No. Dar-lhe-ei um dado fundamental: minha filha mais velha toca piano. Determine a posio (linha e coluna) ocupada pelo nmero 107. 26) Mostrar que o produto de quatro algarismos consecutivos, aumentado de 1, um quadrado perfeito. 27) A soma dos quadrados de dois inteiros 3332 e um deles o qudruplo do outro. Achar os dois inteiros. 28) Escrever os inteiros de 1 a 1993, inclusive, quantas vezes o algarismo 1 escrito? 29) Determinar o inteiro n > 1 de modo que a soma 1! + 2! + 3! + ... + n! seja um quadrado perfeito. 30) A mdia aritmtica de dois inteiros positivos 5 e a mdia geomtrica 4. Encontre esses nmeros. 31) Achar cinco inteiros positivos consecutivos cuja soma dos quadrados igual a 2010. 32) O resto por falta da raiz quadrada de um inteiro positivo 135 e o resto por excesso 38. Achar esse inteiro. 33) Resolver a equao

x ! 3( x 2)! x ! 3( x 2)!

31 29

Determine as idades das filhas e o nmero da casa em frente. 22) Calcular a soma dos trs maiores nmeros inteiros de, respectivamente, trs, quatro e cinco algarismos. 23) Determinar a diferena entre o maior nmero inteiro com seis algarismos diferentes e o maior inteiro com cinco algarismos tambm diferentes.s 24) Um livro tem 1235 pginas. Determinar o nmero de vezes que o algarismo 1 aparece na numerao da pginas deste livro. 25) Os nmeros abaixo esto dispostos em linhas e colunas. 1 2 8 9 15 16 22 23 29 30

34) Achar o inteiro que deve ser somado a cada um dos inteiros 2, 6 e 14 para que, nesta ordem, formem uma proporo contnua. 35) Coloque em ordem crescente: .

260 ; 340 ; 720

36) Achar o valor mnimo de uma soma de 10 inteiros positivos distintos, cada um dos quais se escreve com trs algarismos. 37) O menor nmero natural n, diferente de zero, que torna o produto de 3888 por n um cubo perfeito : 38) Um estudante ao efetuar a multiplicao de 7432 por um certo inteiro achou o produto 1731656, tendo trocado, por engano, o algarismo das dezenas do multiplicador, tomando 3 em vez de 8. Achar o verdadeiro produto. 39) Achar o menor inteiro cujo produto por 21 um inteiro formado apenas por 4 algarismo.

28

CAPTULO 1 NMEROS INTEIROS NOES FUNDAMENTAIS 40) Escreve-se a seqncia natural dos inteiros positivos, sem separar os algarismos: 123456789101112131415... Determinar: a) o 435 algarismo Apertando um boto do bordo do retngulo, trocam de cor ele e seus vizinhos (do lado ou em diagonal). Apertando o boto do centro, trocam de cor todos os seus 8 vizinhos porm ele no. Exemplos: Apertando 1, trocam de cor 1, 2, 4 e 5. Apertando 2, trocam de cor 1, 2, 3, 4, 5 e 6. Apertando 5, trocam de cor 1, 2, 3, 4, 6, 7, 8 e 9. Inicialmente todos os botes esto verdes. possvel, apertando sucessivamente alguns botes, torn-los todos vermelhos? 50) Escrevemos abaixo os nmeros naturais de 1 a 10. 1 2 3 4 5 6 7 8 9 10.

b) o 1756 algarismo. c) o 12387 algarismo.

41) Escreve-se a seqncia natural dos inteiros positivos pares, sem separar os algarismos: 24681012141618... Determinar o 2574 algarismo que se escreve. 42) As representaes decimais dos nmeros 2 e 51999 so escritos lado a lado. O nmero de dgitos escritos igual a: 43) Mostrar que o produto de dois fatores entre 10 e 20 o dcuplo da soma do primeiro com as unidades do segundo mais o produto das unidades dos dois. 44) Achar o menor inteiro positivo que multiplicado por 33 d um produto cujos algarismos so todos 7. 45) Os inteiros a e b so tais que 4 < a < 7 e 3 < b < 4. Mostrar que 0 < a b < 4. 46) Os inteiros a e b so tais que 1 < a < 3 e 2 < b < 0. Mostrar que 1 < a b < 5. 47) Os inteiros a e b so tais que -2 < a < 2 e 2 < b < 2. Mostrar que 4 < a b < 4. 48) Em um quartel existem 100 soldados e, todas as noites, trs deles so escolhidos para trabalhar de sentinela. possvel que aps certo tempo um dos soldados tenha trabalhado com cada um dos outros exatamente uma vez? 49) Um jogo consiste de 9 botes luminosos (de cor verde ou vermelha) dispostos da seguinte forma:
1 4 2 5 3 6

1999

Antes de cada um deles, coloque sinais + ou de forma que a soma de todos seja zero. 51) Escrevemos abaixo os nmeros naturais de 1 a 11. 1 2 3 4 5 6 7 8 9 10 11

Antes de cada um deles, coloque sinais + ou de forma que a soma de todos seja zero. 52) Para numerar as pginas de um livro foram utilizados 663 algarismos. Quantas pginas tinha o livro? 53) Seja Q = 1! + 2! + 3! + ... + n!. Para quantos valores de n tem-se Q quadrado perfeito? 54) Quantos so os nmeros naturais de 4 dgitos que possuem pelo menos dois dgitos iguais? 55) Quantos so os nmeros de 5 algarismos, na base 10: a) Nos quais o algarismo 2 figura? b) Nos quais o algarismo 2 no figura? 56) Permutam-se de todos os modos possveis os algarismos 1, 2, 4, 6, 7 e escrevem-se os nmeros assim formados em ordem crescente. a) Que lugar ocupa o nmero 62417? b) Qual o nmero que ocupa o 66 lugar? c) Qual o 200 algarismo escrito? d) Qual a soma dos nmeros assim formados?

29

Captulo 2:

INDUO MATEMTICA
INTRODUO

s cincias naturais utilizam o mtodo chamado induo emprica para formular leis que devem reger determinados fenmenos a partir de um grande nmero de observaes particulares, selecionadas adequadamente. Esse tipo de procedimento, embora no seja uma demonstrao de que um dado fato logicamente verdadeiro, frequentemente satisfatrio. Por exemplo: ningum duvidaria de que quando um corpo liberado ao seu prprio peso, no vcuo, na superfcie da terra, ele cai segundo a vertical do local. A validade de um teorema matemtico se estabelece de forma totalmente diferente. Verificar que uma certa afirmao verdadeira num grande nmero de casos particulares no nos permitir concluir que ela vlida. Para demonstrar a verdade de uma sequncia infinita de proposies, uma para cada inteiro positivo, introduziremos o chamado mtodo de recorrncia ou induo matemtica.

2.1 Elemento mnimo de um conjunto de inteiros


Definio 2.1: Seja A um conjunto de inteiros. Chama-se elemento mnimo de A um elemento a A tal que a x para todo x A . Representa-se pela notao minA, que se l: mnimo de A. Portanto, simbolicamente: minA = a (a A e ( x
A) ( a
x ))

Teorema 2.1: Se a elemento mnimo de A, ento esse elemento nico. Demonstrao: Com efeito, se existisse um outro elemento mnimo b de A, teramos: i) a b , porque a = minA. ii) b a , porque b = minA..

30

CAPTULO 2 INDUO MATEMTICA

Logo, pela propriedade anti-simtrica da relao de ordem natural em Z, temos a = b. O elemento mnimo de A, se existe, denomina-se tambm primeiro elemento de A ou menor elemento de A Exemplo 2.1: O conjunto N = {1, 2, 3,...} dos inteiros positivos tem o elemento mnimo, que 1 (minN = 1), porque 1 N e 1 n para todo n N . Exemplo 2.2: O conjunto A x | x 12 13), porque 13 A e 13 x para todo x A . tem o elemento mnimo, que 13 (minA =

Exemplo 2.3: O conjunto 0, 1, 2, 3,... dos inteiros no positivos no tem o elemento mnimo, porque no existe a Z- tal que a x para todo x Z- . Exemplo 2.4: O conjunto A porque 3 A (3 divide 9) e 3

x |3divide x 2 tem o elemento mnimo 3 (min A = 3),


x para todo x A (1 Ae2 A).

2.2 Princpio da boa ordenao


Todo conjunto no vazio A de inteiros no negativos possui o elemento mnimo. Em outros termos, todo subconjunto no vazio A do conjunto

Z+ ={0 ,1, 2, 3, ...}


dos inteiros no negativos (

Z+ ) possui o elemento mnimo, isto , simbolicamente:


Z ,A
)
min A

( A

Exemplo 2.5: O conjunto A = {1, 3, 5, 7,...} dos inteiros positivos mpares um subconjunto no vazio de

Z+ (

Z+ ).

Logo, pelo Princpio da boa ordenao, A possui o elemento mnimo (minA = 1). Exemplo 2.6: O conjunto P = {2,3,5,7,11, ...} dos inteiros primos um subconjunto no vazio de Z+ ( P Z+). Logo, pelo Principio da boa ordenao, P possui o elemento mnimo (minP = 2). Teorema 2.2 (de Archimedes): Se a e b so dois inteiros positivos quaisquer, ento existe um inteiro positivo n tal que na b . Demonstrao:

31

CAPTULO 2 INDUO MATEMTICA

Suponhamos que a e b so dois inteiros positivos para os quais na b para todo inteiro positivo n. Ento, todos os elementos do conjunto: S = {b na | n N } so inteiros positivos e, pelo Princpio da boa ordenao, S possui o elemento mnimo,

digamos minS = b ka.


E como b (k + 1)a pertence a S, porque S contm todos os inteiros positivos desta forma, temos: b (k + 1) a = (b ka) a < b ka isto , b ka no o elemento mnimo de S, o que uma contradio. Logo, a propriedade archimediana verdadeira. Assim, por exemplo: i) se a = 2 e b = 11, ento n = 6, porque 6.2 > 11; ii) se a = 9 e b = 5, ento n =1, porque 1.9 > 5.

2.3 Princpio de Induo Finita.


Quando uma proposio enunciada em termos de nmeros naturais, o Princpio de induo finita constitui um eficiente instrumento para demonstrar a proposio no caso geral. Na prtica, o mtodo pode ser entendido por um artifcio muito simples. Vamos supor que temos uma srie de domins idnticos colocados em fila, que comea por um deles e prossegue indefinidamente. Nosso objetivo - empurrando apenas um domin - garantir que todos caiam. Como derrubar todos os domins? Para isso, basta nos assegurarmos de que: 1) O primeiro domin cai; 2) Os domins esto dispostos de tal modo que qualquer um deles - toda vez que cai -, automaticamente, empurra o domin seguinte e o faz cair tambm. Assim, mesmo que a fila se estenda indefinidamente, podemos afirmar que todos os domins cairo.

32

CAPTULO 2 INDUO MATEMTICA

Vamos estabelecer matematicamente esses procedimentos. Teorema 2.3 Seja S um subconjunto do conjunto N dos inteiros positivos ( S satisfaz as duas seguintes condies: i) 1 pertence a S ( 1 S ); ii) para todo inteiro positivo k, se k S , ento (k 1) S . Nestas condies, S o conjunto N dos inteiros positivo: S = N. Demonstrao: Suponhamos, por absurdo, que S no o conjunto N dos inteiros positivos ( S conjunto de todos os inteiros positivos que no pertencem a S, isto : X = {x | x N e x S } = N S Ento, X um subconjunto no vazio de N ( X N ) e, pelo Princpio da boa ordenao, existe o elemento mnimo x0 de X (minX = x0 ). Pela primeira condio, 1 S , de modo que x0 > 1 e, portanto, x0 - 1 um inteiro positivo que no pertence a X. Logo, (x0 - 1) S e, pela segunda condio, segue-se que ( x0 - 1) + 1 = x0
S , o que uma contradio, pois, x0
N ) e seja X o N ) que

N S , isto , x0

S . Assim sendo, X

eS

= N. Consoante este Princpio de induo finita, o nico subconjunto de N que satisfaz s duas condies o prprio N.

2.4 Induo Matemtica


Em matemtica, concluses como as que se obtm a seguir so inadmissveis. Por qu? Em que pecam os raciocnios utilizados? Vamos examin-los... 1) Suponha que desejemos obter uma frmula que d o valor da soma Sn = 1 + 3 + 5 + 7 + ... + (2n - 1), para qualquer inteiro positivo de n. fcil ver que: n=1 n=2 n=3 n=4 S1 = 1 = 12; S 2 = 1 + 3 = 4 = 22 ; S 3 = 1 + 3 + 5 = 9 = 32 S4 = 1 + 3 + 5 + 7 = 16 = 42

33

CAPTULO 2 INDUO MATEMTICA

Por meio de um raciocnio indutivo, os resultados obtidos nos levam a afirmar que para todo inteiro positivo n tem-se Sn = n2. 2) Consideremos o trinmio P(n) = n2 + n + 41. Considerando n = 0, obtemos P(0) = 41, que um nmero primo. Substituindo n por 1, chegamos a outro nmero primo, o 43. Substituindo sucessivamente n por 2, 3, 4, 5, 6, 7, 8, 9 e 10, conseguimos como resultados outros nmeros primos (47, 53, 61, 71, 83, 97, 113, 131 e 151, respectivamente). Ento, os resultados obtidos nos induzem a afirmar que, para todo n natural, o trinmio P(n) = n2 + n + 41, sempre produz como resultado um nmero primo. Nos dois exemplos, props-se um resultado geral, supostamente vlido para todo n, com base no fato de que ele correto para alguns valores particulares de n: tal procedimento, entretanto, pode conduzir a concluses falsas. Assim, ainda que em no primeiro caso a proposio geral enunciada resulte correta - por mero acaso! -, a proposio geral do segundo exemplo falsa. De fato, P(n) gera nmeros primos para n= 0, 1, 2, 3, ..., 39, mas para n = 40, ele vale 412, que no um nmero primo. Portanto, no exemplo 2), encontramos uma proposio que - apesar de vlida em 40 casos particulares - no vlida em geral. Note bem: Uma proposio pode ser vlida em uma srie de casos particulares, mas, mesmo assim, no o ser de maneira geral. Coloca-se, ento, o seguinte problema: temos uma proposio que se mostrou correta em muitos casos particulares. No entanto, impossvel verificar todos os casos particulares. Assim sendo, como podemos saber se a proposio correta de modo geral? O Teorema abaixo, esclarece essa questo. Teorema 2.4: Seja P(n) uma proposio associada a cada inteiro positivo n e que satisfaz s duas seguintes condies: i) P(1) verdadeira;

ii) para todo inteiro positivo k, se P(k) verdadeira, ento P(k + 1) tambm verdadeira. Nestas condies, a proposio P(n) verdadeira para todo inteiro positivo n.

Demonstrao: Seja S o conjunto de todos os inteiros positivos n para os quais a proposio P(n) verdadeira, isto : S = { n N | P(n) verdadeira} Pela primeira condio, P(1) verdadeira e, portanto, 1 S . Pela segunda condio, para todo inteiro positivo k, se k S , ento (k 1) S . Logo, o conjunto S satisfaz s duas condies do Princpio de induo finita e, portanto, S = N, isto , a proposio P(n) verdadeira para todo inteiro positivo n.

34

CAPTULO 2 INDUO MATEMTICA

Nota: O teorema 2.4 geralmente denominado Teorema da induo matemtica ou Princpio de induo matemtica, e a demonstrao de uma proposio usando-se este teorema chama-se demonstrao por induo matemtica ou demonstrao por induo sobre n. Na demonstrao por induo matemtica de uma dada proposio P(n) obrigatrio verificar que as condies i e ii so ambas satisfeitas. A verificao da condio i geralmente muito fcil, mas a verificao da condio ii implica em demonstrar o teorema auxiliar cuja hiptese : H: proposio P(k) verdadeira, k N . denominada hiptese de induo, e cuja tese ou concluso : T: proposio P(k + 1) verdadeira.

2.5. Exemplos de demonstrao por Induo Matemtica


Exemplo 2.7: Demonstrar a proposio: P(n): 1 + 3 + 5 + ... + (2n 1) = n, Demonstrao: i) P(1) verdadeira, visto que 1 = 1.
n N

ii) A hiptese de induo que a proposio: P(k): 1 + 3 + 5 + ... + (2k 1) = k, k N verdadeira. Adicionando (2k + 1) a ambos os membros desta igualdade, obtemos: 1 + 3 + 5 + ... + (2k 1) + (2k + 1) = k + (2k + 1) = (k + 1) e isto significa que a proposio P(k + 1) verdadeira. Logo, pelo Teorema da induo matemtica, a proposio P(n) verdadeira para todo inteiro positivo n.

35

CAPTULO 2 INDUO MATEMTICA

Exemplo 2.8: Demonstrar a proposio:

P(n) :

1 1.2

1 1 1 ... 2.3 3.4 n(n 1)

n n 1

, n N

Demonstrao:

1 1 1.2 1 1 2) A hiptese de induo que a proposio:


1) P(1) verdadeira, visto que

P(k) :

1 1.2

1 1 1 ... 2.3 3.4 k(k 1)

k k 1

,k

verdadeira. Adicionando

1 k 1 k 2

a ambos os membros desta igualdade, obtemos:

1 1.2 k

1 1 1 ... 2.3 3.4 k(k 1) 1 k 1 k 2

1 k 1 k 2 k 1 k 2

k 1

k 2 2k 1 (k 1) k 2

e isto significa que a proposio P k 1 verdadeira. Logo, pelo Teorema da induo matemtica, a proposio P n verdadeira para todo inteiro positivo n. Exemplo 2.9: Demonstrar a proposio:

P(n) : 3| 22n 1 ,
Demonstrao: 1) P (1) verdadeira, visto que 3 | 22 1 . 2) A hiptese de induo que a proposio: P k : 3 | 22k 1 , k N verdadeira. Portanto: 22k 1 = 3q, com q Z

n N

36

CAPTULO 2 INDUO MATEMTICA

o que implica:

22 k 1

1 22.22k 1 4.22k 1 4.22k 4 4 1 4 22k 1 4.3q 3 3(4q 1) 3

isto , a proposio P k 1 verdadeira. Logo, pelo teorema da induo matemtica, a proposio P n verdadeira para todo inteiro positivo n. Exemplo 2.10: Demonstrar a proposio:

P(n) : 2n n,
Demonstrao: 1) P(1) verdadeira, visto que 2 = 2 > 1. 2) A hiptese de induo que a proposio: P(k): 2k

n N

k, k

verdadeira. Portanto: 2.2k > 2k ou 2k+1 > k + k k+1

o que implica: 2k 1 k 1 , isto , a proposio P(k+1) verdadeira. Logo, pelo Teorema da induo matemtica, a proposio P(n) verdadeira para todo inteiro positivo n.

2.6 . Outras formas da induo matemtica


Teorema 2.5 Seja r um inteiro positivo fixo e seja P(n) uma proposio associada a cada inteiro n r e que satisfaz s duas seguintes condies: i) P(r) verdadeira; ii) para todo inteiro k

r, se P(k) verdadeira, ento P(k + 1) tambm verdadeira. r..

Nestas condies, P(n) verdadeira para todo inteiro n

37

CAPTULO 2 INDUO MATEMTICA

Demonstrao: Seja S o conjunto de todos os inteiros positivos n para os quais a proposio P(r + n 1) verdadeira, isto : S = {n N | P(r + n 1) verdadeira} S. E, pela segunda

Pela primeira condio, P(r) = P(r + 1 1) verdadeira, isto , 1 condio, se P(r + k 1) verdadeira, ento: P((r + k 1) + 1) = P(r + (k + 1) 1)

tambm verdadeira, isto , se k S, ento (k + 1) S. Logo, pelo Princpio da induo finita, S o conjunto dos inteiros positivos: S = N, isto , a proposio P(r + n 1) verdadeira para todo n N , ou seja, o que a mesma coisa, a proposio P(n) verdadeira para todo inteiro n r . Exemplo 2.11: Demonstrar a proposio: P(n): 2n Demonstrao: 1) P(4) verdadeira, visto que 24 16 4! 24 . 2) Suponhamos, agora, que verdadeira a proposio: P(k): 2k Ento, por ser 2 < k + 1 para k multiplicando termo a termo ( I ) e ( II ):
4 ( II ),

n!,

k !, k

4 (I)

2k

k !.(k 1) ou 2k

(k 1)!

isto , a proposio P(k + 1) verdadeira. Logo, pelo teorema 2.5, a proposio P(n) verdadeira para todo inteiro n 4 .

Observe-se que a proposio P(n) falsa para n = 1, 2, 3, pois, temos: 2 > 1! , 2 > 2! , 2 > 3! Exemplo 2.12: Demonstrar a proposio:

38

CAPTULO 2 INDUO MATEMTICA

P(n) : n2 > 2n + 1, Demonstrao: 1) P (3) verdadeira, visto que 32 = 9 > 2. 3 + 1= 7. 2) Suponhamos, agora, que verdadeira a proposio:

P(k) : k2 > 2k + 1, k Ento, temos:

k2 + (2k+ 1) > (2k+1) + (2k+1) ou (k +1)2 > 2 (k + 1) + 2k > 2 (k + 1) + 2 > 2 (k +1) + 1, k e, portanto: (k +1)2 > 2 (k +1) + 1, k 3. 3

Isto , a proposio P k 1 verdadeira. Logo, pelo teorema 2.5, a proposio P n verdadeira para todo inteiro n 3 . Observa-se que a proposio P n falsa para n = 1 e n = 2, pois, temos: 12 < 2.1+1 e 22 < 2.2 + 1 Teorema 2.6 Seja P(n) uma proposio associada a cada inteiro positivo n e que satisfaz s duas seguintes condies: i) P(1) verdadeira; ii) para todo inteiro positivo k, se P(1), P(2),..., P(k) so todas verdadeiras, ento P(k + 1) tambm verdadeira. Nestas condies, a proposio P(n) verdadeira para todo inteiro positivo n.

39

CAPTULO 2 INDUO MATEMTICA

Demonstrao: Seja S o conjunto de todos os inteiros positivos n para os quais a proposio P(n) verdadeira, isto : S = { n N | P(n) verdadeira} Suponhamos por absurdo, que S N e seja X o conjunto de todos os inteiros positivos que na pertencem a S, isto : X = {x | x N e x S } = N S Ento, X um subconjunto no vazio de N e, pelo Princpio da boa ordenao, existe o elemento mnimo j de X (minX = j). Pela primeira condio, 1 S , de modo que j > 1, e como j o menor inteiro positivo que no pertence a S, segue-se que as proposies P(1), P(2),..., P(j 1) so todas verdadeiras. Ento, pela segunda condio, a proposio P(j) verdadeira e j S , o que uma contradio, pois j X , isto , j S . Assim sendo, S = N e a proposio P(n) verdadeira para todo inteiro positivo n. Teorema 2.7 Seja r um inteiro positivo fixo e seja P(n) uma proposio associada a cada inteiro n r e que satisfaz s duas seguintes condies: 1) P(r) verdadeira; 2) para todo inteiro k > r, se P(m) verdadeira para todo inteiro m tal que r P(k) verdadeira. Nestas condies, a proposio P(n) verdadeira para todo inteiro n r .
m k , ento

Demonstrao: Seja S o conjunto de todos os inteiros n r para os quais a proposio P(n) falsa, isto : S = { n N | n r e P(n) falsa} Suponhamos, por absurdo, que S no vazio ( S ). Ento, pelo Princpio da boa ordenao, existe o elemento mnimo j de S (minS = j). Pela primeira condio, r S , de modo que j > r, e, por conseguinte P(m) verdadeira para todo inteiro m tal que r m j . Assim sendo, pela segunda condio, P(j) verdadeira e ), e a j S , o que uma contradio, pois, j S . Logo, o conjunto S vazio ( S proposio P(n) verdadeira para todo inteiro n r .

40

CAPTULO 2 INDUO MATEMTICA

Nota Histrica

Era ideia assente na comunidade matemtica do sculo XIX, que a induo era obra do matemtico francs Blaise Pascal , tendo em conta diversas demonstraes que apresenta no seu Trait du Triangle Arithmtique. Essa situao seria integralmente modificada, vinte anos aps a formulao moderna de induo matemtica fixada por Giuseppe Peano , quando Giovanni Vacca , em 1909, num artigo de trs pginas publicado no Bulletin of American Mathematical Society, vem defender que o italiano Francesco Maurolico , pelos trabalhos que desenvolveu no primeiro livro de aritmtica includo na sua Opuscula Mathematica, escrita em 1557 e publicado em Veneza no ano de 1575, como "the first discoverer of the principle of mathematical induction". O artigo de Vacca encontrou eco, ainda que eventualmente sem verificao posterior, em autores importantes como Moritz Cantor ou Siegmund Gnther . M. Cantor, por exemplo, que atribuiu inicialmente a Pascal a principal origem do mtodo de induo completa (em Vorlesungen uber Geschichte der Mathematik, vol. 2, p. 749), viria a transferir esse atributo para Maurolico (em Zeichrift fur Mathematischen und Naturwissenschaftlichen Unterricht, vol 33, 1902, p. 536), segundo conta devido a uma informao oral que lhe foi prestada pelo prprio Vacca. Passar-se-iam mais de quarenta anos sem que o artigo de Vacca fosse alvo de qualquer crtica. At que Hans Freudenthal (em Zur Geschichte der vollstndigen Induktion, Archive Internationale d'Histoire des Sciences 6 (1953) 17-37) depois de um exame detalhado dos trabalhos de Maurolico, vem sustentar que em apenas trs pontos conseguiu reconhecer uma certa forma de induo matemtica: uma forma arcaica, contudo, ao contrrio do que observou em Pascal, onde a induo formulada pela primeira vez de uma maneira abstrata.

41

CAPTULO 2 INDUO MATEMTICA

EXERCCIOS
1) Demonstrar por "induo matemtica": c) 5 | (8n 3n) n n n n N N. N

n (n 1)(2n 1) a) 1 + 2 + 3 + ... + n = 6
2 2 2 2

d) 24 | (52n 1) e) 7 | (2 1) 8 | 32n + 7,
3n

n
3

N
2 2
3 3 3

f) n

b)

n (n 1) 1 + 2 + 3 + ... + n = 4
N 12 + 32 + 52 + ... + (2n 1)2 =

4) Demonstrar que 10n + 1 9n 10 um mltiplo de 81 para todo inteiro positivo n

c)

n ( 4n 3

1)

5) Demonstrar que n N

n3 3

n5 5
N

7n um inteiro 15

positivo para todo n

d) 13 + 33 + 53 + ... + (2n 1)3 = n2(2n2 1) 6) Prove que, para todo inteiro n e) 1.2 + 2.3 + 3.4 + ... + n(n + 1) =

1 , o nmero

n (n 1)( n 3
f)

2)
1 2 1 1 3 ... 1 1 n n 1

an

inteiro e mpar.
n

1 1 1
, n N

7) Para n

1, mostre que S n
k 1

k ! um

inteiro mpar. g) a + aq + aq2 + ...+aqn =

a (q n 1 1) ,q 1 q 1

8) Para n 0 , mostre que an 11n um inteiro divisvel por 133.

122n

2) Demonstrar por "induo matemtica" a) 2n < 2n+1 n


n 2

N 5 5

9)

Para n a) b)

3 , mostre que
n

b) 2 > n c)

n 1 n!
2

nn nn . .

4n > n4 n
n 3 2

d) 2 > n e) f) g)

n n n

10 4 6 10) Mostre que sempre possvel pagar, sem receber troco, qualquer quantia inteira de $, maior que $7, com notas de $3 e $5. N

n!>n

n! > n3

1 4

1 9

...

1 n2

1 , n n

3) Demonstrar por "induo matemtica" a) 2 | (3n 1) n N N

b) 6 | (n3 n) n

42

Captulo 3:

SOMATRIOS E PRODUTRIOS
3.1 . Somatrios
Sejam os n > 1 inteiros a1,a 2 ,...,a n . Para indicar, de modo abreviado, a soma a1 a 2 ... a n desses n inteiros usa-se a notao:
n

ai
i 1

que se l: somatrio de a i de 1 a n. Em particular, para n = 2, 3,..., temos:


2 3

ai
i 1

a1 a 2 ,
i 1

ai

a1 a 2

a 3 , ...

A letra i chama-se o ndice do somatrio e pode ser substituda por qualquer outra diferente de a e de n um ndice mudo. E os inteiros 1 e n que figuram abaixo e acima da letra grega maiscula (sigma) chamam-se respectivamente limite inferior e limite superior do ndice i. O nmero de parcelas de um somatrio sempre igual diferena entre os limites superior e inferior do seu ndice mais uma unidade. Se m e n so dois inteiros, com m n , ento, por definio:
n

ai
i m

am am

am

...a n

Exemplo 3.1: Temos:


7

5i
i 1

5.1 5.2 5.3 5.4 5.5 5.6 5.7 5 10 15 20 25 30 35 140

43

CAPTULO 3 SOMATRIOS E PRODUTRIOS


4

8j 3
j 1

8.1 3

8.2 3 68

8.3 3

8.4 3

5 13 21 29
8

k .2k
k 3

3.23

4.44

5.25

6.26

7.27

8.28

24 64 160 384 896 2048 3576

Exemplo 3.2: Temos:


6

2 4 8 16 32 64
i 1

2i

15

1 3 5 ... 29
j 1

2j 1

3.2. Propriedades dos somatrios


n n n

Teorema 3.1:
i 1

(a i

bi )
i 1

ai
i 1

bi

Demonstrao: Com efeito, desenvolvendo-se o primeiro membro, temos:


n

(a i bi ) (a1 b1 ) (a 2 b 2 ) ... (a n b n )
i 1 n n

(a1 a 2 ... a n ) (b1 b 2 ... b n )


i 1

ai
i 1

bi

Teorema 3.2
i 1

na

Demonstrao: Seja ai

a para i = 1, 2,..., n. Ento, temos:


n n

a
i 1 i 1

ai

a1 a 2 ... a n

a a ... a

na

Teorema 3.3
i 1

(a i

a)
i 1

ai

na

Demonstrao:

44

CAPTULO 3 SOMATRIOS E PRODUTRIOS

Consoante os dois teoremas anteriores, temos:


n n n n

(a i
i 1

a)
i 1

ai
i 1

a
i 1

ai

na

Teorema 3.4
i 1

ka i

k
i 1

ai

Demonstrao: Com efeito, desenvolvendo o primeiro membro, temos:


n n

ka i
i 1

ka1

ka 2 ... ka n

k(a1 a 2 ... a n )

k
i 1

ai

20

Exemplo 3.3: Calcular


i 1

(5i 2)

Consoante os teoremas anteriores temos, sucessivamente:


20 20 20 20

(5i 2)
i 1 i 1

5i
i 1

5
i 1

i 20.2

5(1 2 ... 20) 40

1 5. (1 20)20 40 2

5.210 40 1090

3.3. Produtrios
Sejam os n > 1 inteiros a1,a 2 ,...,a n . Para indicar, de modo abreviado, o produto a1a 2 ...a n desses n inteiros usa-se a notao:
n

ai
i 1

que se l: produtrio de a i de 1 a n. Em particular, para n = 2, 3,..., temos:


2 3

ai
i 1

a 1a 2 ,
i 1

ai

a1a 2 a 3 , ...

45

CAPTULO 3 SOMATRIOS E PRODUTRIOS

A letra i chama-se o ndice do produtrio e pode ser substituda por qualquer outra diferente de a e de n um ndice mudo. E os inteiros 1 e n que figuram abaixo e acima da letra grega maiscula (pi) chamam-se respectivamente limite inferior e limite superior do ndice i. O nmero de fatores de um produtrio sempre igual diferena entre os limites superior e inferior do seu ndice mais uma unidade. Se m e n so dois inteiros, m n , ento, por definio:
n

ai
i m

a m .a m 1.a m 2 ...a n

Exemplo 3.4: Temos:


6

3i
i 1

3.1 3.2 3.3 3.4 3.5 3.6

=3.6.9.12.15.18=524880
4

5j 3
j 1

5.1 3 5.2 3 5.3 3 5.4 3 =2.7.12.17 2856

Exemplo 3.5: Temos:


6

3.9.27.81.243.729
i 1

3i

16

1.3.5.7....31
j 1

2j 1
n

1.2.3... n 1 n

n!=
i=1

3.4. Propriedades dos Produtrios


n n n

Teorema 3.5
i 1

a i bi
i 1

ai.
i 1

bi

Demonstrao: Com efeito, desenvolvendo o primeiro membro, temos:


n n n

a i bi
i 1

(a1b1 )(a 2 b 2 )...(a n b n )

(a 1a 2 ...a n )(b1b 2 ...b n )


i 1

a i.
i 1

bi

46

CAPTULO 3 SOMATRIOS E PRODUTRIOS


n

Teorema 3.6
i 1

an

Demonstrao: Seja ai

a para i = 1, 2,..., n. Ento, temos:


n n

a
i 1 i 1

ai

a1a 2 ...a n

a.a...a

an

Teorema 3.7
i 1

ka i

kn
i 1

ai

Demonstrao: Com efeito, desenvolvendo o primeiro membro, temos:


n n n n i 1

ka i
i 1
4

(ka1 )(ka 2 )...(ka n ) k (a1a 2 ...a n ) k

ai

Exemplo 3.6: Calcular


i 1

(2i 1)

Consoante o teorema 3.7, temos:


4 4

(2i 1)
i 1 i 1 n n

(2i 1) (3.5.7.9) 945 893025


n

Exemplo 3.7: Demonstrar


i, j 1

a ij
i 1 j 1

a ij

Com efeito, desenvolvendo o primeiro membro, temos:


n

a ij (a11a12 ...a1n )(a 21a 22 ...a 2n )...(a n1a n2 ...a nn )


i, j 1 n n n n n

a1j.
j 1 j 1

a 2 j...
j 1

a nj
i 1 j 1

a ij

47

Captulo 4

DIVISIBILIDADE
Um conceito chave em Teoria dos Nmeros o conceito de divisibilidade. Existem muitos aspectos interessantes referentes diviso de nmeros inteiros. Antes que possam ser analisados, necessrio que conceitos bsicos como divisor e divide estejam bem estabelecidos.

4.1. RELAO DE DIVISIBILIDADE EM Z


Definio 4.1: Sejam a e b dois inteiros, com a existe um inteiro q tal que b = aq 0. Diz-se que a divide b se, e somente se,

Se a divide b tambm se diz que a divisor de b, que b mltiplo de a, que a um fator de b ou que b divisvel por a. Notao: a | b ( a divide b) Observao: Se a | b , ento a | b Teorema 4.1: Quaisquer que sejam os inteiros a, b e c tem-se: 1) a | 0 a 0, 1|a e a|a a 1 0

2) Se a | 1 , ento a =

3) Se a | b e se c | d , ento ac | bd 4) Se a | b e se b | c , ento a | c 5) Se a | b e se b | a , ento a = 6) Se a | b com b 0 , ento | a | b |b|

7) Se a | b e se a | c , ento a |(bx + cy) para todo x e y em Z

48

CAPTULO 4 DIVISIBILIDADE

Demonstrao: Em todas as demonstraes estaremos aplicando a Definio 1 e considerando aceitas todas as propriedades operatrias dentro do conjunto . 1) De fato: 0 = a.0; a = 1.a; a = a.1

2) De fato, se a|1, ento 1 = a.q, o que implica a = 1 e q = 1 ou a = -1 e q = -1, ou seja: a = 1. 3) De fato,


a|b b a.q

c|d
Portanto:

c.q1

bd
4) De fato:

ac.(q.q1 )

ac | bd

a|b

a.q

b|c
Logo, c a.(q.q1 ) 5) De fato:
a|b

c b.q1

a|c.

a.q

b|a
Logo:
a a(qq1 ) qq1 1

a b.q1

q1 |1

q1

6) De fato, nas condies da proprieade, temos:


a|b b a.q, ou seja | b | | a | .| q |

Como q 0 , temos que | q | 1 , desse modo temos | b | | a | . 7) De fato:


a|b b a.q

a|c

c a.q1

49

CAPTULO 4 DIVISIBILIDADE

Logo, quaisquer que sejam os inteiros x e y:


bx cy aqx aq1 y a(qx q1 y) a | (bx cy)

Esta propriedade (7) pode ser generalizada; ou seja, se

a | bk , k 1,2,3,..., n
ento, quaisquer que sejam os inteiros

x1, x2 ,..., xn
temos:

a | (bx1 bx2 ... bxn )

De acordo com as propriedades (1) e (4), a relao de divisibilidade em reflexiva e transitiva, mas no simtrica.

4.2. Conjunto dos divisores de um inteiro


O conjunto de todos os divisores de um inteiro qualquer a indica-se por D(a) = {x Z* | x | a }

imediato que, para todo inteiro a, se tem D(a) = D(-a). Qualquer que seja o inteiro a 0 , se x | a , ento:
a x a D(a) [ | a |,| a |]

significando que qualquer inteiro a 0 tem um nmero finito de divisores.

4.3. Divisores comuns de dois inteiros


Definio 4.2 Chama-se divisor comum de dois inteiros a e b todo inteiro d d | a e d | b. 0 tal que

50

CAPTULO 4 DIVISIBILIDADE

Notao: D(a,b) = { x Obs.: D(a,b)

| x | a e x | b} ou seja, D(a,b) = D(a)

D(b)

; D(0,0) = *

Exemplo 4.1: Sejam os inteiros a = 12 e b = -15. temos:

D 12 D 15
Portanto:
D 12, 15

1, 2, 3, 4, 6, 12 1, 3, 5, 15

D 12

15

1, 3

4.4. Teorema da Diviso


O Teorema da diviso, que veremos a seguir, usado por Euclides no seu livro Elementos, estabelece uma diviso com resto. um teorema que foi "provado" uma vez atravs de um algoritmo que explica como se processa a diviso, por esse motivo ficou conhecido como Algoritmo de Euclides. Teorema 4.2 Se a e b so dois inteiros, com b > 0, ento existem e so nicos os inteiros q e r que satisfazem s condies: a = bq + r e 0 r < b Demonstrao:

Existncia Seja S o conjunto de todos os inteiros no-negativos que so da forma a bx, com x , isto : S = {a bx ; x , a bx 0 }

Este conjunto S no vazio, porque, sendo b > 0, temos b resulta: a bx = a + b |a | a+|a| 0

1 e, portanto, para x = - | a |,

Assim sendo, pelo Princpio da boa ordenao, existe o elemento mnimo r de S tal que 0 r e r = a bq ou a = bq + r, com q

51

CAPTULO 4 DIVISIBILIDADE

Alm disso, temos r < b, pois, se fosse r b, teramos: 0 r b = a bq b = a b( q+1 ) < r

isto , r no seria o elemento mnimo de S. Unicidade Para demonstrar a unicidade de q e r, suponhamos que existem dois outros inteiros q 1 e r1 tais que a = bq1 + r1 e 0 Ento, teremos: bq1 + r1 = bq + r por outro lado, temos: -b<-r o que implica: - b < r r1 < b, isto | r1 r | < b Assim, b | (r1 r) e | r1 r | < b e, portanto: r1 r = 0, e como b q q1 0. Logo, r1 = r e q1 = q. 0 e 0 r1 < b r1 r = b(q q1) b | (r1 r) r1 < b

0, tambm temos

Nota: Aqui cabe uma pergunta: Por que o resto deve ser positivo? A resposta
simples: Quando se divide a por b, o que se procura o maior mltiplo de b que menor do que a, de modo que se a =b q + r, ento r = a bq positivo porque a.b menor do que a. Mas, no poderamos definir diviso de modo que o resto fosse negativo? Neste caso b.q seria o menor mltiplo de b maior do que a, e teramos situaes como a do seguinte exemplo: para dividir R$10,00 entre 3 pessoas, cada uma delas receberia 4 reais e haveria um resto de -2(dvida de 2 reais? Quem iria pagar?) O exemplo mostra que esta maneira de se fazer a diviso no teria muito valor prtico. (RPM 8)

Corolrio 4.1 Se a e b so dois inteiros com b 0, existem e so nicos os inteiros q e r que satisfazem s condies: a = bq + r , 0 r < | b |

52

CAPTULO 4 DIVISIBILIDADE

Demonstrao: Com efeito, se b > 0, nada h que demonstrar, e se b < 0, ento | b | > 0, e por conseguinte existem e so nicos os inteiros q1 e r tais que a = | b |q1 + r e 0 ou seja, por ser | b | = - b: a = b(- q1 ) + r e 0 r< |b| r<|b|

Portanto, existem e so nicos os inteiros q = - q1 e r tais que a = bq + r e 0 r < | b |.

Os inteiros a, b, q e r chamam-se respectivamente o dividendo, o divisor, o quociente e o resto na diviso de a por b. Nota: As demonstraes acima garantem a validade do Teorema de Eudoxius: Sejam a e b 0 inteiros, ento, I ) a um mltiplo de b e, portanto, a= bq, q ; II ) a est situado entre dois mltiplos consecutivos de b, isto , existe um inteiro q tal que, para b > 0, bq a < b(q+1) e, para b < 0, bq a < b(q-1). Exemplo 4.2: Achar o quociente q e o resto r na diviso de a = 59 por b = -14 que satisfazem as condies do algoritmo da diviso. Efetuamos a diviso usual dos valores absolutos de a e b, obtemos:

59 14.4 3
o que implica:
59 14 4 3

14

Logo, o quociente q

4 e o resto r

3.

Exemplo 4.3: Achar o quociente q e o resto r na diviso de a = -79 por b = 11 que satisfazem as condies do algoritmo da diviso. Efetuamos a diviso usual dos valores absolutos de a e b, obtemos: 79 = 11.7 + 2 o que implica: -79 = 11
7 2

53

CAPTULO 4 DIVISIBILIDADE

Como o termo r = 2 < 0 no satisfaz a condio 0 r 11 , somando e subtraindo o valor 11 de b ao segundo membro da igualdade anterior, obtemos:
79 11 7 11 11 2 11 8 9

com 0 9 11 . Logo, o quociente q = -8 e o r = 9. Exemplo 4.4: Sejam os inteiros a = 1, -2, 61, -59 e b = -7. Temos:
1 2 7 .0 1 e 7 .1 5 0 1 7 q 0 e r =1

e0 5 7 q 1 e r=5 61 7 8 5 e 0 5 7 q 8 e r=5 59 7 .9 4 e 0 4 7 q 9 e r=4

4.5. Paridade de um Inteiro


Na diviso de um inteiro qualquer a por 2 os possveis restos so r = 1 e r = 0. Se r = 0 , ento o inteiro a = 2q denominado par; e se r = 1, ento o inteiro a = 2q + 1 denominado mpar, q . Dois inteiros que so ambos pares ou ambos mpares dizem-se de mesma paridade, a dois inteiros tais que um par e o outro mpar, dizemos que tem paridades diferentes. De modo geral, dado um inteiro a 2 , pode-se sempre escrever um inteiro qualquer n, de modo nico, na forma n aq r , onde k , r e r a . Teorema 4.3 1) A soma ou a diferena de dois nmeros pares par. 2) A soma ou a diferena de dois nmeros mpares par. 3) A soma ou a diferena de um nmero par com um nmero mpar mpar.

Demonstrao: 1) Sejam a = 2k1 e b = 2k2, ento a k2). 3) Sejam a = 2k1 e b = 2k2 +1, ento a b = 2k1 (2k2 +1) = 2(k1 k2) +1. b = 2k1 2k2 = 2(k1 k2). (2k2 +1) = 2(k1 + k2 + 1) ou 2(k1-

2) Sejam a = 2k1 +1 e b = 2k2 +1, ento a

b = (2k1 +1)

54

CAPTULO 4 DIVISIBILIDADE

Exemplo 4.5: Mostrar que o quadrado de qualquer inteiro mpar da forma 8k+1. Com efeito, pelo algoritmo da diviso, qualquer inteiro de uma das seguintes formas:
4q, 4q 1, 4q 2, 4q 3

Nesta classificao, somente os inteiros das formas 4q +1 e 4q +3 so mpares e , portanto, os seus quadrados so da forma:
4q 1 4q 3
2

8 2q 2 q 8 2q 2

1 8k 1 3q 1 1 8k 1

Assim, por exemplo, 7 e 13 so inteiros mpares, e temos:

72 49 8.6 1 132 169 8.21 1

55

CAPTULO 4 DIVISIBILIDADE

EXERCCIOS
1) Mostrar que se a | b, ento (-a) | b, a | (-b) e (-a) | (-b). 2) Sejam a, b e c inteiros. Mostrar que: a ) se a | b, ento a | bc. b ) se a | b e se a | c, ento a2 | bc. c ) a | b se e somente se ac | bc (c 3) Verdadeiro ou falso: se a | (b + c), ento a | b ou a | c. 4) Mostrar que, se a um nmero inteiro qualquer, ento um dos inteiros a, a + 2, a + 4 divisvel por 3. 5) Sendo a um inteiro qualquer, mostrar: a ) 2 | a(a + 1). b ) 3 | a(a + 1)(a + 2) . 6) Mostrar que um inteiro qualquer da forma 6k + 5 tambm da forma 3t + 2. 7) Mostrar que todo inteiro mpar da forma 4k + 1 ou 4k + 3. 8) Mostrar que o quadrado de um inteiro qualquer da forma 3k ou 3k + 1. 9) Mostrar que o cubo de um inteiro qualquer de uma das formas 9k, 9k + 1 ou 9k + 8. 10) Mostrar que: a) n(n + 1)(2n + 1)/6 um inteiro, qualquer que seja o inteiro positivo n. 0). 14) Demonstrar que: Se a e b so inteiros mpares, ento 8 | a2 b2. 15) Determinar os inteiros positivos que divididos por 17 deixam um resto igual ao quadrado do quociente. 16) Verdadeiro ou falso: se a | c e se b | c, ento a | b. 17) Mostrar que a diferena entre os cubos de dois inteiros consecutivos nunca divisvel por 2. 18) Na diviso do inteiro a = 427 por um inteiro positivo b, o quociente 12 e o resto r. Achar o divisor b e o resto r. 19) Na diviso do inteiro 525 por um inteiro positivo o resto 27. Achar os inteiros que podem ser o divisor e o quociente. 20) Na diviso de dois inteiros positivos o quociente 16 e o resto o maior possvel. Achar os dois inteiros, sabendo-se que sua soma 341. 21) Achar os inteiros positivos menores que 150 e que divididos por 39 deixam um resto igual ao quociente. 22) Seja d um divisor de n (d | n). Mostrar que cd | n se e somente se c | (n/d). 23) Sejam n, r e s inteiros tais que 0 < r < n e 0 < s < n. Mostrar que se n | (r s) ento r = s. 24) Mostrar que o produto de dois inteiros mpares um inteiro mpar. 25) Demonstrar que se m e n so inteiros mpares, ento 8 | (m4 + n4 2). 26) Demonstrar que 30 | (n5 n) 27) Mostrar que, para todo inteiro n, existem inteiros k e r tais que n = 3k + r e r = -1, 0, 1. 13) Sendo m e n dois inteiros quaisquer, mostrar que os inteiros m + n e m n tm sempre a mesma paridade.

b) Se a um inteiro mpar, ento 24 | a (a2 1). 11) Mostrar que se a | (2x 3y) e se a | (4x 5y), ento a | y. 12) Sendo a e b dois inteiros quaisquer, mostrar que os inteiros a e a + 2b tm sempre a mesma paridade.

56

CAPTULO 4 DIVISIBILIDADE 28) Mostrar que (1 + 2 + . . . + n) | 3(1 2 + 22 + . . . + n2) para todo n > 1. 29) Mostre que todo inteiro mpar, quadrado perfeito, da forma 4n + 1. 30) Na diviso de 392 por 45, determinar: a) o maior inteiro que se pode somar ao dividendo sem alterar o quociente.

b) o maior inteiro que se pode subtrair ao dividendo sem alterar o quociente. 31) Numa diviso de dois inteiros, o quociente 16 e o resto 167. Determinar o maior inteiro que se pode somar ao dividendo e ao divisor sem alterar o quociente. 32) Achar o maior inteiro de quatro algarismos divisvel por 13 e o menor inteiro de cinco algarismos divisvel por 15. 33) Achar um inteiro de quatro algarismos, quadrado perfeito, divisvel por 27 e terminado em 6. 34) Mostre que se a, b e c so inteiros mpares, a equao ax racional.
2

bx

0 no tem raiz

35) Um tabuleiro 6 6 est coberto com domins 2 1. Mostre que existe uma reta que separa as peas do tabuleiro sem cortar nenhum domin. 36) Dividindo-se o nmero 245 por um nmero natural b, obtm-se quociente 5 e resto r. Determine o valor da soma dos valores possveis para b. 37) A diviso de um certo nmero inteiro N por 1994 deixa resto 148. Calcule o resto da diviso de N + 2000 pelo mesmo nmero 1994. 38) Considere quatro nmeros inteiros a, b, c e d. Prove que o produto: (a-b) . (c-a) . (d-a) . (d-c). (d-b). (c-b) divisvel por 12. 39) Prove que a mpar.
n

bn divisvel por a+b se n

57

Captulo 5

MXIMO DIVISOR COMUM


5.1. Mximo Divisor Comum de Dois Inteiros
Definio 5.1 Sejam a e b dois inteiros no conjuntamente nulos (a 0 ou b 0). Chama-se mximo divisor comum de a e b o inteiro positivo d (d 0) que satisfaz s condies: 1) d | a e d | b; 2) se c | a e se c | b, ento c d.

Observe-se que, pela condio (1), d um divisor comum de a e b, e pela condio (2), d o maior dentre todos os divisores comuns de a e b.

O mximo divisor comum de a e b indica-se pela notao mdc(a,b). imediato que o mdc(a,b) = mdc(b,a). Em particular: (i) o mdc(0,0) no existe. (ii) o mdc(a,1) = 1 (iii) se a 0, ento o mdc(a,0) = | a |

(iv) se a | b, ento o mdc(a,b) = | a | Assim, por exemplo: mdc(8,1) = 1 mdc(-3,0) = | -3 | = 3 mdc(-6,12) = | -6 | = 6.

Exemplo 5.1 Sejam os inteiros a = 16 e b = 24. Os divisores comuns positivos de 16 e 24 so 1, 2, 4 e 8, e como o maior 8, segue-se que o mdc(16,24) = 8.

Observa-se que mdc(-16,24) = mdc(16,-24) = mdc(-16,-24) = 8. Exemplo 5.2 Sejam os inteiros a = -24 e b = 60. Os divisores comuns positivos de 24 e 60 so 1, 2, 3, 4, 6 e 12, e como o maior deles 12, segue-se que o mdc(-24,60) = 12.

58

CAPTULO 5 MXIMO DIVISOR COMUM

5.2. Existncia e unicidade do MDC.


Teorema 5.1 (Identidade de Bzout ) Se a e b so dois inteiros no conjuntamente nulos ( a 0 ou b 0), ento existe e nico o mdc(a,b); alm disso, existem inteiros x e y tais que mdc(a,b) = ax + by Isto , o mdc(a,b) uma combinao linear de a e b.

Nota: Algumas fontes creditam este teorema ao matemtico francs Claude Gaspard Bachet de Mziriac e no ao tambm francs, Etienne Bzout.

Demonstrao: Seja S o conjunto de todos os inteiros positivos da forma au + bv, com u, v S = { au + bv | au + bv Este conjunto S no vazio (S 0 e u, v Z} 0, ento um dos dois inteiros: Z, isto :

), porque, por exemplo, se a e

a = a.1 + b.0

-a = a.(-1) + b.0

positivo e pertence a S. Logo, pelo Princpio da boa ordenao, existe e nico o elemento mnimo d de S: minS = d 0. E, consoante a definio de S, existem inteiros x e y tais que d = ax + by. Posto isto, vamos mostrar que d = mdc(a,b). Com efeito, pelo algoritmo da diviso, temos: a = dq + r, com 0 O que d: r = a dq = a (ax + by)q = a(1 qx) + d(-qy) Isto , o resto r uma combinao linear de a e b. Como 0 r d e d 0 o elemento mnimo de S, segue-se que r = 0 e a = dq, isto , d | a. Com raciocnio inteiramente anlogo se conclui que tambm d | b. Logo, d um divisor comum positivo de a e b. Finalmente, se c um divisor comum positivo qualquer de a e b ( c | b, c 0), ento: c | (ax + by) c|d c d r d

Isto , d o maior divisor comum positivo de a e b, ou seja: mdc(a,b) = d = ax + by e o teorema fica demonstrado. x, y Z

59

CAPTULO 5 MXIMO DIVISOR COMUM

Nota: A demonstrao do teorema 1 deixa ver que o mdc(a,b) o menor inteiro positivo da forma ax + by, isto , que pode ser expresso como combinao linear de a e b. Mas, esta representao do mdc(a,b) como combinao linear de a e b no pe punica, pois, temos:

mdc(a,b) = d = a(x + bt) + b(y - at) qualquer que seja o inteiro t. Importa ainda notar que, se d = ar + bs. Para algum par de inteiros r e s, ento d no necessariamente o mdc(a,b). Assim, por exemplo, se: mdc(a,b) = ax + by ento t.mdc(a,b) = atx + bty Para todo inteiro t, isto : d = ar + bd onde d = t.mdc(a,b), r = tx e s = ty. Exemplo 5.3 Sejam os inteiros a = 6 e b = 27. Temos: mdc(6,27) = 3 = 6(-4 + 27t) + 27(1 6t) qualquer que seja o inteiro t. Exemplo 5.4 Sejam os inteiros a = -8 e b = -36. Temos: mdc(-8,-36) = 4 = (-8)4 + (-36)(-1) Teorema 5.2 Se a e b so dois inteiros no conjuntamente nulos (a conjunto de todos os mltiplos do mdc(a,b) = d T = { ax + by | x,y Z} 0 ou b 0), ento o

60

CAPTULO 5 MXIMO DIVISOR COMUM

Demonstrao: Como d | a e d | b, segue-se que d | (ax + by), quaisquer que sejam os inteiros x e y, e por conseguinte todo elemento do conjunto T e um mltiplo de d. Por outro lado, existem inteiros x0 e y0 tais que d = ax0 + by0, de modo que todo mltiplo kd de d da forma: kd = k(ax0 + by0) = a(kz0) + b(ky0) isto , kd uma combinao linear de a e b e, portanto, kd elemento do conjunto T.

5.3. Inteiros Relativamente Primos (coprimos ou primos entre si)


Definio: Sejam a e b dois inteiros no conjuntamente nulos (a so relativamente primos se, e somente se, o mdc(a,b) = 1. 0eb 0). Diz-se que a e b

Assim, por exemplo, so relativamente primos os inteiros: 2 e 5, -9 e 16, -27 e 35, pois, temos: mdc(2,5) = mdc(-9,16) = mdc(-27,-35) = 1 Dois inteiros a e b coprimos admitem como nicos divisores comuns 1 e 1. Teorema 5.3 Dois inteiros a e b, no conjuntamente nulos (a se, e somente se, existem inteiros x e y tais que ax + by = 1. Demonstrao: ( ) Se a e b so relativamente primos, ento o mdc(a,b) = 1 e por conseguinte existem inteiros x e y tais que ax + by = 1 ( ) Reciprocamente, se existem inteiros x e y tais que ax + by = 1 e se o mdc(a,b) = d, ento d | a e d | b. Logo, d | (ax + by) e d | 1, o que implica d = 1 ou mdc(a,b) =1, isto , a e b so primos entre si. Corolrio 5.1 Se o mdc(a,b) = d, ento o mdc( a/d , b/d ) = 1. Demonstrao: Preliminarmente, observa-se que a/d e b/d so inteiros, porque d um divisor comum de a e b. Posto isso, se o mdc(a,b) = d, ento existem inteiros x e y tais que ax + by = d, ou seja, dividindo ambos os membros desta igualdade por d: 0eb 0), so primos entre si

61

CAPTULO 5 MXIMO DIVISOR COMUM

(a/d)x + (b/d)y = 1 Logo, pelo teorema anterior, os inteiros a/d e b/d so primos entre si, isto , o mdc (a/d ,b/d) = 1. Assim, por exemplo: mdc(-12,30) = 6 e mdc(-12/6 , 30/6) = mdc(-2,5) = 1. Corolrio 5.2 Se a | b e se o mdc(b,c) = 1, ento o mdc (a,c) = 1. Demonstrao: Com efeito: a | b b = aq, com q Z mdc(b,c) = 1 bx + cy = 1, com x, y Portanto: a(qx) + cy = 1 mdc(a,c) = 1 Z.

Corolrio 5.3 Se a | c, se b | c e se o mdc(a,b) = 1, ento ab | c. Demonstrao: Com efeito: a|c b|c mdc(a,b) = 1 c = aq1, c = bq2, ax + by = 1, acx + bcy = c com q1 com q2 Z Z Z

com x,y

Portanto: c = a(nq2)x = b(aq1)y = ab(q2x + q1y) ab | c

Observe-se que somente as condies a | c e b | c no implicam ab | c. Assim, por exemplo, 6 | 24 e 8 | 24, mas 6.8 | 24 (o mdc(6,8) = 2 1). Corolrio 5.4 Se mdc(a,b) = 1 = mdc(a,c), ento o mdc(a,bc) = 1.

62

CAPTULO 5 MXIMO DIVISOR COMUM

Demonstrao: Com efeito: mdc(a,b) = 1 mdc(a,b) = 1 Portanto: 1 = ax + by(az + ct) = a(x + byz) + bc(yt) o que implica mdc(a,bc) = 1. Corolrio 5.5 Se o mdc(a,bc) = 1, ento mdc(a,b) = 1 = mdc(a,c). Demonstrao: Com efeito: mdc(a,bc) = 1 Portanto: ax + b(cy) = 1 ax + c(by) = 1 mdc(a,b) = 1 mdc(a,c) = 1 ax + (bc)y = 1, com x,y Z. ax + by = 1, com x,y az + ct = 1, com z,t Z Z

Note-se que esta proposio a recproca da anterior. Teorema 5.4 (de Euclides) Se a | bc e se o mdc(a,b) = 1, ento a | c. Demonstrao: Com efeito: a | bc bc = aq, com q Z mdc(a,b) = 1 ax + by = 1, com x, y acx + bcy = c Portanto: c = acx + aqy = a(cx + qy) Note-se que somente a condio a | bc no implica que a | c. Assim, por exemplo, 12 | 9.8, mas 12 | 9 e 12 | 8 mdc(12,9) a|c Z

1 e mdc(12,8)

1.

63

CAPTULO 5 MXIMO DIVISOR COMUM

5.4. Caracterizao do MDC de Dois Inteiros


Teorema 5.5 Sejam a e b dois inteiros no conjuntamente nulos (a positivo d (d 0) o mdc(a,b) se e somente se satisfaz s condies: (1) d | a e d | b (2) se c | a e se c | b, ento c | d Demonstrao: ( ) Suponhamos que o mdc (a, b) = d. Ento, d | a e d | b, isto , a condio (1) satisfeita. Por outra parte, existem inteiros x e y tais que ax + by = d e, portanto, se c | a e se c | b, ento c | (ax + by) e c | d, isto , a condio (2) tambm satisfeita. ( ) Reciprocamente, seja d um inteiro positivo qualquer que satisfaz s condies (1) e (2). Ento, pela condio (2), todo divisor comum c de a e b tambm divisor de d, isto , c | d, e isto implica c d. Logo, d o mdc(a,b). 0 ou b 0). Um inteiro

5.5. MDC de vrios Inteiros


O conceito de mximo divisor comum, definido para dois inteiros a e b, estende-se de maneira natural a mais de dois inteiros. No caso de trs inteiros a, b e c, no todos nulos, o mdc(a,b,c) o inteiro positivo d (d 0) que satisfaz s condies: (1) d | a, d | b e d | c (2) se e | a, se e | b e se e | c, ento e Assim, por exemplo: mdc(39,42,54) = 3 e mdc(49,210,350) = 7 Importa notar que trs inteiros a, b e c podem ser primos entre si, isto , o mdc(a,b,c) = 1, sem que sejam primos entre si dois a dois, que o caso, por exemplo, dos inteiros 6, 10 e 15. Teorema 5.6 O mdc(a,b,c) = mdc(mdc(a,b),c). Demonstrao: Com efeito, seja mdc(a,b,c) = d e mdc(a,b) = e. Ento, d | a, d | b e d | c, e como existem inteiros x e y tais que ax + by = e, segue-se que d | (ax + by) ou d | e, isto , d um divisor comum de e e c (d | e e d | c). d

64

CAPTULO 5 MXIMO DIVISOR COMUM

Por outro lado, se f um divisor comum qualquer de e e c (f | e e f | c), ento f | a, f | b e f | c, o que implica f d. Assim sendo, o mdc(e,c) = d, isto , p mdc(mdc(a,b),c) = mdc(a,b,c). Exemplo 5.5 Determinar o mdc(570,810,495). Pelo teorema anterior, temos: mdc(570,810,495) = mdc(mdc(570,810),495) e como o mdc(570,810) = 30, segue-se que o mdc(570,810,495) = mdc(30,495) = 15

EXERCCIOS
1. Determinar: a) mdc(11, 99) 9. Sendo a e b dois inteiros no conjuntamente nulos (a 0 ou b 0), mostrar: mdc (a, b) = mdc (-a, b) = mdc (a, -b) = mdc (-a, -b).

b) mdc(-21,14) c) 1. mdc(17, 18) 10. Sejam a, b e c inteiros. Demonstrar: a) Achar os elementos do conjunto A = {1, 2, 3, 4, 5} que so relativamente primos com 8. Seja o conjunto A = {1, 2, 3, 4, 5, 6}. Enumerar os elementos do conjunto X = {x A | mdc(x, 6) = 1}. Sabendo que o mdc(a, 0) = 13, achar todos os valores do inteiro a. Achar o menor inteiro positivo c, da forma c = 22x + 55y, onde x e y so dois inteiros. Sendo n um inteiro qualquer, calcular o mdc(n, n + 1). Calcular a) mdc(n, n + 2), sendo n um inteiro par. existem inteiros x e y tais que c = ax + by se e somente se o mdc(a, b) | c.

2.

b) se existem inteiros x e y tais que ax + by = mdc(a, b) ento mdc(x, y) = 1. 11. Sejam a, b e c inteiros. Demonstrar: a) se o mdc(a, b) = 1 ento o mdc (ac, b) = mdc (b, c)

3.

4. 5.

b) Se o mdc(a, b) = 1 e se c | (a + b), ento o mdc(a, c) = 1 e o mdc(b, c) = 1. c) se b | c, ento o mdc(a, b) = mdc (a + c, b).

6.

d) Se mdc (a, b) = 1, ento mdc (am, bn) = 1. e) Se mdc (a, b)= 1, ento ab+b) =1 ou 3 mdc (a+b, a-

b) mdc(n, n + 2), sendo n um inteiro mpar. f) 7. Sendo n um inteiro qualquer, achar os possveis valores do mximo divisor comum dos inteiros n e n + 10. Sendo n um inteiro qualquer, calcular o mdc(n 1, n2 + n + 1).

O mdc (a,b) = mdc (a, b + ac), com c positivo.

12. Calcular o mdc (a + b, a b) sabendo que a e b so inteiros primos entre si. 13. Seja 10 a 120 e mdc ( a, 120 ) =10. Determine o valor de a.

8.

65

CAPTULO 5 MXIMO DIVISOR COMUM 31. O mdc(a, b) = p, sendo p um primo. Achar os possveis valores do a) mdc (a2, b) b) mdc(a3, b) = p, mesma concluso acima. 15. Determinar os inteiros positivos a e b, sabendose que: a) a + b = 63 e mdc(a, b) = 9 c) mdc(a2, b3) = p2. Pois aparecem 2 fatores iguais a p em a2 e 3 fatores iguais a p em b3.

14. Achar o maior inteiro positivo pelo qual se devem dividir os inteiros 160, 198 e 370 para que os restos sejam respectivamente 7, 11 e 13.

b) ab = 756 e mdc(a, b) = 6. 16. Os restos das divises dos inteiros 4933 e 4435 por um inteiro positivo n so respectivamente 37 e 19. Achar o inteiro n. 17. Demonstrar que se n = abc + 1, ento o mdc(n, a) = mdc(n, b) = mdc(n, c) = 1. 18. Demonstrar que mdc(mdc(a, b), b) = mdc(a, b) 19. Demonstrar que o mdc(n + k, k) = 1 se e somente se o mdc(n, k) = 1. 20. Demonstrar que, se a | bc e se mdc(a, b) = d, ento a | cd. 21. Demonstrar que, se a | c, se b | c e se o mdc(a, b) = d ento ab | cd. 22. Demonstrar que se mdc(a, b) = 1 e se mdc(a,c) = d,ento mdc(a, bc) = d. 23. O inteiro mpar d um divisor de a + b e de a b. Demontrar que d tambm um divisor do mdc(a, b). 24. Os inteiros positivos a, b e c so tais que o mdc(a, b) = 1, a | c e c | b. Demonstrar que a = 1. 25. O mdc(n, n + k) = 1 para todo inteiro positivo n. Demonstrar que k = 1 ou k = -1. 26. Demonstrar que mdc(a, b) = mdc(a + kb, b) para todo inteiro k. 27. O mdc(a, 4) = 2 = mdc(b, 4). Demonstrar que o mdc(a + b, 4) = 4. 28. Os inteiros positivos m e n so tais que o mdc(m, n) = d. Mostrar que o mdc (2m 1, 2n 1) = 2d 1. 29. Demonstrar que mdc(a, b) = mdc(a, b, a + b). 30. Demonstrar que mdc(a, b) = mdc(a, b, ax + by), quaisquer que seja os inteiros x e y.

32. Sabendo que o mdc(a, p2) = p e que o mdc(b, p3) = p2, onde p um primo, calcular o mdc (ab, p4) e o mdc(a + b, p4). 33. Demonstrar que se o mdc(a, b) = d ento o mdc (a2, b2) = d2. 34. Demonstrar que mdc(a, b) = mdc(a, c) implica mdc(a2, b2) = mdc(a2, c2). 35. Sejam a e k inteiros no conjuntamente nulos. Demonstrar que mdc(a, a + k) | k. 36. Demonstrar que mdc(a, b) = mdc(a, c) implica mdc(a, b) = mdc(a, b, c). 37. Demonstrar que mdc(a, b, c) = mdc(mdc(a, b), mdc(a, c). 38. Sejam a e b inteiros positivos tais que ab um quadrado perfeito e o mdc(a, b) = 1. Demonstrar que a e b so quadrados perfeitos. 39. Demonstrar que mdc( a + b, a b) > mdc(a, b) 40. Mostrar que o mdc (5n + 6, 5n + 8) = 1 onde n um inteiro mpar. 41. Sejam a, b, c, d (b d) inteiros tais que mdc(a, b) = mdc(c, d) = 1. Mostrar que a soma a/b + c/d no um inteiro. 42. Determinar os inteiros positivos a e b, sabendo que a2 b2 = 7344 e mdc(a, b) = 12. 43. Dividindo-se dois inteiros positivos pelo seu mdc, a soma dos quocientes 8. Determinar os dois inteiros, sabendo-se que sua soma 384. 44. Um enxadrista quer decorar uma parede retangular, dividindo-a em quadrados, como se fosse um tabuleiro de xadrez. A parede mede 4,40 metros por 2,75 metros. Qual o menor nmero de quadrados que ele pode colocar na parede?

66

Captulo 6

ALGORITMO DE EUCLIDES MNIMO MLTIPLO COMUM

ouco se sabe sobre a vida e a personalidade de Euclides e se desconhece a data de seu nascimento. provvel que sua formao matemtica tenha se dado na escola platnica de Atenas. Ele foi professor do Museu em Alexandria. Euclides escreveu cerca de uma dzia de tratados, cobrindo tpicos desde ptica, astronomia, msica e mecnica at um livro sobre seces cnicas; porm, mais da metade do que ele escreveu se perdeu. Entre as obras que sobreviveram at hoje temos: Os elementos, Os dados, Diviso de figuras, Os fenmenos e ptica. Os elementos de Euclides no tratam apenas de geometria, mas tambm de teoria dos nmeros e lgebra elementar (geomtrica). O livro se compe de quatrocentos e sessenta e cinco proposies distribudas em treze livros ou captulos, dos quais os seis primeiros so sobre geometria plana elementar, os trs seguintes sobre teoria dos nmeros, o livro X sobre incomensurveis e os trs ltimos tratam sobre geometria no espao. O livro VII comea com o processo, hoje conhecido como algoritmo euclidiano, para achar o mximo divisor comum de dois ou mais nmeros inteiros e o usa para verificar se dois inteiros so primos entre si; encontramos tambm uma exposio da teoria das propores numrica ou pitagrica. http://www.matematica.br/historia/euclides.html Comecemos com o seguinte Lema: Lema: Se a = bq + r, ento o mdc(a,b) = mdc(b,r). Demonstrao: Se o mdc(a,b) = d, ento d | a e d | b, o que implica d | (a - bq) ou d | r, isto , d um divisor comum de b e r (d | b e d | r). Por outro lado, se c um divisor comum qualquer de b e r (c | b e c | r), ento c | (bq + r) ou c | a, isto , c um divisor comum de a e b, o que implica c d. Assim sendo, o mdc(b,r) = d.

6.1. Algoritmo de Euclides


Sejam a e b dois inteiros no conjuntamente nulos ( a comum se deseja determinar. 0 ou b 0 ) cujo mximo divisor

67

CAPTULO 6 ALGORITMO DE EUCLIDES MNIMO MLTIPLO COMUM

Sejam a e b dois inteiros no conjuntamente nulos ( a comum se deseja determinar. imediato: (1) se a 0, ento o mdc(a,0) = | a | (2) se a 0, ento o mdc(a,a) = | a | (3) se b | a , ento o mdc(a,b) = | b |

0 ou b

0 ) cujo mximo divisor

Alm disso, por ser mdc(a,b) = mdc( | a | , | b | ), a determinao do mdc(a,b) reduz-se ao caso em que a e b so inteiros positivos distintos, por exemplo, com a b, tais que b no divide a, isto : a b 0 e b | a. nestas condies, a aplicao repetida do algoritmo da diviso dnos as igualdades: a = bq1 + r1, b = r1q2 + r2, r1 = r2q3 + r3, r2 = r3q4 + r4,
..............................

0 r1 b 0 r2 r1 0 r3 r2 0 r4 r3
........................

Como os restos r1, r2, r3, r4, ... so todos inteiros positivos tais que b r1 r2 r3 r4 ...

e existem apenas b 1 inteiros positivos menores que b, necessariamente se chega a uma diviso cujo resto rn+1 = 0, isto , finalmente, teremos: rn-2 = rn-1qn + rn, 0 rn rn-1 rn-1 = rnqn+1 + rn+1, rn-1 = 0 O ltimo resto rn 0 que aparece nesta sequncia de divises o mximo divisor comum procurado de a e b, isto , o mdc(a,b) = rn, visto que, pelo lema anterior, temos: mdc(a,b) = mdc(b,r1) = mdc(r1,r2) = ... = = mdc(rn-2,rn-1) = mdc(rn-1,rn) = rn este processo prtico para o clculo do mximo divisor comum de dois inteiros positivos a e b denominado algoritmo de EUCLIDES ou processo das divises sucessivas. usual o seguinte dispositivo de clculo no emprego do algoritmo de EUCLIDES: q1 b r2 q2 r1 r3 q3 r2 r4 qn rn-1 0 qn+1 rn

a r1

...

68

CAPTULO 6 ALGORITMO DE EUCLIDES MNIMO MLTIPLO COMUM

Que se traduz na seguinte REGRA: Para se achar o mdc de dois inteiros positivos, dividi-se o maior pelo menor, este pelo primeiro resto obtido, o segundo resto pelo primeiro, e assim sucessivamente at encontrar um resto nulo. O ltimo resto no nulo o mximo divisor comum procurado. O algoritmo de EUCLIDES tambm pode ser usado para achar a expresso do mdc(a,b) = rn como combinao linear de a e b, para o que basta eliminar sucessivamente os restos rn-1, rn-2, ..., r3, r2, r1 entre as n primeiras igualdades anteriores. Exemplo 6.1 Achar o mdc(963,657) pelo algoritmo de EUCLIDES e a sua expresso como combinao linear de 963 e 657, Temos, sucessivamente: 963 = 657.1 + 306 657 = 306.2 + 45 306 = 45.6 + 36 45 = 36.1 + 9 36 = 9.4 + 0 1 657 306 2 306 45 6 45 36 1 36 9 4 9 0

963

Portanto, o mdc(963,657) = 9 e a sua expresso como combinao linear de 963 e 657 se obtm eliminando os restos 36, 45 e 306 entre as quatro primeiras igualdades anteriores do seguinte modo: 9 = 45 36 = 45 (306 45.6) = = - 306 + 7.45 = - 306 + 7(657 306.2) = = 7.657 15.306 = 7.657 15(963 657) = = 963(-15) + 657.2 isto : 9 = mdc(963,657) = 963x + 657y onde x = -15 e y = 22. Esta respresentao do inteiro 9 = mdc(963,657) como combinao linear de 963 e 657 no nica. Assim, por exemplo, somando e subtraindo o produto 963.657 ao segundo mebro da igualdade: 9 = 963(-15) + 657.22 obtemos: 9 = 963(-15 + 657) + 657(22 - 963) = = 963.642 + 657(-941)

69

CAPTULO 6 ALGORITMO DE EUCLIDES MNIMO MLTIPLO COMUM

que uma outra representao do inteiro 9 = mdc(963,657) como combinao linear de 963 e 657. Exemplo 6.2 Achar o mdc(252,-180) pelo algoritmo de EUCLIDES e a sua expresso como combinao linear de 252 e 180. Temos, sucessivamente: 252 = 180.1 + 72 180 = 72.2 + 36 72 = 36.2

Portanto, o mdc(252,-180) = mdc(252,180) = 36 e como 36 = 180 72.2 = 180 (252 180)2 = = 252(-2) + (-180)(-3) temos: 36 = mdc(252,-180) = 252x + (-180)y onde x = -2 e y = -3, que a expresso do mdc(252,-180) como combinao linear de 252 e 180. Outra representao do inteiro 36 = mdc(252,-180) como combinao linear de 252 e 180 a seguinte: 36 = 252(-2 + 180) + (-180)(-3 + 252) = = 252.178 + (-180)249 Exemplo 6.3 O mdc de dois inteiros positivos a e b 74 e na sua determinao pelo algoritmo de EUCLIDES os quocientes obtidos foram 1, 2, 2, 5, 1 e 3. Calcular a e b. 1 b r 2 r r1 2 r1 r2 5 r2 r3 1 3 r3 74 74 0

Temos, sucessivamente: a = b + r, b = 2r + r1, r = 2r1 + r2 r1 = 5r2 + r3, r1 = r2 + 74, r3 = 74.3 = 222

70

CAPTULO 6 ALGORITMO DE EUCLIDES MNIMO MLTIPLO COMUM

Portanto: r2 = 222 + 74 = 296, r1 = 5.296 + 222 = 1702 r = 2.1702 + 296 = 3700, b = 2.3700 + 1702 = 9102 a = 9102 + 3700 = 12802

Nota: Dispositivo prtico para expressar o MDC...(Jos Paulo Q. Carneiro) Antes de deduzir o dispositivo prtico, vamos v-lo funcionando no exemplo citado. Forma-se uma matriz com duas colunas, intituladas q e s, t. Primeiramente, preenche-se a coluna q com os quocientes obtidos no algoritmo de Euclides (desprezando o ltimo, correspondente ao resto zero), na ordem contrria ao de seu aparecimento no algoritmo, e deixando em branco a primeira linha.

Na coluna s, t, coloca-se o nmero 1 na primeira linha ( sempre 1 mesmo), e na segunda linha repete-se o valor do quociente que aparece ao seu lado na primeira coluna. No nosso exemplo, fica:

A partir da, cada valor seguinte da coluna s, t vai sendo obtido de acordo com o seguinte esquema auto-explicativo: Observa-se que os dois ltimos valores obtidos na segunda coluna so, justamente, em valor absoluto, s = 85e t = 539. Isso no coincidncia. Na realidade, essa matriz resume as contas que precisam ser feitas com os nmeros relevantes que figuram no processo. H a questo dos sinais, isto , o esquema s fornece os valores absolutos de s e t. Para decidir sobre os sinais, aplica-se a seguinte regra: se o nmero de quocientes aproveitados (ou seja, o nmero de linhas preenchidas na coluna q) for mpar, ento s positivo e t negativo; se for par, ocorrer justamente o contrrio: s negativo e t positivo. Deve ser notado que se pode tambm, em vez de decorar mais uma regra, experimentar, comparando os valores de e . No exemplo, | s | a = 85 x 7248 = 616080 e | t | b = 539 x 1143 = 616077, ficando claro que, para obter o mdc 3, necessrio fazer s = 85 e t = 539. Outro exemplo: a = 1741 e b = 85 20 1741 85 41 2 41 3 13 3 2 1 2 1 2 1 0

71

CAPTULO 6 ALGORITMO DE EUCLIDES MNIMO MLTIPLO COMUM

Formando ento a matriz:

Como agora o nmero de quocientes aproveitados par, tomase: s = 29 s = 29 e t = 549, de modo que ( 29) x 1741 + (594) x 85 = 50 489 + 40 490 = 1

Justificativa do dispositivo prtico Vamos agora justificar o dispositivo prtico, por meio da formulao literal do problema. Supondo > b > 0 inteiros, aplica-se o algoritmo de Euclides, encontrando:

onde rn = m = mdc (a, b). Os quocientes j foram numerados de trs para frente, isto , o ltimo quociente (correspondente ao resto 0) q0, o penltimo q1, etc., e o primeiro qn, para manter coerncia com o dispositivo prtico. Desprezando, como se fez nos exemplos, o ltimo quociente q0, v-se, da primeira equao, que r1 pode ser escrito como uma combinao linear de a e b, ou seja, r1 = a qnb. Substituindo esse valor na penltima equao, v-se que r2 tambm pode ser escrito como uma combinao linear de a e b, a saber, r2 = qn-1a + (1 + qnqn1)b. E assim por diante, para todos os r. Em particular, calcula-se m = n = sa+ tb. Repare que acabamos de demonstrar o Teorema Fundamental da Teoria dos Nmeros. Se o leitor no estiver satisfeito com o e assim por diante, pode formalizar a demonstrao por induo. realmente fcil. Basta observar que uma combinao linear de combinaes lineares de a e b tambm uma combinao linear de a e b. n 1 2 3 4 ... s 1 t

...

...

A lei de formao clara (e pode ser verificada por induo):

para n 1:

sk tk

tk

1 1

qk t k

Sk

com os valores iniciais: s1 = 1 e t1 = q1.

72

CAPTULO 6 ALGORITMO DE EUCLIDES MNIMO MLTIPLO COMUM

Observando que cada s repete o t anterior, basta trabalhar com os t e notar que, ao final do processo, o ltimo valor o t procurado, enquanto o penltimo o s. Foi por isso que introduzimos a coluna s, t na matriz da nossa regra prtica. Alm disso, levando em conta a alternncia dos sinais, suficiente lidar com os valores absolutos de t, tomando o cuidado de, no final, fazer a correo de sinal conveniente, conforme n seja par ou mpar. Com essas providncias, a lei de formao fica: | tx | = qx | tx -1 | + | tx-2 |, que justamente o que se fez na regra prtica. Algum pode ainda perguntar: quando se escreve o mximo divisor comum de dois inteiros como uma combinao linear deles, essa representao nica? A resposta no (ver [4], para maiores detalhes). Uma vez encontrados s e t tais que , basta tomar um inteiro qualquer u, e tambm ser verdade que m = s a + tb, onde s = s + bu e t = t au, como pode ser verificado diretamente por substituio. Em nosso primeiro exemplo, tomando , temos:

s' = 85 + 1143 = 1228, enquanto t = 539 7248 = 7728 Verificando: s'a + t'b= 1 228 x 7 248 + ( 7 787) x 1 143 = 8900544 8900541 = 3 Teorema 6.1 Se k Demonstrao: Multiplicando ambos os membros de cada uma das n+1 igualdades que do o mdc(a,b) = rn pelo algoritmo de EUCLIDES por k, obtemos: ak = (bk)q1 + r1k, 0 r1k bk bk = (r1k)q2 + r2k, 0 r2k r1k r1k = (r2k)q3 + r3k, 0 r3k r2k ............................. ..................... rn-2k = (rn-1k)qn + rnk, 0 rnk rn-1k rn-1k = (rnk)qn+1 + 0 Obviamente, estas n+1 igualdades outra coisa no so que o algoritmo de EUCLIDES aplicado aos inteiros ak e bk, e por conseguinte o mdc(ak,bk) o ltimo resto rnk 0, isto : mdc(ak,bk) = rnk = k.mdc(a,b) 0, ento o mdc(ka,kb) = k.mdc(a,b).

73

CAPTULO 6 ALGORITMO DE EUCLIDES MNIMO MLTIPLO COMUM

Assim, por exemplo: mdc(12,30) = mdc(2.6,5.6) = 6.mdc(2,5) = 6.1 = 6 Corolrio 6.1 Para todo k Demonstrao: Se k 0, nada h que demonstrar, e se k 0, ento k = | k | 0 e, pelo teorema anterior, temos: mdc(ak,bk) = mdc(-ak,-bk) = = mdc(a.| k |, b.| k |) = | k |.mdc(a,b) 0, o mdc(ka,kb) = | k |.mdc(a,b)

6.2 . Mltiplos comuns de dois inteiros


O conjunto de todos os mltiplos de um inteiro qualquer a M(A) = {x Z | a | x } = {aq | q Z} 0 indica-se por M(a), isto :

Assim, por exemplo: M(-1) = M(1) = Z M(5) = {5q | q imediato que, para todo inteiro a Z} = {0, +5, +10, +15, +20, ...} 0, se tem M(a) = M(-a). 0eb 0). Chama-se mltiplo

Definio 6.1 Sejam a e b dois inteiros diferentes de zero (a comum de a e b todo inteiro x tal que a | x e b | x.

Em outros termos, mltiplo comum de a e b todo inteiro que pertence simultaneamente aos conjuntos M(a) e M(b). O conjunto de todos os mltiplos comuns de a e de b indica-se por M(a,b). Portanto, simbolicamente: M(a,b) = {x Ou seja: M(a,b) = {x E, portanto: M(a,b) = M(a) M(b) Z|x M(a) e x M(b)} Z | a | x e b | x}

74

CAPTULO 6 ALGORITMO DE EUCLIDES MNIMO MLTIPLO COMUM

A interseo ( ) uma operao comutativa, de modo que M(a,b) no depende da ordem dos inteiros dados a e b, isto : M(a,b) = M(b,a). Obviamente, 0 um mltiplo comum de a e b: 0 so mltiplos comuns de a e b. M(a,b). E os produtos ab e (ab) tambm

Exemplo 6.4 Sejam os inteiros a = 12 e b = -18. Temos: M(12) = {12q | q M(-18) = {-18q | q Portanto: M(12,-18) = M(12) M(-18) = {0, +36, +72, ...} Z} = {0, +12, +24, +36, +48, +60, +72, ...} Z} = {0, +18, +36, +54, +72, +90, ...}

6.3. Mnimo Mltiplo comum de dois inteiros


Definio 6.2 Sejam a e b dois inteiros diferentes de zero (a 0 e b 0). Chama-se mnimo mltiplo comum de a e b o inteiro positivo m (m 0) que satisfaz s condies: (1) (2) a|m e b|m se a | c e se b | c, com c

0, ento m

c.

Observe-se que, pela condio (1), m um mltiplo comum de a e b, e pela condio (2), m o menor dentre todos os mltiplos comuns positivos de a e b. O mnimo mltiplo comum de a e b indica-se pela notao mmc(a,b). Pelo Princpio da boa ordenao , o conjunto dos mltiplos comuns positivos de a e b possui o elemento mnimo e, portanto, o mmc(a,b) existe sempre e nico. Alm disso, por ser o prodtudo ab um mltiplo comum de a e b, segue-se que o mmc(a,b) |ab|. Em particular, se a | b, ento o mmc(a,b) = | b |. Exemplo 6.5 Sejam os inteiros a = -12 e b = 30. Os mltiplos comuns positivos de 12 e 30 so 60, 120, 180, ..., e como o menor deles 60, segue-se que o mmc(-12,30) = 60. Relao entre mdc e o mmc Teorema 6.2 Para todo par de inteiros positivos a e b subsiste a relao: mdc(a,b).mmc(a,b) = |ab|

75

CAPTULO 6 ALGORITMO DE EUCLIDES MNIMO MLTIPLO COMUM

Demonstrao: Seja mdc(a,b) = d e mmc(a,b) = m. Como a | a(b/d) e b | b(a/d), segue-se que ab/d um mltiplo comum de a e b. Portanto, existe um inteiro positivo k tal que ab/d = mk, o que implica: a/d = (m/b)k e b/d = (m/a)k k N

isto , k um divisor comum dos inteiros a/d e b/d. Mas, a/d e b/d so primos entre si (corolrio 1 desta apostila), de modo que k = 1. Assim sendo, temos: ab/d = m ou ab = dm isto : ab = mdc(a,b).mmc(a,b) Esta importante relao permite determinar o mmc de dois inteiros quando se conhece o seu mdc, e vice-versa. Exemplo 6.6 Determinar o mmc(963,657). Pelo algoritmo de EUCLIDES, temos mdc(963,657) = 9. Portanto: mmc(963,657) =
963 .657 =70299 9

Corolrio 6.2 Para todo par de inteiros positivos a e b, o mmc(a,b) = ab se e somente se o mdc(a,b) = 1. Demonstrao: ( ) Se o mdc(a,b) = 1, ento: ab = 1.mmc(a,b) = mmc(a,b) ( ) Reciprocamente, se o mmc(a,b) = ab, ento: mdc(a,b).ab = ab mdc(a,b) = 1

6.5. MMC de vrios inteiros

O conceito de mnimo mltiplo comum, definido para dois inteiros a e b, estende-se de maneira natural a mais de dois inteiros. No caso de trs inteiros a, b e c, diferentes de zero, o mmc(a,b,c) o inteiro positivo m (m 0) que satisfaz s condies:

76

CAPTULO 6 ALGORITMO DE EUCLIDES MNIMO MLTIPLO COMUM

(1) a | m, b | m e c | m (2) se a | e, se b | e e se c | e, com e

0, ento m

e.

Assim, por exemplo: mmc (39,102,75) = 33150. Exemplo 6.7 Achar inteiros x, y e z que verifiquem a seguinte igualdade 11x + 19y + 3z = 1. Achando o mdc(11,19) 1 19 11 8 1 8 3 2 3 2 1 2 1 2 1 0

Usando o algoritmo da diviso, podemos escrever: 19 = 11 x 1 + 8 11 = 8 x 1 + 3 8=3x2+2 3=2x1+1 2=1x2 Desprezando a ltima igualdade, eliminemos os restos a partir da penltima igualdade: 1=32 1 = 3 (8 3 x 2) 1= 3x38 1 = (11 8) x 3 8 1 = 11 x 3 8 x 4 1 = 11 x 3 (19 11) x 4 1 = 11 x 7 19 x 4 (I)

Achemos agora o mdc(3,1) . Como mdc(3,1) = 1 , vamos escrever este mdc como combinao de 3 e 1: 1 = 3 x 1 + 1 x (-2) ( II ) Agora substituamos o valor de 1, dado na igualdade ( I ) , na igualdade ( II ): 1 = 3 x 1 + (11 x 7 19 x 4) (2) 1 = 3 x 1 + 11 (14) + 19 x 8 ou 1 = 11 (14) + 19(8) + 3(1) . Logo x = 14, y = 8 e z = 1

77

CAPTULO 6 ALGORITMO DE EUCLIDES MNIMO MLTIPLO COMUM

EXERCCIOS
1. Usando o algoritmo de Euclides, determinar: a) b) c) d) e) f) 2. mdc(306, 657) mdc(272, 1479) mdc(884, 1292) mdc(-816, 7209) mdc(7469, 2387) mdc(-5376,-3402) Calcula-se o mdc pelo algoritmo de Euclides e a seguir divide o produto ab pelo mdc(a, b) a) b) c) d) e) f) g) h) 8. mmc( 45, 21). mmc(83, 68) mmc( 120, 110) mmc(86, 71) mmc(224, 192) mmc(1287, 507) mmc(143, 227) mmc(306, 657)

Usando o algoritmo de Euclides, determinar: a) mdc(285, 675, 405) b) mdc(209, 299, 102) c) mdc(69, 398, 253)

3.

Usando o algoritmo de Euclides, achar os inteiros x e y que verifiquem cada uma das seguintes igualdades: a) b) c) d) mdc(56, 72) = 56x + 72y mdc(24, 138) = 24x + 138y mdc(119, 272) mdc(1769, 2378) = 1769x + 2378y

O mdc de dois inteiros positivos a e b 8 e na sua determinao pelo algoritmo de Euclides os quocientes sucessivamente obtidos foram 2, 1, 1 e 4. Calcular a e b. Determinar os inteiros positivos a e b sabendo: a) ab = 4032 e mmc(a, b) = 336 b) mdc(a, b) = 8 e mmc(a, b) = 560 c) a + b = 589 e mmc(a, b)/mdc(a,b) = 84

9.

4.

Achar os inteiros x e y que verifiquem cada uma das seguintes igualdades: a) 78x + 32y = 2. b) 104x + 91y = 13 c) 31x + 19y = 7 d) 42x + 26y = 16. e) 288x + 51x = 3. f) 52x + 13y = 1 g) 145x + 58y = 87 h) 17x + 5y = -2 Achar os inteiros x, y e z que verifiquem cada uma das seguintes igualdades. a) 11x + 19y +5z = 1. b) 56x + 6y + 32z = 2. c) 6x + 3y + 15z = 9. d) 14x + 7y + 21z = 4. Achar inteiros x, y e z que verifiquem a igualdade 198x + 288y + 512z = mdc(198, 288, 512) Calcular As solues de todos os itens podem ser obtidas a partir da propriedade mdc(a, b).mmc(a, b) = a . b.

10. Demonstrar que se a e b so inteiros positivos tais que o mdc(a, b) = mmc(a, b) ento a = b. 11. Sabendo que o mdc(a,b) = 1 , demonstrar: a) mdc(2 a + b, a + 2 b) = 1 ou 3 b) mdc(a + b, a2 + b2) = 1 ou 2 c) mdc(a + b, a2 ab + b2) = 1 ou 3 12. Sendo a e b inteiros positivos, demonstrar que o mdc(a, b) sempre divide o mmc(a, b). 13. Quantos pares de inteiros positivos A e B existem cujo mnimo mltiplo comum 126000? Considere o par (A,B) como sendo o mesmo que (B,A) 14. Calcule a soma dos nmeros entre 200 e 500 que so mltiplos de 6 ou de 14, mas no simultaneamente mltiplos de ambos."

5.

6.

7.

15. O mximo divisor comum, o menor divisor


comum e o mnimo mltiplo comum dos nmeros 4, 8 e 12, so, respectivamente.

78

Captulo 7

NMEROS PRIMOS
7.1. Introduo

A noo de nmero primo foi, muito provavelmente, introduzida por Pitgoras,

530 AC,

sendo que a mesma desempenhou um papel central tanto na matemtica como no misticismo pitagrico. A escola pitagrica dava grande importncia ao nmero um, que era chamada de unidade (em grego: Monad). Os demais nmeros inteiros naturais o 2, 3, 4, etc tinham carter subalterno, sendo vistos como meras multiplicidades geradas pela unidade e por isso recebiam a denominao de nmero (em grego: Arithms). Entre os pitagricos a preocupao com a gerao dos nmeros no parava por a. J o prprio Pitgoras teria atinado que existem dois tipos de arithms: Os protoi arithms (nmeros primrios ou primos), que so aqueles que no podem ser gerados atravs da multiplicao por outros arithms, como o caso de 2, 3, 5, 7... Os deuteri arithms (nmeros secundrios), podem ser gerados por outros arithms, por exemplo, 4 = 2.2, 6 = 3.2, etc. Ainda por influncia dos Pitagricos , por muitos sculos houve polemica a respeito da primalidade do nmero dois. Os primeiros pitagricos chamavam-lhe Dyad, atribuam-lhe carter especial embora menos importante que a unidade Monad e alguns deles no o incluam entre os arithms. Consequentemente, muitos pitagricos no consideravam o dois como primo. s pela poca de Aristteles, 350 AC, que passou a ser considerado como

primo, sendo que este costume foi consagrado pelo livro Elementos de Euclides em cerca de

300 AC. Cabe mencionar que entre os gregos, principalmente os pitagricos de vrias geraes aps Pitgoras, surgiram outras denominaes para os nmerosprimos,
como: retilneos, lineares e eutimtricos. Contudo, esta nomenclatura teve uso muito restrito e caram em desuso.

79

CAPTULO 7 NMEROS PRIMOS

Registros de documentos gregos


Foi supracitado que a noo de primo fora, muito provavelmente, introduzida por Pitgoras. Com efeito, impossvel ter completa segurana nessa atribuio, pois Pitgoras no deixou nenhum registro escrito de seus trabalhos e os documentos mais antigos que temos falando de suas idias resumem-se a pequenos fragmentos de textos escritos vrias geraes aps ele. Entretanto, esses fragmentos, apesar de conterem informaes muito escassas, so unnimes em afirmar que Pitgoras iniciou o estudo de nmeros primos. O mais antigo livro de matemtica que chegou completo aos nossos dias e que desenvolve sistematicamente o estudo de nmeros primos Os Elementos de Euclides. Como sabido, Euclides seguiu muito de perto as orientaes matemticas dos pitagricos. Assim no surpreendente que, no captulo em que trata da teoria dos nmeros, ele defina nmero primo de um modo absolutamente compatvel com as idias pitagricas expostas acima. Elementos, Vol. VII, def 11, temos: prots arithms estin monadi mone metroymenos. Ou seja: Nmero primo todo aquele que s pode ser medido atravs da unidade. Surgimento da denominao latina A arithmetik do grego Nikomachos, 100 dC, o mais antigo livro de Teoria dos Nmeros, posterior a Elementos de Euclides, que chegou aos nossos dias. Trata-se de uma viso de filsofo e letrado em Elementos, sendo que no h uma nica demonstrao entre os poucos tpicos abordados. Apesar disso, teve grande repercusso na poca e foi a base do primeiro livro em latim que se escreveu sobre Teoria do Nmeros: o De Institutione Arithmetica, do romano Boethius 500 dC. No livro de Boethius onde aparece, pela primeira vez, a nomenclatura numerus primus como traduo do tradicional prots arithms preservada de Euclides por Nikomachos. Alm disto, Boethius, sempre seguindo Nikomachos, usa a velha classificao pitagrica dos nmeros naturais: primos incompostos versus secundrios ou compostos. O livro de Boethius foi, durante cerca de seiscentos anos, a nica fonte de estudos de Teoria dos Nmeros disponvel na Idade Mdia. Em torno de 1200 dC iniciou o renascimento cientfico e matemtico pela Europa, com o afluxo das obras rabes e a traduo das obras gregas preservadas no Mundo Islamita. dessa poca um dos mais influentes livros de todos os tempos: o Liber Abacci, de Fibonacci. Esse grande matemtico, que havia estudado entre os muulmanos do Norte da frica, diz que acha melhor dizer primus em vez do incomposto preferido pelos rabes. Ficou assim, definitivamente, consagrada a denominao nmero primo na Europa. (http://www.mat.ufrgs.br/~portosil/pqprimo.html)

80

CAPTULO 7 NMEROS PRIMOS

7.2. Nmeros Primos (do lat. primus, principal. Prime em ingls)

Definio 7.1: Diz-se que um nmero positivo p > 1 um nmero primo ou apenas um primo se, e somente se, 1 e p so seus nicos divisores positivos. Um inteiro maior que 1 e que no primo diz-se composto. Teorema 7.1: Se um nmero primo p no divide um inteiro a, ento a e p so relativamente primos (primos entre si). Demonstrao: Seja d o mdc de a e p. Ento d | a e d | p. Da relao d | p, resulta que d = 1 ou d = p, porque p primos, e como a segunda igualdade impossvel, porque p no divide a, segue-se que d = 1, isto , o mdc ( a , p ) = 1. Logo, a e p so relativamente primos. Corolrio 7.1: Propriedade Fundamental dos Nmeros Primos. Se p um primo tal que p | ab, ento p | a ou p | b (podendo ser fator de ambos, a e b). Demonstrao: Se p | a, nada h que demonstrar, e se, ao invs, p no divide a, ento, pelo teorema anterior, o mdc (p, a) = 1. logo, pelo teorema 5.4, p | b.

Nota: Observemos que esta propriedade necessria dos nmeros primos tambm suficiente para que um inteiro positivo n seja primo: Pois, se n = k. s composto (1< s k < n) , temos n| k.s porm tanto n | k e n | s .

Corolrio 7.2: Se p um primo tal que p | a1a2a3 ... an, ento existe um ndice k, com 1 n tal que p | ak. Demonstrao:

Usando Induo, a proposio verdadeira para n = 1(imediato) e para n = 2 (pelo corolrio 5.1). Supondo, pois, n > 2 e que, se p divide um produto com menos de n fatores, ento p divide pelo menos um dos fatores (hiptese de induo). Pelo corolrio 7.1, se p|a1 a2 . . . an-1, ento p|an ou p|a1 a2 ... an-1. Se p|an, a proposio est demonstrada, e se, ao invs, p|a1 a2 ... an-1, ento a hiptese de induo assegura que p|ak, com 1 k n - 1. Em qualquer dos casos, p divide um dos inteiros a1, a2, a3, ..., an. Corolrio7.3: Se os inteiros p, q1,q2 ,..., qn so todos primos e se p | q1q2 ... qn, ento existe um ndice k, com 1 k n tal que p = qk.

81

CAPTULO 7 NMEROS PRIMOS

Demonstrao: De fato, pelo corolrio 7.2, existe um ndice k, com 1 k n , tal que p|qk, como os nicos divisores positivos de qk so 1 e qk, porque qk, segue-se que p = 1 ou p = qk. Mas, p > 1, porque p primo. Logo, p = qk. Teorema 7.2: Todo inteiro composto possui um divisor primo.

Demonstrao: Seja a um inteiro composto. Consideremos o conjunto A de todos os divisores positivos de a, exceto os divisores 1 e a, isto : A = { x | a; 1 < x < a } Pelo Princpio da Boa Ordenao existe o elemento mnimo p de A. que vamos mostrar ser primo. De fato, se p fosse composto admitiria pelo menos um divisor d tal que 1 < d < p, e ento d|p e d|a, o que implica d|a, isto , p no seria o elemento mnimo de A, se fosse composto. Logo, p primo.

7. 3. Teorema Fundamental da Aritmtica.

Todo inteiro positivo n > 1 igual a um produto de fatores primos. Demonstrao: Mostraremos a existncia da fatorao por induo. Se n primo no h o que provar (escrevemos m = 1, p1 = n). Se n composto podemos escrever n = ab, a, b N , 1 < a < n, 1 < b < n. Por hiptese de induo, a e b se decompem como produto de primos. Juntando as fatoraes de a e b (e reordenando os fatores) obtemos uma fatorao de n.

Nota: Este teorema (como qualquer outro teorema chamado de "fundamental") no deveria ser aplicado sem a devida precauo. Existem inmeros sistemas numricos nos quais a fatorao no nica. Por exemplo, imagine um sistema que tenha apenas inteiros pares com a adio e multiplicao usual e denominemos um nmero de "e-primo" se ele no for o produto de dois outros nmeros pares. Neste caso, alguns "e-primos" so 2, 6, 10, 14, 18, ... Observe que neste sistema, 36 possui duas fatoraes diferentes, 6x 6 e 2 x 18. (http://primes.utm.edu/) Corolrio 7.4: A decomposio de um inteiro positivo n > 1 como produto de fatores primos nica, a menos da ordem dos fatores.

82

CAPTULO 7 NMEROS PRIMOS

Demonstrao: Suponha que n = p1 ... pm = q1 ... qr com, p1 ... pm, q1 ... qr . Como p1| q1... qr temos p1 | qi para algum valor de i, donde, como qi primo, p1 = qi e p1 q1 . Analogamente temos q1 p1, donde p1 = q1. Mas por hiptese de induo

admite uma nica fatorao, donde m = r e pi = qi para todo i. Corolrio 7.5: Todo inteiro positivo n > 1 admite uma nica decomposio da forma

onde, para i =1,2,... , r cada ki um inteiro positivo e cada pi um primo, com p1 < p2 < ... < pr, denominada decomposio cannica do inteiro positivo n > 1. Demonstrao: Pelo teorema Fundamental da Aritmtica, n um produto de fatores primos q1 . q2 ... qm, com q1 q2 ... qm (m 1). Agrupando-se os fatores primos repetidos na forma de potncias de primos, temos a representao enunciada neste corolrio e, pelo Teorema Fundamental da aritmtica, tal representao nica.

Nota: Conhecidas as decomposies cannicas de dois inteiros positivos a > 1 e b > 1, o mdc (a, b) o produto dos fatores primos comuns s duas decomposies cannicas tomados cada um com o menor expoente, e o mmc (a, b) o produto dos fatores primos comuns e no comuns s duas decomposies cannicas tomados cada um com o maior expoente.

Corolrio 7.6: Se p1.p2 ... pn divide ar, ento p1 . p2 ... pn divide a, onde p1 p2 ... pn o produto de n primos e n e r so inteiros positivos.

83

CAPTULO 7 NMEROS PRIMOS

Demonstrao: Se p1 p2 ... pn no divide a, ento a no nenhum dos primos p1p2 ... pn Seja pi, 1 i n , um desses primos. Ento, pi tambm no fator primo de ar e, desta forma, no existe pi, 1 i n, que divida ar, o que implica que p1 . p2 ... pn no divide ar. Por contraposio, temos a demonstrao pedida.

Observao: O fato de que os expoentes dos primos pi sejam 1 essencial. Por exemplo 4 = 22 divide 62 = 36 , mas 4 no divide 6.

7.4. A Seqncia dos Nmeros Primos


Teorema 7.4: (de Euclides): H um nmero infinito de primos. Demonstrao: Suponha por absurdo que p1, p2, ..., pm fossem todos os primos. O nmero P = p1 . p2 ... pm + 1 > 1 no seria divisvel por nenhum primo, o que contradiz o teorema fundamental da aritmtica. Proposio 7.1. Para o n-simo nmero primo pn vale a estimativa Demostrao: Para n = 1 verdade que desigualdades . Suponhamos j provadas as .

Se q primo tal que q | p1 . p2 ... pn + 1, ento q > pn, particularmente q Ento,

pn+1.

Esta estimativa exageradamente fraca, no geral pn significativamente menor que por exemplo = 256 e p4 apenas 7. Uma estimativa melhor para pn, postulada por Bertrand e, demonstrada por Chebychev, dada pelo teorema seguinte:

84

CAPTULO 7 NMEROS PRIMOS

Teorema 7.5 (de Chebychev): Para todo inteiro m 2 existe um primo p com m < p< 2m. A demonstrao deste teorema est fora de nosso contexto. Um outro fato provado que entre dois cubos consecutivos existe sempre um primo. Com esse Teorema podemos afirmar que pn+1 < 2pn (para n 1) 2n.

Corolrio 7.6. Para o n-simo nmero primo pn vale a estimativa pn Demonstrao:

Temos 2 = p1 21 e pelo teorema de Chebychev: Para todo inteiro positivo n, tem-se pn < pn+1 < 2 . pn. De pn 2n segue que pn + 1 2. 2n = 2n+1. Ao estudarmos a sequncia de nmeros primos percebemos que existem infinitos primos em subconjuntos particulares dos inteiros, como, por exemplo, na sucesso aritmtica: {4q + 3; q inteiro e q 0} = {3, 7, 11, 15, 19, ...}. Esse resultado foi generalizado pelo matemtico alemo Peter Gustav Le-jeune Dirichlet (1805-1859). Teorema 7.6 (de Dirichlet). Sejam a e b inteiros primos entre si, isto , mdc (a, b) = 1. Existem infinitos primos da forma an + b, onde n inteiro positivo. A demonstrao deste Teorema exige avanados conhecimentos de Anlise Matemtica. Exemplos: Na sequncia 3n + 1, temos os primos 7, 13, 19, 31, 37, 43, 61, 67, 73, 97, 103, ... Na sequncia 9n + 4 temos os primos 13, 31, 67, 103, 139, 157, 193, 211, ... O resultado de Dirichlet diz no s que o nmero de primos infinito, mas tambm que, se considerarmos subconjuntos particulares de inteiros, como as sucesses aritmticas acima, teremos j nesses subconjuntos uma infinidade de primos. Uma aplicao do Teorema de Dirichlet leva-nos a um resultado obtido pelo matemtico polons W. Sierpinski , que nos mostra, mais uma vez, a forma surpreendente como os primos se distribuem nos inteiros. Teorema 7.7 (de Sierpinski). Dado um inteiro m maior que 1, existe um primo p tal que |p 1|, |p 2|, ..., |p m| so compostos. Exemplo: Seja m = 10 e p = 19. Temos: 19+1, 19+2, 19+3, 19-4, 19+5, 19+6, 19+7, 19+8, 19+9 e 19-10. Os resultados so todos nmeros compostos: 20, 21, 22, 15, 24, 25, 26, 27, 28 e 9. Observe que se tivssemos escolhido o primo 17, no seria possvel construir uma sequncia de compostos com m = 10, pois 17 + 6 = 23 e 17 6 = 11, ambos primos. Demonstrao: Vejamos, em primeiro lugar, que existe um primo p tal que p + 1, p + 2, ..., p + m sejam compostos. Para cada m dado, o Teorema 1 garante, em particular, que existe um inteiro primo q maior do que m. Seja a = (q + 1) (q + 2) (q + 3) ... (q + m). Se q divide a, ento q divide q + i, e, portanto, q divide i, o que impossvel para 0 < i m < q. Ento a e q so primos entre si. Pelo teorema de Dirichlet, existe um primo p na sequncia

85

CAPTULO 7 NMEROS PRIMOS

an + q. Seja p = (q +1) (q + 2) ... (q + m) n + q este primo. Ento os nmeros p + 1, p + 2, ..., p + m so m nmeros compostos. Para ampliar este resultado, observemos que, por motivos anlogos aos de cima, a' = (q - m).[q - (m- 1)]...(q - 1)...(q + m) primo com q e se p' for um primo da sequncia a'n + q, isto , p' = (q - m) ... (q - 1) . (q + 1) ... (q + m) . n + q os nmeros p' m, ... p' 1, p' + 1, ... p' + m sero compostos. Assim o nmero primo p' se encontra na sucesso dos inteiros, "isolado" por m compostos de cada lado. (RPM 11)

7.5. O Crivo de Eratstenes.


Eratstenes, matemtico, astrnomo, historiador, gegrafo e filsofo grego, nasceu em Cirene por volta de 276 a.C. e passou grande parte de sua juventude em Atenas. Com aproximadamente 40 anos, foi convidado pelo rei Ptolomeu III do Egito para ser bibliotecrio da Universidade de Alexandria. Ficou conhecido como Beta, e a respeito dessa alcunha existem algumas hipteses. Alguns acreditam que, por causa de seu saber, foi elevado condio de um segundo Plato. Outros, dizem que tal apelido lhe fora dado por ter sido o segundo bibliotecrio da Universidade de Alexandria. Uma terceira explicao sugere que, apesar de ser talentoso, Eratstenes no conseguiu ser o primeiro de seu tempo em nenhum ramo de estudo, em outras palavras, foi sempre o segundo. Por fim, o historiador James Gow sugeriu que talvez Beta indicasse simplesmente o nmero (grego) 2 referente a um gabinete ou a uma sala de leitura da universidade. Escreveu diversas obras, mas muitas se perderam, inclusive o tratado Sobre a medida da Terra. Eratstenes morreu em Alexandria, em 194 a.C. (http://www.moderna.com.br/moderna/didaticos/ef2/matematica/erato/bio_eratostenes.htm) Teorema 7.8: Se um inteiro positivo a > 1 composto, ento a possui um divisor primo p . Demonstrao: Com efeito, se o inteiro positivo a > 1 composto, ento: a = bc, com 1 < b < a e 1 < c < a Portanto, supondo b c, teremos: b2 bc = a b

86

CAPTULO 7 NMEROS PRIMOS

O teorema 7.8 fornece um processo que permite reconhecer se um dado inteiro a >1 primo ou composto, para o que basta dividir a sucessivamente pelos primos que no excedem o valor . Tal resultado a base do chamado Crivo de Eratstenes que veremos em seguida. Uma questo natural sobre os nmeros primos a de determinar, dentre os inteiros positivos, todos os nmeros primos at certo nmero dado. Esta questo tambm foi resolvida na antiguidade por Eratstenes. A ele devemos o chamado Crivo de Eratstenes. Com o crivo de Eratstenes podem-se determinar, sem auxlio de mquinas, todos os nmeros primos at 200, 400 ou 500, por exemplo. Com o auxlio de computadores, o crivo de Eratstenes, convenientemente adaptado, permite determinar os nmeros primos at limites bem altos. Mesmo antes dos computadores, j haviam sido determinados os nmeros primos at 10.000.000. Isto ocorreu por volta de 1914, por obra do matemtico americano D. N. Lehmer. Dois outros matemticos (Bays e Hudson) calcularam, em 1976, (usando computadores, evidentemente!), a tabela dos nmeros primos at 12 x 1011. Alm disso, h tabelas de nmeros primos em determinados intervalos de inteiros e conhecem-se tambm nmeros primos bem grandes, como o nmero 244497 1, que possui 13395 algarismos! (RPM 19) A construo de uma tabela de nmeros primos que no excedam um dado inteiro n usando o Crivo de Eratstenes consiste no seguinte: escrevem-se na ordem natural todos os inteiros a partir de 2 at n e, em seguida, eliminam-se todos os inteiros compostos que so mltiplos dos primos p tais que p isto , 2p, 3p, 4p, ... Exemplo: Construir a tabela de nmeros primos menores que 200. Soluo: Como , basta eliminar sucessivamente da tabela os nmeros que so mltiplos dos primos p menores que 14, ou seja, 2, 3, 5, 7, 11 e 13.

. 11 21 31 41 51 61 71 81 91 101 111 121 131

2 12 22 32 42 52 62 72 82 92 102 112 122 132

3 13 23 33 43 53 63 73 83 93 103 113 123 133

4 14 24 34 44 54 64 74 84 94 104 114 124 134

5 15 25 35 45 55 65 75 85 95 105 115 125 135

6 16 26 36 46 56 66 76 86 96 106 116 126 136

7 17 27 37 47 57 67 77 87 97 107 117 127 137

8 18 28 38 48 58 68 78 88 98 108 118 128 138

9 19 29 39 49 59 69 79 89 99 109 119 129 139

10 20 30 40 50 60 70 80 90 100 110 120 130 140

87

CAPTULO 7 NMEROS PRIMOS

141 151 161 171 181 191

142 152 162 172 182 192

143 153 163 173 183 193

144 154 164 174 184 194

145 155 165 175 185 195

146 156 166 176 186 196

147 157 167 177 187 197

148 158 168 178 188 198

149 159 169 179 189 199

150 160 170 180 190 200

Os valores em vermelho so os nmeros primos que no foram riscados da tabela.

Listamos a seguir a os 199 primeiros nmeros primos: 2, 3, 5, 7, 11, 13, 17, 19, 23, 29, 31, 37, 41, 43, 47, 53, 59, 61, 67, 71, 73, 79, 83, 89, 97, 101, 103, 107, 109, 113, 127, 131, 137, 139, 149, 151, 157, 163, 167, 173, 179, 181, 191, 193, 197, 199, 211, 223, 227, 229, 233, 239, 241, 251, 257, 263, 269, 271, 277, 281, 283, 293, 307, 311, 313, 317, 331, 337, 347, 349, 353, 359, 367, 373, 379, 383, 389, 397, 401, 409, 419, 421, 431, 433, 439, 443, 449, 457, 461, 463, 467, 479, 487, 491, 499, 503, 509, 521, 523, 541, 547, 557, 563, 569, 571, 577, 587, 593, 599, 601, 607, 613, 617, 619, 631, 641, 643, 647, 653, 659, 661, 673, 677, 683, 691, 701, 709, 719, 727, 733, 739, 743, 751, 757, 761, 769, 773, 787, 797, 809, 811, 821, 823, 827, 829, 839, 853, 857, 859, 863, 877, 881, 883, 887, 907, 911, 919, 929, 937, 941, 947, 953, 967, 971, 977, 983, 991, 997, 1009, 1013, 1019, 1021, 1031, 1033, 1039, 1049, 1051, 1061, 1063, 1069, 1087, 1091, 1093, 1097, 1103, 1109, 1117, 1123, 1129, 1151, 1153, 1163, 1171, 1181, 1187, 1193, 1201, 1213 ,1217.

Nota: Podemos fazer um crivo mais econmico, j que no possvel evitar completamente o fato de que alguns nmeros so riscados vrias vezes. Podemos proceder da seguinte maneira: Primeiro escrevemos uma lista com os mpares de 3 at n. Como queremos riscar os nmeros de p em p claro que os mltiplos de p que so mltiplos de primos menores que p j foram riscados da lista. Ento, nesta etapa, podemos comear a riscar de p em p a partir do menor mltiplo de p, que no mltiplo de um primo menor que p; isto , a partir de p2. Isto evita muitas duplicaes.[Coutinho]

3, 5, 7, 32, 11, 13, 15, 17, 19, 21 23, 52, 27, 29, 31, 33, 35, 37, 39, 41, 43, 45, 47, 72, 51, 53, 55, 57, 59, 61, 63, 65, 67, 69, 71, 73, 75, 77, 79, 81, 83, 85, 87, 89, 91, 93, 95, 97, 99.

88

CAPTULO 7 NMEROS PRIMOS

Definio 7.2: Para qualquer nmero real x > 0, seja (x) o nmero de primos p (x) quantidade dos nmeros primos menores que ou iguais a x. Tabela dos 20 primeiros valores inteiros da Funo p (x) x 1 2 3 4 5 6 7 8 9 10 11 12 13 14 15 16 17 18 19 20 (x) 0 1 2 2 3 3 4 4 4 4 5 5 6 6 6 6 7 7 8 8

x , isto ,

De acordo com o Teorema de Chebychev podemos afirmar que (2n) (n) 1 (para n 2)

Nota: Um problema prtico, onde as propriedades dos nmeros primos tm reflexos importantes, o problema do reconhecimento da fala por computadores que exige o desenvolvimento de algoritmos to rpidos quanto possvel para a decomposio de sons nas suas frequncias fundamentais, uma tcnica conhecida como Anlise de Fourier. A velocidade terica mxima desses algoritmos esta diretamente relacionada com a funo (x) que fornece o numero de primos menores que ou iguais a x. Frmula de Minc: Dado um inteiro m contagem dos nmeros primos (m) : 2 , J. Minc estabeleceu uma frmula para a

Demonstrao: Ser vista aps estudarmos o Teorema De Wilson.

89

CAPTULO 7 NMEROS PRIMOS

Exemplo:

(6) = 1 + 1 + 0 + 1 + 0 = 3. Resultado que nos diz que existem trs primos antes do nmero seis.

Frmula Para o n-simo Nmero Primo

Devido ao resultado acima podemos escrever uma frmula que nos retorna o n-simo nmero primo estabelecida por C. P. Willans em 1964:

Exemplo:

Definio 7.3: Para todo nmero primo p, seja p# o produto de todos os nmeros primos q p . p# chamado o primorial de p. Tabela dos 17 primeiros Primoriais P 2 3 5 7 11 13 17 23 29 p# 2 6 30 210 2310 30030 510510 9699690 223092870

90

CAPTULO 7 NMEROS PRIMOS

31 37 41 43 47 53 59 61

6469693230 200560490130 7420738134810 304250263527210 13082761331670030 614889782588491410 32589158477190044730 1922760350154212639070

Teorema 7.8: p# +1 no possui nenhum fator primo menor do que ou igual a p. Demonstrao: Suponhamos, por contradio, que p# + 1 seja divisvel por um primo q p . Ou seja, existe um inteiro positivo s tal que tal que p# + 1 = q.s, isto q.s p# =1. Como q p , ento q necessariamente um fator de p#. Logo q divide ambas as parcelas da diferena q.s p#. Portanto q divide 1, o que um absurdo uma vez que q primo. Nota: Veja que resultado interessante:

Leitura: A Distribuio dos Nmeros Primos Ao contemplar uma tabela de nmeros primos, a primeira impresso que se tem a de que no h nenhuma ordem entre os nmeros primos: s vezes eles aparecem prximos uns dos outros, s vezes afastados, ora menos, ora mais afastados; enfim, analisando-os individualmente ou em pequenos grupos, no divisamos qualquer regularidade em sua distribuio. Entretanto, a sagacidade de inteligncias privilegiadas consegue ver mais fundo, e foi precisamente isso o que aconteceu por obra do matemtico francs Adrien - Marie Legendre (1752-1833). Ele se ocupou dessa questo e por volta de 1800 formulou uma conjectura que revela certa ordem no que parecia ser um caos completo. Para explicarmos a conjectura de Legendre, introduzimos o smbolo (x) como sendo o nmero de nmeros primos at certo valor x. Assim, (8) = 4, ou seja, o nmero de nmeros primos at 8 4; (11) = 5, pois h cinco nmeros primos at 11, precisamente, 2, 3, 5, 7, 11; e assim por diante. Pois bem, o que Legendre conjecturou, empiricamente, analisando tabelas de nmeros primos (em 1797 uma dessas tabelas foi publicada, contendo todos os nmeros primos at 400031), que (x) podia ser aproximado pela funo (o logaritmo que aqui aparece o logaritmo natural, isto , na base e 2,718281...), e que essa aproximao seria tanto melhor quanto maior fosse x. Mas isto deve ser entendido em termos relativos, isto , o erro que se comete tomando em lugar de (x) torna-se tanto menor quanto maior for x, relativamente a Em outras palavras, seja

91

CAPTULO 7 NMEROS PRIMOS

o erro que se comete ao tomar crescer de x o erro relativo

em lugar. de (x). Pois bem, o que se torna pequeno com o

(2) Este erro pode ser feito, em valor absoluto, to pequeno quanto quisermos, desde que faamos x suficientemente grande. Carl Friedrich Gauss (1777-1855), que considerado por muitos o maior matemtico de todos os tempos, conta, numa carta de 1849, publicada vrios anos mais tarde, que quando ainda bem jovem, com apenas 15 anos de idade, pensou muito sobre a distribuio dos nmeros primos, chegando a conjecturar algo equivalente ao que conjecturou Legendre. Seja como for, essa conjectura logo impressionou os matemticos como algo notvel, pois quem diria que a seqncia dos nmeros primos pudesse ter algo a ver com a funo logaritmo! A descoberta de Legendre e Gauss demorou a ser demonstrada. Embora ela tenha sido objeto da ateno dos melhores matemticos do sculo, desafiou a argcia desses homens por cerca de 100 anos. De fato, foi somente em 1896 que ela foi demonstrada pela primeira vez. E nesse mesmo ano apareceram duas demonstraes, uma pelo matemtico francs Jacques Hadamard (1865-1963) e outra, pelo belga Charles de Ia Valle Poussin (1866-1962). Essas demonstraes, independentes uma da outra, baseavam-se nas idias de um outro grande matemtico do sculo, Bernhard Riemann (1826-1866). Embora no tenha logrado demonstrar a conjectura de Legendre e Gauss, Riemann, num memorvel trabalho intitulado Sobre o nmero de nmeros primos menores que um certo nmero, deixou ideias notveis sobre teoria dos nmeros, que vm sendo exploradas pelos estudiosos do assunto at os dias de hoje. Antes mesmo das demonstraes de Hadamard e de la Valle Poussin, o matemtico russo Pafnutii Chebyshev (1821-1894) provou, por volta de 1850, um resultado prximo conjectura de Legendre e Gauss. Segundo Chebyshev, existem constantes positivas c e C (c 0,92, C 1, 106) tais que (3) Para bem entendermos o significado da aproximao (4) vamos comparar os grficos das funes y = x e y = log x. Eles nos revelam que ambas as funes crescem com o crescer de x, tendendo a infinito.

92

CAPTULO 7 NMEROS PRIMOS

No entanto, como podemos ver, claramente, a primeira dessas funes cresce mais depressa que a segunda, distanciando-se mais e mais desta ltima, medida que x cresce acima de qualquer nmero dado. Isto fica mais claro ainda quando levamos em conta que o grfico do logaritmo tem a concavidade voltada para baixo, significando que, embora esta funo esteja crescendo sempre com o crescer de x, trata-se de um crescimento cada vez mais lento, quanto maior for x. Isto quer dizer que o quociente no segundo membro de (4) tambm cresce, tendendo a infinito com o crescer de x, o que est de acordo com o fato de que existem infinitos nmeros primos, isto , (x) cresce acima de qualquer nmero, desde que faamos x suficientemente grande. No obstante tudo isso, o erro absoluto expresso em (1) pode tornarse muito grande, mas no o erro relativo expresso em (2); este tende a zero, isto , pode ser feito menor do que qualquer nmero positivo dado, desde que faamos x suficientemente grande. Uma concluso simples que podemos tirar de (4) que, em certo sentido, os nmeros primos vo ficando cada vez mais raros, medida que avanamos na seqncia dos nmeros naturais. Para bem entender o que estamos dizendo, observe que significa de sorte que a densidade mdia dos nmeros primos no intervalo que vai de 1 at x. O fato de que essa densidade decresce com o crescer de x significa precisamente o que dissemos acima: os nmeros primos vo ficando cada vez mais raros, medida que avanamos na seqncia dos nmeros naturais. ( RPM19)

Definio 7.3 Chamam-se primos gmeos dois inteiros positivos mpares e consecutivos que so ambos primos. Em outras palavras, dizemos que dois primos mpares so gmos quando a diferena entre eles igual a 2. Assim, por exemplo, so pares de primos gmeos: 3 e 5, 5 e 7, 11 e 13, 17 e 19, 29 e 31 No se sabe at hoje se h um nmero infinito de pares de primos gmeos, mas so conhecidos primos gmeos muito grandes, tais como: 140.737.488.353.507 e 140.737.488.353.509 140.737.488.353.699 e 140.737.488.353.701 Um fato interessante a existncia de apenas um terno de inteiros positivos mpares e consecutivos que so todos primos: 3, 5 e 7.

93

CAPTULO 7 NMEROS PRIMOS

7.6. Seqncia de Inteiros Consecutivos Compostos


Existem, na sequncia dos primos, primos consecutivos to afastados quanto se deseje. Ou seja, existem saltos arbitrariamente grandes na seqncia dos primos. Teorema 7.9: Dado um inteiro positivo n >1, possvel determinar n inteiros consecutivos tais que nenhum deles seja primo. Demonstrao: De fato, evidente que na sequncia: (n + 1)! + 2, (n + 1)! +3, (n + 1)! + 4, ..., (n + 1)! + (n + 1) os seus n termos so inteiros positivos consecutivos, e cada um deles composto, porque (n +1)! + j divisvel por j se 2 j n + 1. Assim, por exemplo, supondo n = 4, obtemos a sequncia: 5! + 2, 5! + 3, 5! + 4, 5! + 5 Cujos termos so 4 inteiros positivos consecutivos, cada um dos quais composto, pois, temos: 5! + 2 = 122 = 2 . 61, 5! + 3 = 123 = 3 . 41 5! + 4 = 124 = 4 . 31, 5! + 5 = 125 = 5 . 25 Outras sequncias de 4 inteiros consecutivos e compostos existem, tais como 24, 25, 26, 27 e 32, 33, 34, 35 54, 55, 56, 57 e 74, 75, 76, 77 Nota: Em 1984 Samuel Yates iniciou uma lista dos "Maiores Primos Conhecidos" e criou o nome primo titnico para designar qualquer nmero primo com 1.000 ou mais dgitos decimais. Denominou tambm de tits aqueles que provaram a sua primalidade. A maioria dos primos so titnicos e dezenas de milhares deles so "conhecidos". Entretanto, na poca em que Yates definiu os primos titnicos, tinha-se conhecimento de apenas alguns poucos. Cerca de dez anos mais tarde, Yates designou como primo gigante todo nmero primo que possusse 10.000 ou mais dgitos decimais. E os Megaprimos so nmeros primos que possuam no mnimo um milho de dgitos decimais. http://www.numaboa.com.br/criptologia/matematica/primos.php

94

CAPTULO 7 NMEROS PRIMOS

Corolrio 7.7: Dado um inteiro positivo n, existem dois primos consecutivos ph, ph+1 tais que ph+1 ph > n. Demonstrao: Seja ph o maior dos primos que so menores que ( n +1 )! + 2. Ento, ph (n + 1)!+ 1. Do teorema anterior, temos ainda que ph +1 > (n + 1)! + (n + 1) Fazendo a diferena entre ambas as desigualdades, temos ph +1 > (n + 1)! + (n + 1) Exemplo: Seja n = 6, de acordo com a demonstrao podemos considerar os primos p1 = 5039 e p2 = 5059. Assim, 5059 5039 > 6, isto , 20 > 6. Teorema 7.10: O produto de qualquer sequncia de k inteiros consecutivos divisvel por k!. Demonstrao: Vamos considerar n e k inteiros positivos com k de n, tomadas k a k, um inteiro dado por: n. Sabemos que o nmero de combinaes

Sendo o numerador o produto de k inteiros consecutivos temos o resultado para uma sequncia de k inteiros positivos. No caso de zero ser um elemento na seqncia o resultado trivial, uma vez que zero divisvel por qualquer inteiro no nulo. Se a sequncia contiver s nmeros negativos, a frao do lado direito da igualdade acima sofrer, no mximo, uma mudana de sinal continuando a ser um inteiro, o que conclui a demonstrao.

95

CAPTULO 7 NMEROS PRIMOS

7.7 . Conjecturas

Conjectura de Goldbach. Em 1742, numa carta a Leonhard Euler (1707-1783), Christian Goldbach (1690-1764) expressou a seguinte conjectura: Todo inteiro n > 5 a soma de trs nmeros primos. Em resposta, Leonhard Euler observou que essa conjectura era equivalente seguinte: Todo inteiro par maior que ou igual a 4 a soma de dois primos. Esta conjectura conhecida como conjectura de Goldbach. Um romance interessantssimo sobre a dificuldade desse assunto Tio Petros e a Conjectura de Goldbach escrito por Apostolos Doxiadis e publicado pela Editora 34. Exemplos: 4=2+2 6=3+3 8=3+5 10=3+7, 5+5 12=5+7 14=3+11, 7+7 16=3+13, 5+11 18=5+13, 7+11 20=3+17, 7+13 22=3+19, 5+17, 11+11 24=5+19, 7+17, 11+13 26=3+23, 7+19, 13+13 28=5+23, 11+17 30=7+23, 11+19, 13+17 32=3+29, 13+19 34=3+31, 5+29, 11+23, 17+17 36=5+31, 7+29, 13+23, 17+19 38=7+31, 19+19 40=3+37, 11+29, 17+23 Muitos matemticos continuam tentando encontrar um contra-exemplo ou uma demonstrao para essa conjectura. Por exemplo: Georg Cantor (1845-1918), efetuou em 1894 todas as decomposies possveis, como soma de dois nmeros primos, de todos os nmeros pares inferiores a 1000. Aubry estendeu a lista de Cantor at 2000.

96

CAPTULO 7 NMEROS PRIMOS

R. Haussner em 1897 estendeu essa tabela at 5000. Em 1937 o matemtico sovitico I.M.Vinogradov demonstrou, usando somas trigonomtricas adequadas, que qualquer nmero mpar suficientemente grande soma de trs nmeros primos. Em 1966 o matemtico chins Jeng-Run Chen provou que a partir de algum nmero n, todo par maior que 2 ou soma de dois primos, ou a soma de um primo com o produto de dois primos. O argumento de Chen no diz qual esse n; apenas demonstra que ele existe.

Alm da Conjectura de Goldbach, em Teoria dos Nmeros, particularmente em Nmeros Primos, existem muitos problemas em aberto. Segue uma lista com algumas conjecturas que, embora j tenham sido testadas para inmeros casos, ainda no foram demonstradas. Eis algumas: Todo nmero mpar maior que cinco a soma de trs primos. Esse fato j foi provado, por Vinogradov, para nmeros suficientemente grandes. Em 1956, Borodzkin mostrou que n > 314348907 suficiente. Esse nmero foi diminudo, em 1989, para 1043000, por Chen e Wang, mas ainda muito grande para que os casos menores possam ser testados com o uso de um computador. Exemplos: 7 = 3 + 2 + 2; 21 = 11 +7 + 3 ; 41 = 11 + 13 + 17; 49 = 13 + 17 + 19 Existem infinitos primos da forma k2 + 1. Exemplos: 5 = 22 +1; 17 = 42 +1; 37= 62 + 1. Existem infinitos pares de primos consecutivos (Primos Gmeos) . Exemplos: (3 e 5), (5 e 7), (11 e 13), (17 e 19), (29.879 e 29.881), ...

Em 2000, foi apresentado um par de primos gmeos cada um com 18075 dgitos. o par 4 648 619 711 505. 260000 1 Existe sempre um primo entre dois quadrados consecutivos. Exemplos: 3 entre 1 e 4; 5 e 7 entre 4 e 9; 11 e 13 entre 9 e 16, .... Primos de Sophie Germain. Um nmero primo p um nmero primo de Sophie Germain se 2p + 1 tambm primo. So famosos porque Sophie Germain provou que o ltimo Teorema de Fermat verdadeiro para estes nmeros. A existncia de um nmero infinito de tais nmeros primos uma uma afirmao ainda no provada. Os primeiros primos de Sophie Germain so 2, 3, 5, 11, 23, 29, 41, 53, 83, 89, 113, 131, 173, 179, 191, 233 ...

97

CAPTULO 7 NMEROS PRIMOS

Nota: Primos em Progresso Aritmtica. Um problema famoso que permaneceu por muito tempo em aberto, era o de provar se existiam progresses aritmticas arbitrariamente longas formadas exclusivamente por primos. Van der Corput j havia provado em 1939 que h uma infinidade de progresses aritmticas formadas por 3 primos. Ben Green do Instituto de Matemtica de Vancouver e Terence Tao da Universidade da Califrnia, provaram em 2006, que tais sequncias existem. Mas a prova no especifica como encontr-las ou entre quais primos tais sequncias se encontram. A mais longa progresso aritmtica de nmeros primos conhecida at o momento, tem 24 termos. Foi descoberta por Jaroslaw Wroblewski em janeiro de 2007: 468395662504823 + 45872132836530.k, para k = 0, 1, ..., 23.

7.8. Frmulas que geram alguns nmeros primos


Muitas tentativas tm sido realizadas para encontrar frmulas aritmticas simples que forneam somente primos. Nesta seo ser apresentada algumas frmulas famosas sobre primos. 1) Frmula de Fermat: Fermat fez sua famosa conjectura de que os nmeros da forma

so primos. Para n = 1, 2, 3, 4 obtemos: F1 = 22 + 1 = 5 F2 = + 1 = 24 + 1 = 17 F3 = + 1 = 28 + 1 = 257 F4 = + 1 = 216 + 1 = 65.537 todos primos. Porm em 1732, Euler descobriu a fatorao + 1 = 4294967297 = (641).(6700417) portanto, F(5) no primo. At este momento (05 /2005) o maior primo de Fermat conhecido F4

98

CAPTULO 7 NMEROS PRIMOS

2) Frmula de Euler: Em 1772 Leonhard Euler descobriu um polinmio tendo uma longa sucesso de valores primos, dado por F(n) = n2 + n + 41 que fornece primos para n = 1, 2, ..., 39. Entretanto, para n = 40 o valor composto: F(40) = 402 + 40 + 41 = 40. (40 + 1) + 41 = 40.41 + 41 = 41.(40 + 1) = 41.41. 3) Frmula de Mersenne: Marin Mersenne em 1644 fez a seguinte afirmao: Todo natural Mp = 2p 1 primo para os primos p = 2, 3, 5, 7, 13, 17, 19, 31, 67, 127 e 257, e composto para todos os outros primos p < 257. Entretanto, esta afirmao incorreta, pois, segundo o site http://www.mersenne.org/ prime.htm, at setembro de 2006 j eram conhecidos, 44 primos de Mersene, para os primos p = 2, 3, 5, 7, 13, 17, 19, 31, 61, 89, 107, 127, 521, 607, 1279, 2203, 2281, 3217, 4253, 4423, 9689, 9941, 1213 ,19937, 21701, 23209, 44497, 86243, 110503, 132049, 216091, 756839, 859433, 1257787, 1398269, 2976221, 3021377, 6972593, 13466917, 20996011, 24036583, 30402457 e 32582657. Esse ltimo primo tem 9.808.358 dgitos. Como se pode ver, Mersenne cometeu duas falhas: Incluiu p= 67, 257 na sua lista de primos e excluiu dessa lista p= 61, 89, 107. Somente em 1947 ( mais de 300 anos depois) a lista correta p = 2, 3, 5, 7, 13, 17, 19, 31, 61, 89, 107 e 127 onde p < 257, ficou pronta.. 4) Outras frmulas que geram alguns primos so: F(n) = n2 - n + 41para n = 1, 2, 3, 4, ..., 40 F(n) = n2 - 79n + 1601para n = 0, 1, 2, ..., 79 F(n) = n2 + n + 17 para n = 0, 1, 2, ..., 15 F(n) = 3n2 + 3n + 23para n = 0, 1, 2, ..., 21 F(n) = 6n2 + 6n + 31para n = 0, 1, 2, ..., 28 Cabe agora a pergunta: Existe algum polinmio (no-constante), com coeficientes inteiros, que fornea a sequncia dos nmeros primos ou apenas nmeros primos? Infelizmente a resposta no! Teorema 7.11: No existe polinmio algum P(x) = anxn + an-1xx-1+ ... + a0, a0 0 com coeficientes ak, 0 k n , todos inteiros, cujos valores numricos sejam sempre primos para valores inteiros da varivel x. ( VER RPM 45)

99

CAPTULO 7 NMEROS PRIMOS

Demonstrao: Suponhamos, por contradio, que o polinmio P(x), nas condies do teorema, produz sempre primos para valores inteiros da varivel x. Ento, para x = j, sendo j um inteiro fixo, P( j ) = p um primo, e qualquer que seja o inteiro s, temos: P(j + ps) = an (j + ps)n + na-1 (j + ps)n-1 + ... + a2 (j +ps)2 +a1 (j + ps) + a0 Desenvolvendo cada uma das potncias pela frmula do binmio e agrupando os primeiros termos de cada desenvolvimento, temos: P(j + ps) = (an jn + an-1 jn-1 + ... + a2j2 +a1j + a0) + pg(s) = P(j) + pg(s) = p + pg(s) onde g(s) indica um certo polinmio no constante em s com coeficientes inteiros, de grau n, logo: P(j + ps) = p(1 + g(s)) Ento, p| P(j + ps). Se P(j + ps) primo devemos ter P( j + ps) = p, donde 1+ g(s) = 1, para todo s. Temos uma contradio, pois g(s) no constante. Nota: O teorema anterior refere-se a polinmios numa varivel. Os trabalho de Putnam, Davis, Robison e Matijasevic conduziram a uma surpreendente concluso: Existe um polinmio de coeficientes inteiros, tal que o conjunto dos nmeros primos coincide com o conjunto dos valores positivos assumidos por esse polinmio, quando as variveis percorrem o conjunto dos inteiros positivos. Jones, Sato, Wada e Wiens (1976) foram os primeiros a escrever, explicitamente, um polinmio desse tipo, de grau 25 e com 26 varveis. [ Ribombim ] Leitura: Uma Frmula que Fornece todos os Nmeros Primos Sejam x e y nmeros naturais, y 0 e a = x( y + 1) - ( y!+ 1).

A frmula que d todos os nmeros primos e somente esses : f(x, y) = Por exemplo: Se x = 1 e y = 1, ento a = 0 e f(1,1) = 2; Se x = 1 e y = 2, ento a = 0 e f(1,2) = 3; Se x = 1 e y = 3. ento a = 3 e f(1,3) = 2; e, atribuindo-se a x e a y mais alguns valores, percebe-se logo que a funo f tem uma predileo muito grande pelo nmero primo 2. Mas ela fornece todos os nmeros primos:

100

CAPTULO 7 NMEROS PRIMOS

5 = f (5,4); 7 = f (103,6); 11= f (329891,10); 13 = f (36846377, 12); .... Como foram achados os pares (x,y) acima? A resposta simples: para obter o nmero primo p, calcule f(x,y) para

Assim, para obter 13, fizemos

Como se v, a frmula existe, mas no nada prtica, uma vez que envolve clculos com nmeros muito grandes(RPM 37). A demonstrao dessa frmula ser vista aps estudarmos o Teorema de Wilson.

7.9. Decomposio do Fatorial em Fatores Primos


Mostraremos como achar a fatorao em nmeros primos de n! onde n um nmero natural arbitrrio. Proposio 7.2: Sejam a 0 e b, c > 0 . Temos que

Demonstrao: Sejam.

Logo, a = bq1 + r1 , com r1 e b1

portanto, a = bq1 + r1 = b(cq2 + r2) + r1 = bcq2 + br2 + r1 como br2 + r1 b(c - 1) + b - 1 = bc - 1

101

CAPTULO 7 NMEROS PRIMOS

segue-se que o quociente da diviso de a por bc, ou seja, q2 = Dados um nmero primo p e um nmero natural m, vamos definir por Ep (m) o expoente da maior potncia de p que divide m, ou seja, o expoente da potncia de p que aparece na fatorao de m em fatores primos. Em particular, Ep (n!) representar a potncia de p que aparece na fatorao de n! em fatores primos. Teorema de Legendre. Sejam m um nmero natural e p um nmero primo. Ento Ep (n!) = Demonstrao: Note, inicialmente, que a soma acima finita, pois existe um nmero natural r tal que pi > n para todo i > r portanto , se i r Vamos demonstrar o resultado por induo sobre n . A frmula vale trivialmente para n = 0. Suponha que o resultado vale para qualquer natural m com m < n Sabemos que os mltiplos de p entre 1 e n so: p, 2 p, ..., Portanto, pela hiptese de induo, temos que

O resultado, agora, decorre da preposio 7.2. Para calcular Ep (n!) faz-se uso do seguinte algoritmo: n = pq1 + r1 q1 = pq2 + r2 ..... qs-1 = pqs + rs

Como q1 > q2 > ..., seguem-se que, para alguns s, tem-se que. Portanto, seguem-se que. E (n!) = q1 + q2 + ... + qs

102

CAPTULO 7 NMEROS PRIMOS

Exemplo: Vamos determinar a decomposio de 10! Em fatores primos. Para resolvermos o problema, devemos achar Ep (10!) para todo primo p 10. Sendo

Seguem-se que 10! = 2834527 . Lema 7.1. Sejam a1,..., am ,b inteiros positivos. Temse que

Demonstrao: Sejam qi e ri respectivamente o quociente e o resto da diviso de ai por b para i = 1, ...., m. somando, membro a membro, as igualdade ai = bqi + ri temos que a1 + ... + am = (q1 + ... + qm) b + r1 + ... + rm Seguese dai que o quociente da diviso de a1 + ... + am por b maior ou igual do que q1 + ... + qm pois r1 + ... + rm poderia superar b 1 . Isto o que se queria provar. Corolrio 7.8. Se a1,..., am,b so nmeros naturais com b 0 , ento natural o nmero

Demonstrao: De fato, pelo Lema 7.1, para todo nmero primo P e todo nmero natural i, temos que

Somando, membro a membro, as desigualdades acima, obtemos que Ep ((a1 + ... + am)!) Ep (a1 !) + ... + Ep (am)

103

CAPTULO 7 NMEROS PRIMOS

O que prova o resultado. O prximo resultado relacionar Ep ( n!) e a representao p-dica de n (i.e., a representao relativa base p) Teorema 7.12. Sejam p,n inteiros positivos, com p primo. Suponha que. n = nrpr + nr-1pr-1 + ... + n1p + no Seja a representao p dica de n . Ento.

Demonstrao: Sendo 0

ni

p , temos que

Portanto,

Exemplo: Seja determinar a potncia de 3 na decomposio de 53! em fatores primos. Primeiramente escrevemos 53 na base 3, isto : 53 = (1222)3 Aplicando o Teorema 7.12

104

CAPTULO 7 NMEROS PRIMOS

Verificando esse resultado pelo Teorema de Lagrange:

7.10. Mtodo da Fatorao de Fermat


At o momento, um dos procedimentos matemticos mais difceis o de fatorar um nmero arbitrariamente grande e isso s vezes requer um tempo razovel. Para os casos mais simples podemos usar os conhecidos testes de divisibilidade, mas fatorar nmeros grandes objeto de intensas pesquisas matemticas. Damos uma aqui um uma ideia desse difcil problema matemtico, utilizando o chamado mtodo da Fatorao de Fermat. Em cursos mais avanados outros mtodos so apresentados. Proposio 7.3: Seja n > 1 um inteiro mpar. H uma correspondncia biunvoca entre a fatorao de n e a representao de n como diferena de dois quadrados. Demonstrao: Se n = a.b, e n mpar, ento a e b so mpares. Logo a+b e a-b so pares, ento so inteiros. Ento, e

Expressa n como a diferena de dois quadrados. Reciprocamente, suponha n escrito como a diferena de dois quadrados: n = s2 t2, ento n = (s-t) . (s+t) a forma fatorada de n. Voc pode ver que esses dois procedimentos da fatorao para a diferena e da diferena para a fatorao determinam uma relao biunvoca.

7. 11 Algoritmo de Fermat
A proposio acima nos permite descrever um algoritmo, que muito eficiente quando n tem um fator primo que no muito menor que . Para comear vamos supor que n mpar, j que se n for par ento 2 um de seus fatores. A idia do algoritmo de Fermat tentar achar nmeros inteiros positivos x e y tais que n = x2 - y2 . Supondo que encontramos estes nmeros, temos que n = x2 - y2 = (x - y) (x + y).

105

CAPTULO 7 NMEROS PRIMOS

Logo x - y e x + y so fatores de n. O caso mais fcil do algoritmo de Fermat ocorre quando n um quadrado perfeito; isto , quando existe algum inteiro r tal que n = r2. Neste caso temos que r fator de n. Alm disso, na notao acima x = r e y = 0. Observe que se y > 0 ento

Isto sugere a seguinte estratgia para encontrar x e y. Entrada: inteiro positivo mpar n. Sada: um fator de n ou uma mensagem indicando que n primo. Etapa 1: Comece com ; se n = x2 ento x fator de n e podemos parar. . :

Etapa 2. Caso contrrio incremente x de uma unidade e calcule

Etapa 3. Repita a Etapa 2 at encontrar um valor inteiro para y, ou at que x seja igual a no primeiro caso n tem fatores x+y e x-y, no segundo n primo.

Exemplo: Seja n = 1342127 o nmero obtido como produto de dois primos. A varivel x inicializada com a menor parte inteira da raiz quadrada de n . Mas x2 = 11582 = 1340964 < 1342127 logo passamos a incrementar x de um em um. Fazemos isso at que seja inteiro, ou x seja igual a , que neste caso valeria 671064. mais fcil resumir isto em uma tabela x 1159 1160 1161 1162 1163 1164 33,97 58,93 76,11 90,09 102,18 113

Obtivemos assim um inteiro no sexto lao. Portanto x = 1164 e y = 113 so os valores desejados. Os fatores correspondentes so x + y = 1277 e x y = 1051. Logo, 1051 e 1277 so os dois nmeros primos procurados.

106

CAPTULO 7 NMEROS PRIMOS

EXERCCIOS
1. Com uma calculadora, achar todos os primos da forma n 2, para 25 n 35
2

15. Achar o mdc(a, b) e mmc(a, b) sabendo a = 230 . 521 . 19 . 233 e b = 26 . 3 . 74 . 112 . 195 . 237 16. Achar o menor inteiro positivo pelo qual se deve dividir 15! para se obter um quadrado. Qual o menor valor do nmero natural n que torna n! divisvel por 1000? 17. Achar todos os primos que so divisores de 50!. 18. Verifique com uma calculadora, se so primos gmeos: a) 1949 e 1951 b) 1997 e 1999 19. Achar uma sequncia de quatro inteiros positivos consecutivos e compostos. 20. Achar um sequncia de 100 inteiros positivos consecutivos e compostos. 21. Mostre que nenhum nmero inteiro da forma

2.

Determine todos os primos que so iguais a diferena de quadrado entre dois primos. De quantos modos podem escrever 497 como a soma de dois nmeros primos? Mostrar que a soma de dois inteiros positivos mpares e consecutivos nunca um primo. Em um quadro esto escritos alguns nmeros naturais. Dentre eles, h nove mltiplos de 4, sete mltiplos de 6, cinco mltiplos de 12, trs nmeros primos e nada mais. Qual a quantidade mnima de nmeros escritos? Achar todos os primos p e q, tais que p q = 3. Achar todos os primos que so iguais a um quadrado menos 1. Achar todos os primos que so iguais a um cubo menos 1. Escreva os nmeros 55, 83 e 211 como uma soma de trs primos.

3.

4.

5.

6.

7.

8.

9.

1 4n divisvel pelo nmero primo 3.


22. Com uma calculadora, verificar a conjectura de Goldbach para n par, 42

10. Determinar todos os inteiros positivos n tais que n, n + 2 e n + 4 so todos primos. 11. Determinar todos os primos p tais que 3p + 1 um quadrado. 12. Com uma calculadora, determinar se so primos os nmeros a) b) c) d) 1699 7429 21793 1189

n 100 .

23. Determinar o menor valor positivo do inteiro n tal que 2n2 + p, seja um nmero inteiro composto e p um primo terminado em 7. 24. Demonstrar que todo primo, p 5 da forma 6k 1 ou 6k + 1, onde k um inteiro positivo. 25. Demonstrar que todo primo p 3, da forma 4k + 1 ou 4k 1, onde k um inteiro positivo. 26. Determine todos os primos p 3003 tambm seja primo. 5tais que 8p4 -

13. Encontre todos os primos p, tais que 17p + 1 um quadrado. 14. Usando a decomposio em fatores primos dos inteiros 507 e 1287, achar o mdc (507, 1287) e o mmc (507, 1287).

27. Mostrar que todo inteiro da forma n4 + 4, com n > 1 no primo. 28. Mostrar que todo inteiro da forma 8 n + 1, com n > 1, no primo. 29. Mostrar que se n2 + 2 primo ento 3 | n, para todo n > 1.

107

CAPTULO 7 NMEROS PRIMOS

30. Se p > 5 um primo, ento p2 + 2 composto. 31. Demonstrar as seguintes propriedades: a) Todo primo da forma 3n + 1 tambm da forma 6m + 1. b) Todo inteiro n > 11 pode ser expresso como a soma de dois inteiros no-primos. c) Se p 5 um primo mpar, ento p2 1 ou p2 + 1 divisvel por 10. d) Se p > q > 5 e se p e q so ambos primos, ento 24 | p2 q2. e) Todo inteiro da forma 3n + 2 tem um fator primo desta forma. f) Se p um primo e se p | an , ento pn | an. 32. Demonstrar que o inteiro positivo a > 1 um quadrado se e somente se todos os expoentes dos fatores primos da sua decomposio so inteiros pares. 33. Demonstrar que, se o inteiro k 2, no primo, ento 2k 1 nunca ser primo. 34. Demonstrar que se 2k 1, (k ento k tambm primo. 2) primo,

45. Mostre que se n 1 natural ento, o nmero

22

2n 1

no primo.

46. Sendo n > 1 um inteiro, prove que 4n + n4 no primo. 47. Mostrar, mediante um exemplo, que a seguinte conjectura falsa: Todo inteiro positivo maior que 1, pode-se escrever sob a forma a2 + p, com a > 0 e p um inteiro primo ou 1. 48. Determine todos os nmeros primos p e q, para os quais os q nmeros p, p + (q + 1), p + 2 (q + 1), p + 3 (q + 1), . . . , p + (q 1) (q + 1), tambm so primos. 49. Demonstrar que existem infinitos primos da forma 4n + 3, com n inteiro positivo. 50. Seja m um intero positivo. Demonstre que no existem nmeros primos da forma 25m + 2m +1. 51. Determinar o nmero inteiro positivo n que que produto dos primos p, q e r, sabendo que r - q = 2p e rq + p2 = 676. 52. Mostre que existem infinitos valores primos p para os quais 8.p2 + 5 divisvel por 77. 53. Seja p > 2 um primo. Determine todos os valores inteiros positivos de m e n, tal que (p 1) (pn + 1) = 4m (m + 1).

35. Seja p o maior fator primo do nmero 314 + 313 12, ento p igual a: 36. Sejam p, q inteiros positivos. Mostre que 2p + 1 = q2 implica p e q primos e p = q = 3. 37. Mostrar que um inteiro da forma 42n+1 + 1, onde n 1, nunca primo. 38. Sendo n um inteiro positivo, mostre que 24(n+1) 1 nunca ser primo. 39. Mostrar que se n > 4, no primo, ento n divide (n 1)!. 40. Verificar que todo inteiro pode escrever-se sob a forma 2k m, onde o inteiro k > 0 e m um inteiro mpar. 41. Demonstrar que, se o inteiro n > 2, ento existe um primo p tal que n < p < n!. 42. Qual o menor nmero primo que um fator da soma 19992002 + 20012002? 43. Prove que um tringulo retngulo no pode apresentar as medidas de seus lados sendo nmeros primos. 44. Se p e 8p + 1 so nmeros primos, prove que p = 3.
2

Nos problemas que se seguem faa uso de uma calculadora para verificar os resultados e explicite bem os passos utilizados na resoluo.

54. Segundo o Teorema de Chebychev, para um inteiro m 2, existe um primo p tal que m < p < 2 m. Determine todos os primos entre 600 e 1200. 55. Segundo o Teorema de Dirichlet, se o mdc (a, b) =1, ento existem infinitos primos da forma an+b com n um inteiro positivo. Determine todos os primos p da forma 4n+9, com 88 < 4n + 9 < 388. 56. Usando o Teorema de Sierpinski, determine um primo p>19 e escreva 20 inteiros compostos. 57. Usando a Frmula de Minc, determine

(12) .

108

CAPTULO 7 NMEROS PRIMOS 58. Usando a Frmula do n-simo nmero primo, determine o quarto nmero primo. 59. Calcule: a)

3#.5# 11# 5#.7 # 13#

7# 11#

b)

60. Verifique se existem primos gmeos entre 600 e 700. 61. Determine dois nmeros primos consecutivos tais que a diferena entre eles seja maior que 7. 62. Decomponha 98! Em fatores primos. 63. Determine a potncia de 5 na decomposio de 75! em fatores primos, fazendo a decomposio p-dica de 75. 64. Com quantos zeros termina o nmero 1000! ? Qual a potncia de 3 que aparece na decomposio de 1000! em fatores primos? 65. Justifique se o nmero

93.94. ... .112.113 21!

inteiro. Em caso afirmativo, calcule o seu valor. 66. Encontrar o maior valor do inteiro n que

0 tal

10200! seja inteiro. 504 n

67. Utilizando o Teorema do Nmero Primo: a) Faa uma estimativa (sem muito rigor) de quantos primos de 200 dgitos existem. b) Mostre que entre os nmeros de k-dgitos, um em cada 2, 3k primo. 68. Qual o menor valor do nmero natural n que torna n! divisvel por 1000?

109

Captulo 8:

EQUAES DIOFANTINAS LINEARES


UM POUCO DE HISTRIA SOBRE DIOFANTO Diofanto tem o seu nome ligado cidade que foi o maior centro de atividade matemtica na Grcia antiga. Pouco se sabe acerca da sua vida, o desconhecimento impede-nos mesmo de fixar com segurana em que sculo viveu. Tm sido sugeridas datas distanciadas de um sculo, antes ou depois do ano 250 d. C. Por uns versos encontrados no seu tmulo, escritos em forma de um enigmtico problema, deduz-se que viveu 84 anos. Positivamente, tal problema no deve ser tomado como o paradigma dos problemas sobre os quais se interessou Diofanto, pois ele pouca ateno deu a equaes do 1 grau. Alexandria foi sempre um centro muito cosmopolita e a matemtica que se originou nela no era toda do mesmo tipo. Os resultados de Heron eram bem diferentes dos de Euclides ou dos de Apolonios ou dos de Arquimedes, e na obra de Diofanto h novamente uma quebra abrupta da tradio clssica grega. Sabido que os gregos, na poca clssica, dividiram a aritmtica em dois ramos: a aritmtica propriamente dita como "teoria dos nmeros naturais". Frequentemente, tinha mais em comum com a filosofia platnica e pitagrica do que com o que habitualmente se considera como matemtica, e logstica ou clculo prtico que estabelecida as regras prticas de clculo que eram teis Astronomia, Mecnica, etc. O principal tratado de Diofanto conhecido, e que. ao que parece, s em parte chegou at ns, a "Aritmtica". Apenas seis dos livros originais em grego sobreviveram, o nmero total (13) no passa de uma conjectura. Era um tratado caracterizado por um alto grau de habilidade matemtica e de engenho, pelo que pode ser comparado aos grandes clssicos da "Primeira idade Alexandrina", ou seja, da "poca de ouro" da matemtica grega, no entanto, quase nada tm em comum com esses ou, na verdade, com qualquer matemtica grega tradicional. Representa essencialmente um novo ramo e usa um mtodo diferente, dai a poca em que possivelmente Diofanto viveu se chamar "segunda idade Alexandrina", conhecida por sua vez por "poca de prata" da matemtica grega. Diofanto, mais que um cultor da aritmtica, e, sobretudo da geometria, como o foram os matemticos gregos anteriores, deve considerar-se um precursor da lgebra, e, em certo sentido, mais vinculado com a matemtica dos povos orientais (Babilnia, ndia,...) que com a dos gregos. A sua "Aritmtica assemelha-se lgebra babilnica em muitos aspectos, mas enquanto os matemticos babilnicos se ocupavam principalmente com solues aproximadas" de equaes "determinadas" e, sobretudo de equaes "indeterminadas" do 2 e do 3 graus das formas cannicas, em notao atual, Ax2+Bx+C = y2 e Ax3+Bx2+Cx+D=y2, ou conjuntos (sistemas) destas equaes. exatamente, por esta razo em homenagem a Diofanto -que a esta "Anlise indeterminada" se chama Anlise diofantina ou Anlise diofntica". No desenvolvimento histrico da lgebra considera-se, em geral, que podem ser reconhecidos trs estdios: o primitivo ou retrico, em que tudo era completamente escrito em palavras, um intermdio ou sincopado, em que foram adaptadas algumas abreviaturas e convenes, e um final ou simblico, em que so usados somente smbolos. A "Aritmtica" de Diofanto deve ser colocada no segundo estdio; nos seus seis livros h um uso sistemtico de abreviaturas para potncias de nmeros e para relaes e operaes.

110

CAPTULO 8 EQUAES DIOFANTINAS LINEARES

3.1. Generalidades
A teoria das Equaes Diofantinas o ramo da Teoria dos Nmeros que investiga as solues inteiras de equaes polinomiais, como por exemplo: x2 + y2 = z2, possui infinitas solues representadas pelas ternas ordenadas (x,y,z) conhecidas como ternos pitagricos. xn + yn = zn, que no possui solues no nulas para para n > 2, e conhecida como o ltimo Teorema de Fermat. y2 = x3 + 17, que vlida, por exemplo, para os seguintes valores positivos: (4,9); (8,23); (43, 282); (52, 375); ... Equao de Pell : x 2 dy 2 m , onde quadrado e m um inteiro qualquer. Etc...

(2,5) ;

d um inteiro positivo que no seja um

O tipo mais simples de equao diofantina a equao diofantina linear com duas incgnitas x e y: ax + by = c onde a, b e c so inteiros dados, sendo ab 0 . Todo par de inteiros x 0 , y 0 tais que a x 0 + b y 0 = c diz-se uma soluo inteira ou apenas uma soluo da equao ax + by = c. Consideremos, por exemplo, a equao diofantina linear com duas incgnitas: 3x + 6y = 18 Temos: 3.4 + 6.1 = 18 3(-6) + 6.6 = 18 3.10 + 6(-2) = 18 Logo, os pares de inteiros: 4 e 1, -6 e 6, 10 e -2 so solues da equao 3x + 6y = 18 Existem equaes diofantinas lineares com duas incgnitas que no tm soluo. Assim, por exemplo, a equao diofantina linear: 2x + 4y = 7

111

CAPTULO 8 EQUAES DIOFANTINAS LINEARES

no tem soluo, porque 2x + 4y um inteiro par quaisquer que sejam os valores inteiros de x e y, enquanto que 7 um inteiro mpar (observe-se que 2 = mdc (2, 4) no divide 7). De modo geral, a equao diofantina linear ax + by = c no tem soluo todas as vezes que d = mdc (a, b) no divide c, como bvio.

3.2. Condio de Existncia de Soluo


Teorema 3.1: A equao diofantina linear ax + by = c tem soluo se e somente se d divide c, sendo d = mdc (a, b). Demonstrao: Suponhamos que a equao ax + by = c tem uma soluo, isto , que existe um par de inteiros x 0 , y 0 tais que a x 0 + b y 0 = c. Por ser o mdc (a, b) = d, existem inteiros r e s tais que a = dr e b = ds, e temos: c = a x 0 + b y 0 = dr x 0 + ds y 0 = d(r x 0 + s y 0 ) e como r x 0 + s y 0 um inteiro, segue-se que d divide c ( d | c ). Reciprocamente, suponhamos que d divide c ( d | c ), isto , que c = dt, onde t um inteiro. Por ser o mdc (a, b) = d, existem inteiros x 0 e y 0 tais que d = a x 0 + b y0 o que implica: c = dt = (a x 0 + b y 0 )t = a(t x 0 ) + b(t y 0 ) isto , o par de inteiros: x = t x 0 = (c/d) x 0 , y = t y 0 = (c/d) y 0 uma soluo da equao ax + by = c.

112

CAPTULO 8 EQUAES DIOFANTINAS LINEARES

3.3. Solues da equao ax + by = c.


Teorema 3.2: Se d divide c ( d | c ), sendo d = mdc (a, b), e se o par de inteiros x 0 , y 0 uma soluo particular da equao diofantina linear ax + by = c, ento todas as outras solues desta equao so dadas pelas frmulas:
x x0 b t, y d y0 a t d

onde t um inteiro arbitrrio Demonstrao: Suponhamos que o par de inteiros x 0 , y 0 uma soluo particular da equao ax + by = c, e seja x1 , y1 uma outra soluo qualquer desta equao. Ento, temos: a x 0 + b y 0 = c = a x 1 + b y1 e, portanto: a( x1 - x 0 ) = b( y1 - y 0 ) Por ser o mdc (a, b) = d, existem inteiros r e s tais que a = dr e b = ds, com r e s primos entre si. Substituindo estes valores de a e b na igualdade anterior e cancelando o fator com d, obtemos: r( x1 - x 0 ) = s( y 0 - y1 ) Assim sendo, r | s(y0

y1 ) , e como o mdc (r, s) = 1, segue-se que r | (y0 y0 y1 = rt e x1 x 0 = st

y1 ) , isto :

onde t um inteiro. Portanto, temos as frmulas:

x1 x0 st x0 (b / d)t y1 y0 rt y0 (a / d)t
Estes valores de x1 e y1 satisfazem realmente a equao ax + by = c, qualquer que seja o inteiro t, pois, temos: a x1 + b y1 = a[ x 0

(b / d)t ] + b[ y0 (a / d)t ] = a x 0 + b y 0 + (ab/d ab/d)t = c + 0.t = c

113

CAPTULO 8 EQUAES DIOFANTINAS LINEARES

Como se v, se d = mdc (a, b) divide c ( d | c ), ento a equao diofantina linear ax + by = c admite um nmero infinito de solues, uma para cada valor do inteiro arbitrrio t. Corolrio 3.1: Se o mdc (a, b) = 1 e se x 0 , y 0 uma soluo particular da equao diofantina linear ax + by = c, ento todas as outras solues desta equao so dadas pelas frmulas:

x
onde t um inteiro arbitrrio.

x0 bt , y

y0 at

Nota: Uma soluo particular da equao diofantina linear se obtm por tentativas ou pelo algoritmo de Euclides. E em ambos os casos a soluo geral se pode obter usando o teorema 3.2, conforme se vai ver nos exemplos a seguir. Exemplo 3.1: Determinar todas as solues inteiras e positivas da equao diofantina linear 18x + 5y = 48 Determinemos o mdc (18, 5) pelo algoritmo de Euclides: 18 = 5.3 + 3 5 = 3.1 + 2 3 = 2.1 + 1 2 = 1.2 3 18 5 3 1 3 2 1 2 1 2 1 0

Portanto, o mdc (18, 5) = 1 e a equao dada tem soluo. e para exprimir 1 como combinao linear de 18 e 5 basta eliminar os restos 2 e 3 entre as trs primeiras igualdades anteriores do seguinte modo: 1 = 3 2 = 3 (5 3) = 2.3 5 = 2(18 5.3) 5 = 18.2 + 5(-7) isto : 1 = 18.2 + 5(-7) e 48 = 18.96 + 5(-336) Logo, o par de inteiros x 0 = 96, y 0 = -335 uma soluo particular da equao proposta, e todas as demais solues so dadas pelas frmulas: x = 96 + 5t, y = -336 18t

114

CAPTULO 8 EQUAES DIOFANTINAS LINEARES

onde t um inteiro arbitrrio. As solues inteiras e positivas se acham escolhendo t de modo que sejam satisfeitas as desigualdade: 96 + 5t > 0, -336 18t > 0 Isto : t > 19, 2 e t < 18,6

o que implica t = -19 e, portanto: x = 96 + 5(-19) = 1, y = -336 -18(-19) = 6 Assim, o par de inteiros x = 1, y = 6 a nica soluo inteira e positiva da equao 18x + 5y = 48. Exemplo 3.2: Resolver a equao diofantina linear 39x + 26y = 105 O mdc (a, b) = 13 e como 13 no divide 105, segue-se que a equao dada no tem soluo.

EXERCCIOS
1) Determinar todas as solues inteiras das seguintes equaes diofantinas lineares: a) b) c) d) e) f) g) h) i) j) k) l) 56x + 72y = 40 24x + 138y = 18 221x + 91y = 117 84x 438y = 156 48x + 7y = 5 57x 99y = 77 11x + 30y = 31 27x 18y = 54 13x 7y = 21 44x + 66y = 11 21x 12y = 72 17x + 54y = 8 2) Determinar todas as solues inteiras e positivas das seguintes equaes diofantinas lineares: a) b) c) d) e) f) g) h) 5x 11y = 29. 32x + 55y = 771 58x 87y = 290 62x + 11y = 788 30x + 17y = 300 54x + 21y = 906 123x + 360y = 99 158x 57y = 7

3)

Determinar o menor inteiro positivo que dividido por 8 e por 15 deixa os restos 6 e 13, respectivamente.

115

CAPTULO 8 EQUAES DIOFANTINAS LINEARES

4)

Exprimir 100 como soma de dois inteiros positivos de modo que o primeiro seja divisvel por 7 e o segundo seja divisvel por 11. Determinar as duas menores fraes positivas que tenham 13 e 17 para denominadores e cuja soma seja igual a 305 . 221 Determine todas as solues inteiras do sistema de equaes

5)

13) O laboratrio Sangue Bom, dispe de 2 mquinas para examinar amostras de sangue. Uma delas examina 15 amostras de cada vez, enquanto a outra examina 25. Quantas vezes essas mquinas devem ser acionadas para examinar exatamente 2 mil amostras? 14) Num determinado lugar a moeda o mirrl. Suponhamos que s existam moedas de 15 e 7 mirris e que se queira pagar uma determinada quantia em mirris. Ser que sempre possvel? E se existirem moedas de 12 e 30 mirris? 15) Para agrupar 13 avies em filas de 3 ou de 5, exatamente quantas filas sero formadas de cada tipo? 16) Para participar de um evento comemorativo em um clube, no scios pagavam R$ 12,00 e scios R$ 8,00. Sabendo-se que foram arrecadados R$ 908,00 na portaria, quantos scios estiveram no evento? 17) Um galo custa 5 mirris, uma galinha 3 mirris, e trs pintinhos 1 mirrl. Com 100 mirris um fazendeiro comprou 100 dessas aves. Quantos galos, galinhas e pintinhos foram comprados? 18) Demonstrar que se a e b so inteiros positivos primos entre si, ento a equao diofantina ax by = c tm um nmero infinito de solues inteiras e positivas.

6)

2 x 3 y 5z 201 3x 5 y 7 z 315

7)

Encontre todas as solues da equao Diofantina (6x+15y)(8x+7y) = 129. Uma pessoa foi ao banco para descontar um cheque no valor de x reais e y centavos. O caixa do banco errou na leitura do valor do cheque e pagou y reais e x centavos. A pessoa guardou o dinheiro no bolso sem verificar a quantia. No caminho de casa, ela gastou cinco centavos e quando chegou em casa verificou que tinha exatamente o dobro do valor do cheque. Sabendo-se que essa pessoa no levou dinheiro nenhum consigo quando foi ao banco, pergunta-se qual era o valor do cheque. Um grupo de pessoas gastou 1000 dlares num hotel. Sabendo-se que apenas alguns dos homens estavam acompanhados pelas esposas e que cada homem gastou 19 dlares e cada mulher gastou 13 dlares, pede-se determinar quantas mulheres e quantos homens estavam no hotel.

8)

9)

10) Um grupo de pessoas gastou 690 dlares num hotel. Sabendo-se que apenas alguns dos homens estavam acompanhados pelas esposas e que cada homem gastou 18 dlares e cada mulher gastou 15 dlares, pede-se determinar quantas mulheres e quantos homens estavam no hotel. 11) Ao entrar num bosque, alguns viajantes avistam 37 montes de mas. Aps serem retiradas 17 frutas, o restante foi dividido igualmente entre 79 pessoas. Qual pode ter sido a parte de cada pessoa? 12) Sabendo que um time de basquete composto de 5 jogadores e um time de vlei formado por 6 jogadores. Quantas quadras de basquete e quantas de vlei so necessrias para que 80 alunos joguem simultaneamente qualquer um dos esportes? E se forem 77 alunos?

116

Captulo 9

CONGRUNCIAS
9.1. Congruncias
Definio 9.1 Sejam a e b inteiros quaisquer e seja m >1 um inteiro positivo fixo. Diz-se que a congruente a b mdulo m se, e somente se, m divide a diferena a b. Em outros termos a congruente a b mdulo m se, e somente se, existe um inteiro k tal que a b = km Simbolicamente: a Exemplos 9.1 3 b (mod m) m|(ab) a - b = km a = km + b 63 (mod 8)

24 (mod 7) ; 31

11 (mod 6) ; 15

Definio 9.2 Se m no divide a diferena a b, ento diz-se que a incongruente a b mdulo m . Notao: a b (mod m)

Observaes: 1) Dois inteiros quaisquer so congruentes mdulo 1 2) Dois inteiros so congruentes mdulo 2, se ambos so pares ou ambos so mpares 3) a 0 (mod m) se, e somente se, m | a.

9.2. Caracterizao de Inteiros Congruentes


Teorema 9.1 Dois inteiros a e b so congruentes mdulo m se, e somente se, a e b deixam o mesmo resto quando divididos por m. Demonstrao: ( ) Suponhamos que a b ( mod m). Ento, pela definio: a b = km, k Z

117

CAPTULO 9 CONGRUNCIAS

Seja r o resto da diviso de b por m; ento pelo algoritmo da diviso: b = mq + r, 0 Portanto: a = km + b = km + mq + r = (k + q)m + r e isto significa que r tambm o resto da diviso de a por m, isto , os inteiros a e b divididos por m deixam o mesmo resto r. ( ) Reciprocamente, suponhamos que a e b divididos por m deixam o mesmo resto r. Ento, podemos escrever: a = mq1 + r e b = mq2 + r , 0 e, portanto: a b = ( q1 q2) m m (a b) a b ( mod m)

r<m

r<m

9.3. Propriedades das Congruncias


Teorema 9.2 Seja m um inteiro positivo fixo (m > 1) e sejam a, b e c inteiros quaisquer. Valem as propriedades: 1) a a (mod m) (Reflexiva) 2) Se a b (mod m), ento b a (mod m) (Simtrica) 3) Se a b (mod m) e se b c(mod m), ento a c (mod m) (Transitiva) Demonstrao: (1) (2) a 0 ou seja, a (a a), o que implica: a a (mod m) Se a b (mod m), ento a b = km, k z.

Portanto: b a = -(km) = (-k)m (3) Se a b (mod m) e se b b a (mod m)

c (mod m), ento existem inteiros h e k tais que a b = hm e b c = km

Portanto: a c = (a - b) + (b - c) = hm + km = (h + k)m

118

CAPTULO 9 CONGRUNCIAS

e isto significa que a

c (mod m).

Nota: Consoante este teorema, as relao binria R no conjunto Z dos inteiros definidas por aRb a b (mod m) reflexiva, simtrica e transitiva, ou seja, R uma relao de equivalncias em Z. Esta relao de equivalncia R em z denominada congruncia mdulo m .

Teorema 9.3 Seja m um inteiro positivo fixo (m > 1) e sejam a, b dois inteiros quaisquer. Valem as seguintes propriedades: 1) Se a b (mod m) e se n | m, com n > 0, ento a b (mod n)

Demonstrao: Com efeito: a b (mod m) a b =km e n m m = nq

onde k e q >0 so inteiros. Portanto: a b = (kq)n a b (mod n)

2) Se a

b (mod m) e se c > 0 , ento ac

bc (mod mc)

Demonstrao: Com efeito, se a a b = km

b (mod m), ento: ac bc = k(mc) ac bc (mod mc)

3) Se a b (mod m) e se a, b, m so todos divisveis pelo inteiro d > 1, ento a b m (mod ) d d d Demonstrao: Com efeito, se a b (mod m), ento:
a b m = k( ) d d d a d b m (mod ) d d

a - b = km

119

CAPTULO 9 CONGRUNCIAS

Teorema 9.4 Seja m um inteiro positivo fixo (m > 1) e sejam a, b, c, d inteiros quaisquer. Valem as seguintes propriedades: 1) Se a b (mod m) e se c (mod m). Demonstrao: Se a b (mod m) e se c d (mod m), ento existem inteiros h e k tais que a b = hm e c d = km.Portanto: (a + c) (b + d) = (a - b)+ (c- d) = (h + k)m e ac bd = (b + hm) (d + km) - bd = (bk + dh + hkm)m o que implica: a+c b + d (mod m) e ac bd (mod m)

d (mod m), ento a + c

b + d (mod m)

e ac

bd

2) Se a b (mod m) e c um inteiro qualquer, ento a + c (mod m). Demonstrao: Temos: a Logo, pela propriedade anterior: a+c Em particular, se c = -1, ento: a (-1) b(-1) (mod m) b + c (mod m) e ac b (mod m) e c c (mod m)

b + c (mod m)

e ac

bc

bc (mod m)

ou -a -b (mod m)

3)

Se a

b (mod m), ento an

bn (mod m) para todo inteiro positivo n.

Demonstrao: Usando o Teorema da induo Matemtica, a proposio verdadeira para suposta verdadeira para o inteiro positivo k temos: ak b k (mod m) e a b (mod m) n = 1, e

Portanto, pela propriedade 1 acima

120

CAPTULO 9 CONGRUNCIAS

a k .a

b k .b (mod m) ou ak+1

bk+1 (mod m)

isto , as proposio verdadeira para o inteiro positivo k + 1. logo, a preposio verdadeira para todo inteiro positivo n. Teorema 9.5: Se ac bc (mod m) e se o mdc(c,m) = d, ento a bc (mod m), ento: Z. b (mod
m ) d

Demonstrao: Com efeito, se ac

ac bc = (a b)c = km , com k

Como o mdc (c,m) = d, existem inteiro r e s tais que c = dr e m = ds, onde r e s so primos entre si. Portanto: (a b) dr = kds ou (a - b)r = ks

o que implica que s (a b)r, com o mdc (r,s) = 1. Logo, pelo Teorema 5.4 ( de Euclides): s m m (a b) e a b (mod s) ou, por ser s = , a b (mod ). d d Corolrio 9.1 Se ac bc ( mod m) e se o mdc (c,m) = 1, ento a b (mod m). Esta propriedade mostra que permitido cancelar fatores de ambos os membros de uma congruncia que so primos com o mdulo. Corolrio 9.2 Se ac a b (mod p) p no divide c e p primo, implicam que o bc (mod p), com p primo, e se p no divide c, ento

Demonstrao: Com efeito, as condies: mdc(c, p) = 1.

9.4. Sistemas Completos de Restos


Definio 9.3 Chama-se sistema completo de restos mdulo m todo conjunto S = {r1 , r2 , ... , rm} de m inteiros tal que um inteiro qualquer a congruente mdulo m a um nico elemento de S. Exemplo 9.2: Cada um dos conjuntos: {1, 2, 3} , {0, 1, 2} , { 1, 0, 1} , {1, 5, 9} um sistema completo de restos mdulo 3.

121

CAPTULO 9 CONGRUNCIAS

Teorema 9.6 O conjunto S = {0,1, 2, ..., m 1} um sistema completo de restos mdulo m. Demonstrao: Com efeito, o conjunto S tem m elementos e, alm disso, qualquer que seja o inteiro a temos, pelo algoritmo da diviso: a = mq + r, com 0 r m o que implica a r (mod. m), e como o resto r s pode assumir os m valores 0, 1, 2, ..., m-1, segue-se que o inteiro a congruente mdulo m a um nico desses m inteiros. Corolrio 9.3 Se S = {r1 , r2 , ... , rm} um sistema completo de restos mdulo m, ento os elementos de S so congruentes mdulo m aos inteiros 0, 1, 2, ... , m 1, tomados numa certa ordem. Demonstrao: Com efeito, se a um inteiro qualquer, ento: a r1 (mod. m), com r1 S a k (mod. m), com 0 k m-1 o que implica: r1 k (mod. m).

9.5 Aritmtica Mdulo m


Definio 9.4.: Seja a um inteiro. Chama-se classe de congruncia de a mdulo m (m > 1) o conjunto formado por todos os inteiros que so congruentes a a mdulo m. Denotamos esse conjunto por a . Temos, ento:
a ={x

Z;x

a (mod m) }

Como x a (mod m), se e somente se, x da forma x = a + k.m, para algum k Z, tambm podemos escrever:
a = { a + k.m | k

Z}

Mostraremos a seguir que a relao de congruncia entre nmeros se traduz em igualdade no sentido estrito entre classes. Proposio 9.1: Sejam a e b inteiros. Ento a b (mod m), se e somente se, a b . Demonstrao: Suponhamos que a b (mod m), queremos provar que a = b , isto , uma igualdade entre conjuntos. Dado x a (mod m). Da a , temos por definio que x propriedade transitiva de congruncia e da hiptese, segue imediatamente que x b . Logo, a b . A incluso b a em sentido contrrio segue de forma anloga.

122

CAPTULO 9 CONGRUNCIAS

Reciprocamente, se a = b , como a

a , temos tambm que a b , ento a b =

b , logo, a

b (mod m).

Corolrio 9.4: Sejam a e b inteiros. Se a

Demonstrao: Se a b = , consideremos um inteiro c que pertena a ambas as classes. Como c a , temos que c a (mod m) e, de forma anloga, c b (mod m). Portanto, a b (mod m) e, da proposio acima, a = b . Note que, por exemplo, para as classes mdulo 7, temos que 0 7 14 7 ... ou 4 11 3 ... etc. Mais precisamente, dada uma classe a , para qualquer inteiro x tal que x a , temos que x = a . Por causa disto, cada inteiro pertencente a uma dada classe diz-se um representante daquela classe. Por exemplo, 11 e 3 so representantes da classe 4 mdulo 7. Consideremos um sistema completo de classes ou resduos mdulo m, por exemplo, os inteiros 0, 1, ..., m 1 e suas respectivas classes:
0 = { 0,

m,

2.m,

3.m, ... } 3.m, ... }

1 = { 1, 1

m, 1

2.m, 1

...
m 1 = { m 1, m 1

m, m 1

2.m, m 1

3.m, ... }

Conforme j foi considerado, cada inteiro pertence a uma e apenas uma das m classes. Por exemplo, se m = 7, todas as classes possveis, mdulo 7, so as seguintes:
0 = { 0,

7,

14,

21, ... } 14, 1 14, 2 14, 3 14, 4 14, 5 14, 6 21, ... } 21, ... } 21, ... } 21, ... } 21, ... } 21, ... }

1 = { 1, 1 7, 1 2 = { 2, 2 7, 2
3 = { 3, 3 7, 3

4 = { 4, 4 7, 4
5 = { 5, 5 7, 5 6 = { 6, 6 7, 6

Denotaremos pelo smbolo m o conjunto das classes de congruncias mdulo m e o chamaremos de Conjunto dos Inteiros Mdulo m. Assim, Z7 = { 0 , 1 , 2 , 3 , 4 , 5 , 6 }.

123

CAPTULO 9 CONGRUNCIAS

Note que, por exemplo,


0 = 7 , 1 = 15 , 2 = 9 , 3 = 6=

11, 4 = 25 , 5 =

16 ,

8 e tambm podemos escrever:

Z7 = { 7 , 15 , 9 ,

11, 25 ,

16 ,

8 }.

Em geral, se { a1, a2, ..., am } um sistema completo de restos mdulo m, temos que:

m = { a1 , a 2 , ..., a m }
Tomando o sistema de restos mais simples, podemos escrever:

m = { 0 , 1 , 2 , ..., m 1 }

Note que, conforme as observaes acima, o conjunto m tem precisamente m elementos.

9.6. Adio e Multiplicao em m


Agora gostaramos de introduzir operaes de soma e produto em m e estudar suas propriedades. Existe uma forma natural de faz-lo. Por exemplo, para somar e multiplicar 3 e 6 em Z7, faramos:
3+6=9=2 3 . 6 = 18 = 4

124

CAPTULO 9 CONGRUNCIAS

Exemplificando a adio e a multiplicao em uma tabuada mdulo 7: + 0 1 2 3 4 5 6 0 0 1 2 3 4 5 6 1 1 2 3 4 5 6 0 2 2 3 4 5 6 0 1 3 3 4 5 6 0 1 2 4 4 5 6 0 1 2 3 5 5 6 0 1 2 3 4 6 6 0 1 2 3 4 5 * 0 1 2 3 4 5 6 0 0 0 0 0 0 0 0 1 0 1 2 3 4 5 6 2 0 2 4 6 1 3 5 3 0 3 6 2 5 1 4 4 0 4 1 5 2 6 3 5 0 5 3 1 6 4 2 6 0 6 5 4 3 2 1

Observaes: Observe, na tabela de adio, o conceito de inverso aditivo mdulo m. Dizemos que dois elementos de m so inversos aditivos, se e somente se, a b 0(mod m) . Assim, por exemplo, 4 e 3 so inversos aditivos mdulo 7, uma vez que 4 3 0(mod 7) .

Mais explicitamente, definimos soma e produto em Zm por:


3+6=9=2 3 . 6 = 18 = 4

Quer dizer, para efetuar a soma de duas classes mdulo m, tomamos representantes (quaisquer) a e b dessas classes, efetuamos a soma a + b em Z e consideramos como resultado da soma a classe de a + b mdulo m . A operao de produto se faz de forma anloga. Surge agora uma pergunta natural: ser que o resultado das operaes no depende dos representantes escolhidos? Voltando ao exemplo de Z7 , para somar 3 + 6 , poderamos tomar 38 como um representante de 3 e 27 como representante de 6 . Ser que 38 + 27 = 65 o mesmo resultado que aquele obtido acima, 3 + 6 = 2 ? A resposta afirmativa. Como 65 2 (mod 7), felizmente o resultado o mesmo. O lema abaixo mostra que isso no uma mera coincidncia. Lema 9.1: Sejam a, a, b e b inteiros tais que a = a' e b = b' . Ento,
a b = a' b' e a . b = a' . b' .

Demonstrao: uma conseqncia imediata das propriedades. Proposio 9.2: Em m valem as seguintes propriedades:

125

CAPTULO 9 CONGRUNCIAS

(P1) Propriedade Associativa: Para toda terna a , b , c de inteiros mdulo m, tem-se que: a + (
b + c )=( a + b )+ c

(P2) Existncia do Elemento Neutro: Existe um nico elemento em m que precisamente 0 a classe do elemento 0, tal que: a + 0 = a , para todo a m (P3) Existncia do Elemento Oposto: Para cada inteiro mdulo m, a , existe um nico elemento em Zm que chamaremos oposto de a e indicaremos por a , tal que: a + ( a ) = 0 (P4) Propriedade Comutativa: Para todo par a , b de elementos de Zm tem-se que: a + b = b + a. Demonstrao: As demonstraes so feitas apoiando-se nos axiomas para as operaes com nmeros inteiros. A ttulo de ilustrao, provaremos P1 e P3. (P1)Utilizando repetidamente a definio de soma em m , temos:
a +( b + c )= a +( b
c )= a

(b

c )

Agora, como vale a associativa da soma entre nmeros inteiros, ou seja: a+(b+c)=(a+b)+c temos que: a ( b c ) = ( a b ) c logo, a + ( b + c ) = a ( b c ) = ( a b ) c = ( a + b ) + c Na ltima sequncia de igualdade usamos, novamente, apenas a definio de soma em m . ( P3) Dado a m , basta tomar a classe a e verificar que:
a + ( a) = a

( a) = 0

Para provar a unicidade, suponhamos que b comutatividade, b + a = 0 . Temos ento:

m tambm verifica a + b = 0 ou, usando da

b = b + 0 = b + ( a + ( a) ) = ( b + a ) + ( a) = 0 + ( a) =

a.

A verificao de P2 imediata. Note, porm, que a classe do elemento neutro formada tambm pelos mltiplos de m. Temos, assim, que 0 = m . Da demonstrao de P3 vem que o oposto de a em m a classe
a = a . claro que, se

explicitamos m na forma m = { 0 , 1 , 2 , ..., m 1 } e a um dos representantes utilizados, ento a no um deles. Para obter o menor representante positivo da classe de a, fazemos:
a = 0 + ( a) = m + ( a) = m a = m a

126

CAPTULO 9 CONGRUNCIAS

Por exemplo, em Z7 temos que

2=02=72=7

2 = 5 (de fato, 2 + 5 = 7 = 0 ).

Listamos na prxima proposio as propriedades do produto e a propriedade distributiva, que relaciona ambas as operaes. Proposio 9.3: Em m valem as seguintes propriedades: (P5) Propriedade Associativa: Para toda terna a , b , c de inteiros mdulo m, tem-se que: a . ( b . c )=( a . b ). c (P6) Existncia do Elemento Neutro: Existe o nico elemento em m que precisamente 1 , tal que: a . 1 = a . (P7) Propriedade do Elemento Oposto: Comentrio mais adiante. (P8) Propriedade Comutativa: Para todo par a , b de elementos de m tem-se que: a . b = b . a. (P9) Propriedade Distributiva: Para toda terna a , b , c de elementos de Zm tem-se que: a . (b + c ) = a .b + a .c . Demonstrao: Tal como na proposio anterior, deixaremos as demonstraes como exerccio. So feitas reduzindo-as ao caso dos inteiros. Voc deve ter notado que no listamos uma propriedade P7 que, no paralelismo que estvamos fazendo com as propriedades das operaes nos inteiros, corresponderia propriedade cancelativa. Isso ocorreu porque ela no vlida em geral. Com efeito, por exemplo, em Z6 temos que 3 . 2 = 6 = 0 , 3 . 4 = 12 = 0 ; logo, 3 . 2 = 3 . 4 , porm 2 4 . No contra-exemplo acima, temos dois elementos no-nulos de Z6 cujo produto zero, situao que no acontece em . Para melhor estudar a propriedade cancelativa, comearemos formalizando esse conceito. Definio 9.5: Um elemento no-nulo a m diz-se um divisor de zero se existe b m , tambm no-nulo, tal que a . b = 0 . Agora, determinaremos quais so os divisores de zero em m . Lema 9.2: Um elemento no-nulo a m divisor de zero, se e somente se, m.d.c.(a, m) 1. Demonstrao: Seja a um divisor de zero e b 0 um elemento de m tal que a . b = 0 . Como a . b = b . a = 0 , temos que a.b 0 (mod m), isto , m divide a.b (representamos por m | a.b). Supondo por absurdo que m.d.c.(a, m) = 1, o Teorema de Euclides diz que para a e b inteiros tais que a | b.c, se m.d.c.(a, b) = 1, ento a | c , assim, vem que m | b, logo, b = 0 , uma contradio. Reciprocamente, suponhamos que m.d.c.(a, m) = d > 1. Vamos determinar um elemento b 0 em Zm tal que a . b = 0 . Podemos escrever a = a1 . d , e m = m1 . d, em que 0 < m1 < m (j que d > 1). Logo, m1
0 . Agora, temos que:

127

CAPTULO 9 CONGRUNCIAS

a . m1 = ( a1 . d ) . m1 = a1 . ( d . m1 ) = a1 . m Logo, em Zm temos:
a . m1 = a1 . m = 0

Assim, basta tomar b = m1.

Como consequncia imediata deste Lema temos o Corolrio abaixo. Corolrio 9.5: Seja p > 1 um inteiro primo. Ento, p no contm divisores de zero. Vale tambm a recproca. Lema 9.3: Se m no contm divisores de zero, ento m primo. Demonstrao: Suponhamos, por absurdo, que m seja composto, isto da forma m = r . s com 1 < r < m, 1 < s < m. Temos, ento, que: 0 = m = r . s em que r 0 e s 0, uma contradio. Proposio 9.4: A propriedade cancelativa do produto vale em m , se e somente se, m primo. Demonstrao: Suponhamos inicialmente que m seja primo, e sejam a , b , c elementos de

m , com a
Como a

0, tais que a . b = a . c . Ento, a .( b c ) = 0.

0 e m no tem divisores de zero, deve ser b c = 0, donde b = c .

Suponhamos que a propriedade cancelativa seja vlida, mostraremos que nesse caso que m no contm divisores de zero. A tese seguir ento do Lema anterior. Sejam a , b Zm tais que a . b = 0 . Se a cancelar, temos que b = 0.
0 , escrevemos a . b = a . 0 e, como podemos

Para continuar nosso estudo comparativo de com m , introduzimos ainda outro conceito. Definio 9.6: Um elemento a m diz-se inversvel se existe a' m tal que a . a' = 1. Um elemento a' nessas condies diz-se um inverso de a . O conjunto dos elementos de m que tm inversos muito importante. Vamos denot-lo por (m). Em outras palavras,

128

CAPTULO 9 CONGRUNCIAS

(m) = { a

m ; m.d.c.(a, m) = 1 }

No caso de m ser primo, todas as classes diferentes de 0 possuem inverso. Os nicos elementos inversveis de so 1 e 1. Obviamente, 1 e 1 so sempre inversveis em m . Porm h outros exemplos. Em Z5 temos que 2 . 3 = 6 = 1 e 4 . 4 = 16 = 1 , logo 2 , 3 e 4 so tambm inversveis de Z5, 2 o inverso de 3 e, reciprocamente, 4 o seu prprio inverso. Em Z6 temos que 5 . 5 = 25 = 1 ; logo, 5 um inversvel de Z6. Por outro lado, claro que 0 no inversvel em m , para nenhum valor de m. De fato, para qualquer a m temos que 0 . a = 0 1 . Observe as tabelas de multiplicao em Z6 e em Z7. * 0 1 2 3 4 5 6 0 0 0 0 0 0 0 0 1 0 1 2 3 4 5 6 2 0 2 4 6 1 3 5 3 0 3 6 2 5 1 4 4 0 4 1 5 2 6 3 5 0 5 3 1 6 4 2 6 0 6 5 4 3 2 1 * 0 1 2 3 4 5 0 0 0 0 0 0 0 1 0 1 2 3 4 5 2 0 2 4 0 2 4 3 0 3 0 3 0 3 4 0 4 2 0 4 2 5 0 5 4 3 2 1

Com exceo de 0, todos os elementos em Z7 (7 primo) possuem inverso, enquanto que em Z6 (6 no primo), apenas 1 e 5 possuem inverso. Isso est de acordo com a proposio abaixo. Proposio 9.5: Seja a um elemento no-nulo de m . Ento, a inversvel, se e somente se, m.d.c.(a,m) = 1. Demonstrao: Suponhamos que m.d.c.(a, m) = 1. O Teorema de Bzout afirma que sendo a e b inteiros, d = m.d.c.(a, b). Ento existem inteiros r e s tais que d = r.a + s.b . Tambm, existem inteiros r e s tais que a.r + m.s = 1. Tomando classes temos que:
1 = a.r m.s = a.r + m.s = a . r + m . s = a . r + 0 . s = a . r

Logo, r o inverso de a . Reciprocamente, se m.d.c.(a, m) 1, ento a divisor de zero e existe b 0 tal que a . b = 0 . Mostraremos que, nesse caso, a no pode ser inversvel. Com efeito, suponhamos que existe a' tal que a.a' = 1 . Teramos, ento: b = b . 1 = b .( a.a' ) = ( b . a ). a' = ( a . b ). a' = 0 . a' = 0 , uma contradio.

129

CAPTULO 9 CONGRUNCIAS

Uma consequncia imediata da proposio anterior a seguinte: Corolrio 9.6: Seja p > 0 um inteiro primo. Ento, todo elemento no-nulo de p inversvel. Exemplo 9.3: Veja na tabuada de Z11 que todo elemento no nulo, possui um inverso: * 1 2 3 4 5 6 7 8 9 10 1 1 2 3 4 5 6 7 8 9 10 2 2 4 6 8 10 1 3 5 7 9 3 3 6 9 1 4 7 10 2 5 8 4 4 8 1 5 9 2 6 10 3 7 5 5 10 4 9 3 8 2 7 1 6 6 6 1 7 2 8 3 9 4 10 5 7 7 3 10 6 2 9 5 1 8 4 8 8 5 2 10 7 4 1 9 6 3 9 9 7 5 3 1 10 8 6 4 2 10 10 9 8 7 6 5 4 3 2 1

9.7. Subtrao em m

A operao de subtrao no sistema aritmtico mdulo m, tal como na aritmtica comum, definida em termos de adio. Na aritmtica comum, podemos achar a resposta para 13 4, procurando o nmero que somado a quatro nos d 13. Como 4 + 9 = 13, o nmero 9 a resposta do problema 13 4 = 9. Voc deve ter usado muito este processo ao conferir resultados de subtraes. Na verificao adicionamos a diferena ao subtraendo para ver se a soma se iguala ao minuendo. Dizemos que a subtrao a operao inversa da adio. Usando os elementos md. 7, resolva o problema de subtrao 6 5 = ? Qual o elemento que somado a 5 resulta em 6? Da tabela da adio md. 7, 5 + 1 = 6. Portanto, podemos substituir 1 no lugar do ponto de interrogao, e 6 5 = 1.

130

CAPTULO 9 CONGRUNCIAS

Na aritmtica comum, no podemos subtrair 5 6 sem termos um resultado negativo. Os nmeros negativos no so necessrios na subtrao md. m. No problema 5 6 = ?, procuramos um elemento que somado a 6 nos d 5. A tabela da adio nos revela que a soma de 6 + 6 5. Portanto, 6 pode substituir o? e 5 6 = 6. Voc encontrar outros problemas de subtrao nesse sistema que so bem diferentes e interessantes. Considere o problema 4 5 = n, onde n a resposta. Pela nossa definio de subtrao n + 5 deve ser 4. Qual o nmero que somado a 5 em md. 7 4? Da tabela da adio, vemos que n = 6 e, portanto, 4 5 = 6. Neste sistema finito, 1 - 4 = 4 1? No primeiro membro, qual o elemento que somado a 4 d 1? Como 4 + 4 = 1, 1 4 = 4. Do mesmo modo, temos que achar um nmero que somado a 1 resulta em 4. Como 1 + 3 = 4, 4 1 deve ser 3. Voc v, portanto, que 1 4 4 1 e a propriedade comutativa no vlida para a operao subtrao. A tabela abaixo resume o que foi dito.

7
0 1 2 3 4 5 0 0 6 5 4 3 2 1 1 0 6 5 4 3 2 2 1 0 6 5 4 3 3 2 1 0 6 5 4 4 3 2 1 0 6 5 5 4 3 2 1 0 6 6 5 4 3 2 1

9.8. Diviso em m

No necessrio usar fraes se voc trabalhar no sistema aritmtico de mdulos. Como isto possvel? Vamos considerar a operao de diviso. Na diviso de nmeros naturais, verificamos as respostas, multiplicando o quociente pelo divisor para chegarmos ao dividendo. Assim, 125 25 = 5 pois 5 . 25 = 125. A operao diviso, na aritmtica mdulo m, tambm pode ser definida como operao inversa da multiplicao. Por exemplo, usando o md.7, o problema 5 4 resolvido encontrando um nmero n, tal que n . 4 = 5. J que completamos a tabela de multiplicao md. 7 podemos us-la para achar a resposta. Temos que, 3 . 4 = 5 e, portanto, 5 4 = 3. Qual a resposta de 6 3? Como 2 . 3 = 6, a resposta ao problema 6 3 2. Neste sistema a diviso, quando existe, sempre exata.

131

CAPTULO 9 CONGRUNCIAS

Se, consideramos a expresso 3 2 como sendo o mesmo que teremos uma interessante vantagem na aritmtica de mdulos. 3 1 32=5 e tambm 1 3 = 5 2 3

3 1 e 1 3 o mesmo que , 2 3

As fraes so rapidamente substitudas por algum elemento do sistema md. 7. Uma expresso tal 1 2 como 4 simplificada de modo anlogo. 1 3 3 A frao
1 equivalente a 2, pois em md. 7, 4 . 2 = 1. 4 1 1 equivalente a 2 + 2, ou 4. O denominador, 3 , igual a 1, pois 4 3 4 1 4 , pois 1 . 4 = 4.

Portanto, 2
1 3

5 e 3 + 5 = 1 em md. 7. Finalmente

Considere o problema 2 6 em aritmtica md. 7. Neste caso, temos que achar a resposta n, tal que n . 6 = 2. Como 5 . 6 = 2, ento 2 6 = 5. Entretanto, se mudarmos o problema para 6 2 teremos uma resposta totalmente diferente. No caso de 6 2 a resposta 3, pois 3 . 2 = 6. evidente que 6 2 no igual a 2 6, portanto, a propriedade comutativa no se verifica na diviso mdulo m. A tabela abaixo resume o que foi dito.

7
1 2 3 4 5 6 1 1 4 5 2 3 6 2 2 1 3 4 6 5 3 3 5 1 6 2 4 4 4 2 6 1 5 3 5 5 6 4 3 1 2 6 6 3 2 5 4 1

Podemos resolver facilmente o problema 3.x = 0, usando a multiplicao de nmeros naturais da aritmtica comum. Sabemos que o produto 0; portanto, o numero que devemos colocar no lugar de x deve ser o zero. Voc j deve ter visto muitas vezes a afirmao de que se um produto zero, pelo menos um dos fatores deve ser igual a zero. Isto no sempre verdade! Pelo menos no verdade em alguns sistemas aritmticos finitos. possvel um problema de diviso ter mais de uma resposta? Voc deve estar imaginando que isto nunca possvel. Entretanto, no sistema aritmtico mdulo m isso pode ocorrer.

132

CAPTULO 9 CONGRUNCIAS

Consideremos sistema mdulo 6. Neste sistema, 4 4 = 1. Isto no deve ser estranho para voc, mas acontece que 4 4 tambm igual a 4 na aritmtica md. 6. Vamos explorar este sistema finito mais detalhadamente. J vimos que os sistemas aritmticos mdulo m, podem ser divididos em duas categorias: o de mdulo primos e no- primos. Consideremos os sistemas aritmticos onde m no-primo. Vamos usar como exemplo o mdulo 6. Na tabela completa de multiplicao abaixo, notamos alguns padres um pouco estranhos. Quando a tabela simtrica com relao a diagonal, dizemos que a operao comutativa. Entretanto, repare nos produtos resultantes da multiplicao por 2. Alm do estranho resultado de 2 . 3 ser igual a 0, apesar de nenhum fator ser 0. E nesta multiplicao s temos como respostas o 0, 2 e 4. * 1 2 3 4 5 1 1 2 3 4 5 2 2 4 0 2 4 3 3 0 3 0 3 4 4 2 0 4 2 5 5 4 3 2 1

Do mesmo modo, somente teremos respostas 0 ou 3 na multiplicao por 3. Estes fatos tornarse-o mais significativos quando estudarmos a diviso neste sistema. Alguma vez voc trabalhou num problema de matemtica durante tanto tempo, sem conseguir resolv-lo? Voc deve ter comeado a desconfiar que talvez o problema no tivesse soluo. Isto pode parecer estranho, mas alguns problemas em matemtica no tem soluo para as condies dadas. Isto acontece com a diviso no sistema da aritmtica de mdulos no-primos. Seja calcular 5 1(mod 6), quer dizer, um nmero tal que, quando multiplicado por 1 d o produto 5. Em termos gerais, a b = c ,tal que c . b = a . Podemos ver na tabela de multiplicao que 1 . 5 = 5, portanto, 5 1 = 5. Deparamos com uma situao diferente no problema 4 2. A tabela de multiplicao mostra claramente que 2 . 2 = 4 e tambm 2 . 5 = 4. Portanto, ambos os elementos 2 e 5 satisfazem aos requisitos para o quociente 4 2. Mas h mais! No problema 5 3, procuramos um quociente n, tal que n . 3 = 5. Olhe a tabela de multiplicao para mdulo 6. No h um nmero n tal que n . 3 = 5. O problema 5 3 no tem resposta neste sistema. Verifique as respostas na tabela da diviso mdulo 6 completa absixo. Relacione estas respostas com a tabela de multiplicao desenvolvida anteriormente. A diviso por zero

133

CAPTULO 9 CONGRUNCIAS

excluda. Na tabela abaixo, e nas demais, estaremos considerando cada um dos elementos da primeira linha como o dividendo e cada um dos elementos da primeira coluna como o divisor. Dividendo 1 2 1 5 2 1; 4 2; 5 4

1 2 3 4 5

3 3 1; 3; 5 3

4 4 2; 5 1; 4 2

5 5 1 Divisor

Exemplo 9.4: Construa as tabelas de multiplicao e diviso mdulo 8.

* 1 2 3 4 5 6 7

1 1 2 3 4 5 6 7

2 2 4 6 0 2 4 6

3 3 6 1 4 7 2 5

4 4 0 4 0 4 0 4

5 5 2 7 4 1 6 3

6 6 4 2 0 6 4 2

7 7 6 5 4 3 2 1

1 2 3 4 5 6 7

1 1 3 5 7

2 2 1; 5 6 2 3; 7 6

3 3 1 7 5

4 4 2; 6 4 1; 3; 5;7 4 2; 6 4

5 5 7 1 3

6 6 3; 7 2 6 1; 5 2

7 7 5 3 1

134

CAPTULO 9 CONGRUNCIAS

9.9. Potenciao em m
Exponenciao Rpida Sejam a e n inteiros positivos. Temos que
k

n=
i=0

n i 2i

a expanso binria de n. Os coeficientes n i so 0 ou 1. Portanto


k

n i 2i

i=0

=
i=0

(a 2 ) ni
0 i ki ni 1

a2

Com isto, desenvolvemos a seguinte ideia: 1. 2. Calculamos os quadrados sucessivos de a 2 , 0 i k . i Determinamos a n como o produto daqueles a 2 para os quais ni =1 .
i+1

Observe que a 2
i+1

(a 2 )2
i

Portanto, a 2 pode ser calculado de a 2 por uma elevao ao quadrado. Vejamos um exemplo: Seja calcular 144823 (mod1037) . Primeiro faamos a expanso binria de 823: 823 = (1100110111)2 Escrevamos essa expanso na base dez: (1100110111)2 = 29 + 28 + 25 + 24 + 22 + 21 + 20 (1100110111)2 = 512 + 256 + 32 + 16 + 4 + 2 + 1 Logo, 144823 = 144512 . 144256 . 14432 . 14416 . 1444 . 1442 . 1441 Agora, calculemos cada uma das potncias por uma elevao sucessiva potncia 2. 144 144 (mod 1037) 1442 1033 (mod 1037) 1444 16 (mod 1037)

135

CAPTULO 9 CONGRUNCIAS

1448 256(mod 1037) 14416 205(mod 1037) 14432 545(mod 1037) 14464 443(mod 1037) 144128 256(mod 1037) 144256 205(mod 1037) 144512 545 Assim,
144823 545.205.545.205.16.1033.144 = 766.766.117 mod1037 = 851.117 mod1037 = 99567 mod1037 = 15 mod1037 mod1037

Concluso: 144823 15 (mod 1037). Voltaremos a falar deste assunto, depois de estudarmos o Algoritmo Chins do Resto e o Pequeno Teorema de Fermat. Leitura: Teste de Primalidade Circular O Teste de Primalidade Circular , em termos gerais, semelhante verificao tradicional de primalidade de dividir um inteiro N > 1 pelos primos menores ou iguais ao piso da raiz quadrada de N. A principal diferena entre o Teste Circular e o mtodo tradicional que, neste caso, os divisores no so todos primos. Podemos tambm perceber uma diferena no que diz respeito ao desempenho. O Teste Circular, tambm pode ser considerada um tipo especial de crivo.

136

CAPTULO 9 CONGRUNCIAS

Procedimento O primeiro passo consiste em escolher uma pequena quantidade dos primeiros primos. Por exemplo, escolhe-se os primos 2, 3 e 5. As classes de congruncias que sero consideradas, tero mdulo m igual ao produto desses primos: m = 2 . 3 . 5 = 30 Em seguida, toma-se todos os inteiros positivos menores que 30 que no sejam mltiplos de 2, 3, e 5 ( relativamente primo com 30). Elaborando uma pequena tabela, e removendo os mltiplos, obtm-se: 1 7 13 19 25 2 8 14 20 26 3 9 15 21 27 4 10 16 22 28 5 11 17 23 29 6 12 18 24 30

Os nmeros que no foram marcados formam a base do crivo: 1, 7, 11, 13, 17, 19, 23, 29 (mdulo 30). Cada um desses nmeros indicar as classes de congruncias mdulo 30 que sero utilizadas, ou seja,

1,7,11,13,17,19, 23 e 29 .

Finalmente, semelhante ao mtodo tradicional, para se verificar que um dado nmero N primo, divide-se N pelos primos iniciais, usados para gerar m, e pelos elementos das classes de congruncias (com exceo do 1) que sejam maiores que, ou iguais a N . Se N no for divisvel por nenhum desses termos, ento N primo. Um exemplo prtico Para verificar se 3331 primo, define-se o limite superior 3331 = 57. Em seguida, divide-se 3331 pelos primos iniciais 2, 3, 5 e por todos os elementos das classes (mod 30), que no ultrapassam o valor de 57, at se obter um resto zero ou a diviso chegar ao ltimo elemento das classes (mod 30) sem se obter uma diviso exata, o que signficar que 3331 primo. Sejam as classes com seus respectivos elementos menores que 57:
1 {1,31,...}
7 {7,37,...} 11 {11, 41,...} 13 {11, 41,...} 17 {17, 47,...}

1,7,11,13,17,19, 23 e 29

19 {19, 49,...}
23 {23,53,...} 29 {29,...} .

137

CAPTULO 9 CONGRUNCIAS

Assim, os elementos das classes, que no ultrapassam o valor 57, com exceo do 1, sero usados para o teste de primalidade de 3331 e mais os primos 2, 3 e 5, ou seja, dividiremos 3331 pelos seguintes nmeros : 2, 3, 5, 7, 11, 13, 17, 19, 23, 29, 31, 37, 41, 43, 47, 49, 53. Observe que nesta lista, diferente do mtodo tradicional, temos o nmero 49 que no primo! Como 3331 no divisvel por nenhum desses nmeros, ento ele primo. Nota: claro que um inteiro maior que 5, que candidato a primo no pode ser par e nem terminar em 5. Logo, a diviso por 2 e 5 devem ser descartadas, bem como por 1. Qualquer candidato a primo,obrigatoriamente dever ser um mpar que termine em 1 ou 3 ou 7 ou 9.

Uma forma indireta de crivo claro que o aparecimento de nmeros compostos na sequncias dos provveis divisores de N uma mera consequncia terica. Antes que um N seja divisvel por um composto, j ter sido divisvel por algum primo que seja um divisor deste composto. Uma vantagem deste mtodo no ser preciso saber se um nmero ou no primo para us-lo como provvel divisor, como acontece no mtodo tradicional. Pelo Teorema de Dirichlet, existem infinitos primos em cada classe de congruncia a mdulo m. O que podemos afirmar que, na sequncia dos nmeros a serem usados, se houver divisores de N, o menor deles necessariamente um primo de uma das classes de congruncias. Isso nos d um mtodo indireto para saber se um nmero primo, ou seja, uma forma indireta de crivo. Vamos exemplificar: Queremos saber se N = 3127 composto. Se existirem nmeros que dividam N em alguma das classes a serem usadas, ento o menor desses divisores primo. De fato, seja 3127 55 . Consideremos m = 2.3 = 6, logo as classes mdulo 6 so:

1={7,13,19,25,31,37,43,49,55} 5={11,17,23,29,35,41,47,53}

Da classe 5 , temos que 53 o menor divisor de 3127, logo 53 primo.

138

CAPTULO 9 CONGRUNCIAS

EXERCCIOS
1) Verdadeiro (V) ou falso (F) a) 91 0 (mod.7). b) 3 + 5 + 7 5 (mod.10). c) 2 2 (mod.8). d) 112 1 (mod.3). e) 17 9 (mod.2). f) 42 -8 (mod.10). b) c) d) e) f) 4165 por 7 25100 + 11500 por 3. 35555 por 80. 51000 por 126. 3399300 + 29 por 13

2) Verdadeiro (V) ou falso (F) a) x 3 (mod.5) x { .... 7, -2, 3, 8, 13 .... }. b) 5 -1 (mod.6) e -1 -7 (mod.6) 5 -7 (mod.6). 3) Achar o menor inteiro positivo que represente a soma: a) 5 + 3 + 2 + 1 + 8 (mod. 7) b) 2 + 3 1 + 7 2 (mod.4) 4) Sabendo-se que 1766 1066 (mod. m), achar todos os possveis valores do mdulo m. 5) Exprimir que n mpar de trs outras maneiras. 6) Achar todos os inteiros x tais que 0 < x < 15 e 3x 6 (mod. 15) 7) Achar todos os inteiros x tais que 1 < x < 100 e x 7 (mod. 17) 8) Sabendo-se que k 1 (mod. 4), mostrar que 6k + 5 3 (mod. 4) 9) Mostrar, mediante um exemplo, que a2 (mod.m) no implica a b (mod.m). b2

15) Mostrar que a) 89 | 244 1. b) 97 | (248 1) c) 13| 270 + 370 d) n| 1n 2n ... (n 1)n ,, n mpar. 16) Demonstrar que, se a b (mod. m) ento mdc(a, m) = mdc(b, m). 17) Mostrar, mediante um exemplo, que ak (mod. m) e k j no implica aj bj. bk

18) Demonstrar as seguintes proposies: a) Se a um inteiro mpar ento a2 1 (mod. 8) b) Se a um inteiro qualquer, ento a3 0, 1 ou 8 (mod. 9). c) Se a um inteiro qualquer, ento a3 a (mod. 6). 19) Mostre que 7 divide 22225555+55552222. 20) Determine o resto da diviso de 61987 por 37. 21) Prove que 7 divide 32n+1+2n+2 para todo natural n. 22) Determine o algarismo das unidades de

77 .

23) Determine os infinitos valores positivos de n tal que 2n +27 seja divisvel por 7. 24) Prove que 2k 5, k 4 nunca deixa resto 1 quando dividido por 7. 25) Prove que 4 divide 12233 . 455679+87653 3 26) Prove que 11. 31. 61 | 2015 - 1.

10) Mostrar que todo primo mpar congruente mdulo 4 a 1 ou 3. 11) Mostrar que todo primo maior que 3 congruente mdulo 6 a 1 ou 5. 12) Mostrar que a) 1110 1 (mod 100) b) 31000 + 3 divisvel por 28. 13) Mostrar que 41 divide 220 1. 14) Achar os restos das divises de: a) 250 por 7

27) Calcular o resto da diviso do n = (116 + 1717)21 por 8. 28) Calcular o resto da diviso do inteiro 7100 + 11100 por 13

inteiro

139

CAPTULO 9 CONGRUNCIAS 29) Mostrar que o inteiro n = 13 divisvel por 3.


16

2 .5

43

17

45) Determine para que inteiros positivos n, 2n 1 divisvel por 7. 46) Prove que 2n + 1 nunca divisvel por 7, para qualquer inteiro positivo n. 47) Demonstre que para todo inteiro n um divisor de 34n + 2 + 26n + 3.

30) Mostrar que a expresso 3.5n+1 + 2n+1 divisvel por 17, sendo n um inteiro positivo qualquer. 31) Mostrar que o nmero de 2 + 1 divisvel por 641. 32) Demonstrar que, se o inteiro positivo n no divisvel por 4, ento a soma: S = 1n + 2n + 3n + 4n divisvel por 5. 33) Determinar todos os inteiros positivos n para os quais 2n + 1 divisvel por 3. 34) Mostrar que 63! 61! (mod 7)

0 , 17

48) Determine o algarismo das dezenas do nmero 7999999. 49) Determine o resto da diviso por 7 do nmero 2 3 100 1010 1010 1010 ... 1010 . 50) Mostre que, para todo 102n + 1 + 1 0 (mod 11). n natural, temos

35) Mostrar que no existe inteiro algum n que verifique as condies: n 5 (mod 12) e n (mod 15)

36) Na diviso do inteiro n por 10 o quociente q e o resto r, tais que 2r q 0 (mod 7). Mostrar que 7 n. 37) Determinar quais dos seguintes conjuntos so sistemas completos de restos mdulo 4. a) { -2, -1, 0, 1} b) {0, 4, 8, 12} c) { -13, 4, 17, 18 } d) {5, 0, 6, 22 } 38) Determinar quais dos seguintes conjuntos so sistemas completos de restos mdulo 6. a) { 1, 2, 3, 4, 5} b) {0, 5, 10, 15, 20, 25} c) {-4, -3, -2, -1, 0, 1} d) {17, -4, 6, 7, 10, 3} 39) Achar um sistema completo de restos {p1, p2, ...pi, ..., p7} mdulo 7, tal que todo pi primo. 40) Achar um sistema completo de restos mdulo 7 formado s de mltiplos no negativos de 4. 41) Se n um mltiplo positivo de 4, qual o resto da diviso de 1n + 2n + ... + 8n + 9n por 10? 42) Mostre que 121n 25n + 1900n (4)n divisvel por 2000, para todo natural n.. 43) Determinar o algarismo das unidades do nmero 3100. 44) Que valores de a e b tornam o nmero 30a0b03 divisvel por 13?

140

CAPTULO 10 FERMAT, WILSON E EULER

Captulo 10

TEOREMAS DE FERMAT, WILSON e EULER


Fermat foi um dos poucos matemticos amadores famosos. Filho de um rico comerciante de couro, pde se dedicar completamente aos estudos. Por influncia de sua me, descendente de uma famlia de juristas, estudou leis na Universidade de Orleans e formou-se em advocacia. Trabalhou durante toda sua vida na corte de justia de Toulouse. Foi nomeado juiz e ocupava os seus momentos de folga em diversos lazeres, entre os quais a poesia e a Matemtica. Seu interesse pela matemtica iniciou-se em 1629 com o estudo dos trabalhos de Apolnio (matemtico grego, 260 A.D.) sobre curvas planas. Trocava correspondncia com os maiores matemticos da poca, como Torricelli, Roberval, Huyghens e Pascal, e, dessa forma relatava suas descobertas. Jamais publicou seus trabalhos de nenhuma outra forma, mas o contedo das cartas de Fermat atualmente includo em todos os textos usuais de teoria dos nmeros. Seu interesse na teoria dos nmeros surgiu aps ler o livro Aritmtica de Diofanto (matemtico grego, 200 A.C.) e alguns dos problemas propostos por Fermat, nesta rea, eram to difceis que somente muitos anos mais tarde foram provados. Um desses problemas afirmava que "todo nmero inteiro pode ser escrito como a soma de no mximo quatro quadrados" e foi provado em 1770, pelo matemtico francs Lagrange. Entretanto, seu resultado mais famoso resistiu por mais de 350 anos e inspirou a publicao, em 1996, do bestseller O ltimo Teorema de Fermat. Este teorema diz que se n um natural maior que 2, ento no existem nmeros inteiros x, y e z que satisfaam a equao xn + yn = zn. Isto foi provado definitivamente, em 1994, pelo matemtico ingls Andrew Wiles (repare que no caso n = 2 o teorema satisfeito por todos os ternos pitagricos, isto , por inteiros que satisfaam o Teorema de Pitgoras).

10.1.
Teorema 10.1 (Pequeno Teorema de Fermat PTF): se p primo e se o MDC(p, a) = 1, ento: ap1 1 (mod p)

Demonstrao: consideremos os (p 1) primeiros positivos de s, isto , os inteiros

141

CAPTULO 10 FERMAT, WILSON E EULER

a, 2 . a, 3 . a, ..., (p 1) . a Obviamente, nenhum desses (p 1) inteiros divisvel por p e, alm disso, dois quaisquer deles so incongruentes mdulo p, pois, se fosse: r.a s.a (mod p), 1 r<s p1

ento, o fator comum a poderia ser cancelado, visto que o MDC(a, p) = 1, e teramos: r s (mod p), isto p | (a x)

o que impossvel, porque 0 < s r < p. Assim sendo, dois quaisquer dos (p 1) inteiros a, 2.a, 3.a, ..., (p 1).a divididos por p deixam restos distintos, e por conseguinte cada um desses p 1 inteiros congruente mdulo p a um nico dos inteiros 1, 2, 3, ..., p 1, naturalmente numa certa ordem, multiplicando ordenadamente essas p 1 congruncias, teremos: a . 2a . 3a . ... . (p 1)a ou seja, ap1 (p 1)! (p 1)! (mod p) 1 . 2 . 3 . ... . (p 1) (mod p)

Como o MDC (p, (p 1)!) = 1, porque p primo e p no divide (p 1)!, podemos cancelar o fator (p 1)!, o que d a congruncia de Fermat: ap1 1 (mod p)

Exemplo 10.1: seja o primo p = 7 e o inteiro a = 3 tais que 7 no divide 3, temos os p 1 = 6 primeiros mltiplos positivos de 3: 3, 6, 9, 12, 15, 18. Nenhum desses 6 inteiros divisvel por 7, todos so incongruentes mdulo 7, e cada um deles congruente mdulo 7 a um nico dos inteiros 1, 2, 3, 4, 5, 6: 3 3 (mod 7), 6 6 (mod 7), 9 2 (mod 7), 12 5 (mod 7), 15 1 (mod 7), 18 4 (mod 7). Multiplicando ordenadamente essa 6 congruncias, temos: 3 . 6 . 9 . 12 . 15 . 18 ou seja, 36 . 6! 6! (mod 7) 3 . 6 . 2 . 5 . 1 . 4 (mod p)

Como o MDC(7, 6!) = 1, podemos cancelar o fator comum 6!, que resulta em:

142

CAPTULO 10 FERMAT, WILSON E EULER

36 Corolrio 1: se p um primo, ento ap Demonstrao: se p divide a, ento a

1 (mod 7)

a (mod p), qualquer que seja o inteiro a. 0 (mod p) e ap ap a (mod p) 0 (mod p), que implica em:

Se, ao invs disto, p no dividisse a, ento pelo PTF: ap1 1 (mod p), e ap a (mod p) a (mod q) e aq a (mod p), ento:

Teorema 10.2: se p e q so primos distintos tais que ap apq a (mod p.q)

Demonstrao: pelo corolrio 1 e da hiptese, temos: (aq)p (ap)q aq (mod p) ap (mod q) aq aq a (mod p) a (mod q) apq apq a (mod p) a (mod q)

Portanto, p | (apq a), e q | (apq a), que implica em: (p.q) | (apq a), isto , apq a (mod p.q)

Pelo corolrio 1, se p primo, ento 2p 2 (mod p), isto , p | (2p 2). Entretanto, a recproca se n | (2n 2), ento n primo no uma proposio verdadeira, e os inteiros positivos mpares compostos que satisfazem essa condio so chamados de pseudoprimos. Por exemplo, 2341 2 (mod 341), isto , 341 | (2341 2), onde o inteiro 341 = 11.31 composto. Com efeito, temos: 210 = 1024 = 3.11.31 + 1 que implica em 210 Portanto, 211 2 (mod 31), e 231 2.(210)3 (mod 11) 2.(1)3 (mod 11) 2 (mod 11) 1 (mod 11), e 210 1 (mod 31)

143

CAPTULO 10 FERMAT, WILSON E EULER

Como os inteiros 11 e 31 so primos distintos, temos, de acordo com o Teorema 2: 211.31 2 (mod 11.31), isto , 2341 2 (mod 341), ou que 2340 1 (mod 341)

Congruncia que mostra ser falsa a recproca do PTF. Logo, 341 um pseudoprimo na base 2. Definio 1 (Pseudoprimos): todo inteiro positivo composto n tal que 2n chama um pseudoprimo para a base 2. 2 (mod n) se

H um nmero infinito de pseudoprimos para a base 2, mas somente sete so menores que 2000, e todos mpares: 341 = 11 . 31 645 = 3 . 5 . 43 1387 = 16 . 73 561 = 3 . 11 . 17 1105 = 5 . 13 . 17 1729 = 7 . 13 . 19

Nota: O primeiro exemplo de um pseudoprimo par para a base 2 foi dado pelo matemtico americano LEHMER em 1950. O menor deles 161038 = 2 . 73 .1103 e, em 1951, BEEGER mostrou a existncia da infinidade de pseudoprimos pares para a base 2. Teorema 10.3: seja um inteiro a > 1 e um primo p > 2.

a2p 1 a2 1

ap 1 a 1

ap 1 a 1

um pseudoprimo na base a. Demonstrao: pelo PTF,

ap 1 a 1

ap 1 (mod p) a 1

1 (mod p)

e se verifica facilmente que estes nmeros so mpares, de onde n 1 (mod 2.p), ou n = 2.k.p + 1, para um k inteiro. Assim, como a2p 1 (mod n), temos: an a2kp+1 (mod n) (a2p)k . a (mod n) a (mod n)

Existe uma infinidade de pseudoprimos de base a > 1. Faamos a demonstrao no caso de a = 2.

144

CAPTULO 10 FERMAT, WILSON E EULER

Proposio 1: se n um pseudoprimo de base 2, ento 2n 1 tambm um pseudoprimo de base 2. Demonstrao: dada a igualdade (xt 1) = (x 1).(x(t1) + x(t2) + ... + x2 + x + 1), sejam d, k tais que n = d.k, d > 1, k < n, e seja m = 2n 1. Ento m composto.Para isto, basta considerar na igualdade acima x = 2d e t = k. Por hiptese 2n1 1 (mod n) e, portanto, existe k > 0 tal que 2n1 1 = k.n. Assim,

2m 1 1

2(2

2)

22.(2

n 1

1)

22km

Voltando igualdade acima e fazendo x = 2n e t = 2.k, conclumos que 2m1 1 um mltiplo de m = 2n 1, ou seja 2m1 1 (mod m) 2m 2 (mod m). Conclumos assim que m um pseudoprimo de base 2. Corolrio 2: existe uma infinidade de pseudoprimos de base 2. Demonstrao: basta notar que 341 um pseudoprimo de base 2 e aplicar sucessivamente a proposio anterior.

Nota: O fato de existir uma infinidade de pseudoprimos em qualquer base no implica a existncia de uma infinidade de pseudoprimos.

EXERCCIOS
1) Verificar o PTF com a = 2 e p = 13. 2) Verificar o PTF com a = 3 e p = 17. 8) Demonstrar que 13 | (270 + 370) atravs do PTF. 3) Verificar utilizando o PTF: a) 1850 2 (mod 7) b) (b) 19933 8 (mod 31) 4) Mostrar que 538 5) 4 (mod 11) pelo PTF. 9) Demonstra que 22225555 + 55552222 divisvel por 7 . 10) Achar o resto da diviso de 21137 por 17 com a ajuda do PTF. 11) Mostrar que, se o MDC(a, 35) = 1, a12 1 (mod 35). ento 7) Sendo a um inteiro, mostrar que n5 e n tm o mesmo algarismo das unidades.

Mostrar que o inteiro 117 composto usando o PTF.

6) Achar o algarismo das unidades do inteiro 3 400 com o auxlio do PTF.

12) Demonstrar que, para todo inteiro a, se tem: a) a13 a (mod 7) b) a37 a (mod 13)

145

CAPTULO 10 FERMAT, WILSON E EULER c) a21 a (mod 15) d) a7 a (mod 42) 13) Demonstrar que, para todo inteiro positivo n, se tem: a) 22n 1 (mod 3) b) 23n 1 (mod 7) 14) Achar todos os primos p tais que p | (2p + 1). 15) Demonstrar que 1105 | (31105 3) [sugesto: PTF]. 16) Demonstrar que 161038 um pseudoprimo para a base 2, i.e., que 161038 | (2161038 2). 17) Mostrar que 2047 um pseudoprimo para a base 2. 18) Mostre que 341 no um pseudoprimo para a base 3. 22) Demonstrar que se MDC(a, 240) = 1, ento 240 um divisor de a4 1. 23) Demonstrar que todo primo maior que 5 divide um inteiro formado s de algarismos 1. 24) Demonstrar que para todo inteiro positivo n, temos que (n3 n).(58n+4 + 34n+2) um mltiplo de 3804. 25) Prove que 15 divide 3n5 19) Mostrar que 561 um pseudoprimo para a base 2, 3 e 5. 20) Demonstrar que se 7 no divide n, n6 1996 (mod 7). 21) Demonstrar que se MDC(n, 7) = 1 7 | (n12 1). ento

ento,

5n3 7n .

26) Prove que 91 divide a12 b12 , onde a e b so relativamente primo com 91.

10.2. TEOREMA DE WILSON


Teorema 10.4 (Teorema de Wilson): se p um primo, ento (p 1)! Demonstrao: o teorema verdadeiro para p = 2 e para p = 3, pois: (2 1)! = 1! = 1 (3 1)! = 2! = 2 1 (mod 2) 1 (mod 3) 1 (mod p).

de modo que vamos supor p 5. Consideremos a congruncia linear a.x 1 (mod p), onde a um dos (p 1) primeiros inteiros positivos 1, 2, 3,..., p 1 de modo que o MDC(a, p) = 1. Nestas condies, existe um nico inteiro positivo a, com 1 a.a 1 (mod p) a p 1, tal que

Como p primo, tem-se que a = a se e somente se a = 1, ou a = p 1, visto que a2 1 (mod p) implica em (a 1).(a + 1) 0 (mod p) e, portanto, a1 0 (mod p), ou (a + 1) 0 (mod p)

146

CAPTULO 10 FERMAT, WILSON E EULER

isto , a = 1 ou a = p 1. Nota: Este teorema foi descoberto primeiramente por John Wilson (1741 - 1793), estudante do matemtico ingls Edward Waring. Waring anunciou o teorema em 1770, embora nenhum deles tenha conseguindo prov-lo. Lagrange deu a primeira prova em 1773. H uma evidncia que Leibniz estava ciente do resultado um sculo antes, mas nunca o publicou. Exemplo 10.2: Com p = 13, existe um nico inteiro positivo a, com 1 a.a a a' 1 1 2 7 3 9 4 10 5 8 1 (mod p) 6 11 7 2 8 5 9 3 10 4 11 6 12 12 a p 1, tal que

Omitindo os inteiros 1 e p 1, com os p 3 restantes: 2, 3, ..., p 2 podemos formar pares (a, a), com a a tais que a.a p 3 essas congruncias, obtemos a congruncia: 2 2 . 3 . ... . (p 2) ou, multiplicando por (p 1): (p 1)! (p 1) (mod p) 1 (mod p)

p 3 2 1 (mod p). Ento, multiplicando ordenadamente todas

1 (mod p), ou (p 2)!

1 (mod p)

que a prpria congruncia de Wilson. Com p = 13, por exemplo, os 10 inteiros 2, 3,..., 11 do lugar a 5 pares tais que o produto dos inteiros de cada par congruente a 1 mdulo 13: 2.7 1 (mod 13) 4.10 1 (mod 13) 6.11 1 (mod 13) 3.9 1 (mod 13) 5.8 1 (mod 13) multiplicando ordenadamente essas cinco congruncias, obtemos: (2.7) . (3.9) . (4.10) . (5.8) . (6.11) e, portanto, 11! (mod 13) 1 (mod 13)

147

CAPTULO 10 FERMAT, WILSON E EULER

12! isto ,

12 (mod 13)

1 (mod 13)

(p 1)!

1 (mod p), com p = 13 1 (mod p), ento n primo.

Recproca do Teorema de Wilson: se (n 1)!

Demonstrao: a recproca verdadeira. Suponhamos, por absurdo, que o inteiro n composto. Ento, n tem um divisor d tal que 1 < d < n, de modo que d um dos fatores do produto: 1 . 2 . 3 . ... . (n 1) = (n 1)! e, portanto, d | (n 1)!. Mas, por hiptese n | (n 1)! + 1, e como d | n, segue-se que: d | (n 1)! + 1, e d | 1 o que absurdo, visto que d > 1. Logo, n no possui divisores menores que ele mesmo e diferentes de 1, ou seja, n primo. Nota: Este teorema recproco d um critrio para se reconhecer se um inteiro dado primo. Mas, do ponto de vista prtico, no momento ainda de interesse apenas terico, pois o clculo de fatoriais exigiria muito tempo. Teorema 10.5 (Teorema de Leibniz): um inteiro n > 1 primo, se e somente se, (n 2)! Demonstrao: ( ) suponhamos que o inteiro n > 1 primo. Ento, pelo teorema de Wilson: (n 1)! obviamente, (n 1)! = (n 1). (n 2)! portanto, (n 2)! ( ) reciprocamente, se (n 2)! (n 1)! 1 (mod n) 1 (mod n) (n 2)! (mod n) 1 (mod n) 1 (mod n)

1 (mod n), ento: (n 2)!

Logo, pelo recproco do Teorema de Wilson, o inteiro n primo.

148

CAPTULO 10 FERMAT, WILSON E EULER

EXERCCIOS

1) Verificar o Teorema de Wilson para p = 5 e para p = 7.

2) Mostrar que 11, 13, 17 e 19 so primos usando o Teorema de Wilson.

3) Mostrar que 8 composto usando o Teorema de Wilson.

4) Achar o resto da diviso de 15! por 17.

5) Mostrar que 18! + 1

0 (mod 437).

6) Sendo p um primo mpar, demonstrar que 2.(p 3)!

1 (mod p).

7) Verificar o Teorema de Leibniz com o primo p = 13.

8) Usando o Teorema de Leibniz, mostrar que 17 primo.

9) Formar com os inteiros 2, 3, 4, ... , 21, todos os pares de a e b tais que a.b

1 (mod 23).

149

CAPTULO 10 FERMAT, WILSON E EULER

10.3. TEOREMA DE EULER


Leonhard Euler (15/04/1707 18/07/1783) foi um matemtico e fsico de origem sua. Nasceu na Basilia, filho do pastor calvinista Paul Euler (l-se iler) que, desprezando seu prodigioso talento matemtico, determinou que ele estudasse Teologia e seguiria a carreira religiosa. Daniel e Nikolaus Bernoulli convenceram o pai de Euler a permitir que seu filho trocasse o hbito pelos nmeros. Euler, logo aps, deixou a Sua, indo para os palcios de Berlim e So Petersburgo, onde passou os mais criativos anos de sua vida. Os governos da Europa estavam interessados em usar a Matemtica para resolver problemas prticos e competiam entre si para empregar os melhores crebros. Durante sua vida resolveu enorme quantidade de problemas, da navegao s finanas, da acstica irrigao. A soluo de tais problemas, que atendiam aos reclamos do mundo prtico, no o entediava, principalmente porque cada novo trabalho inspirava-o para criar uma Matemtica nova e engenhosa. Era capaz de escrever vrios trabalhos em um nico dia com os clculos completos e prontos para serem publicados. Conseguiu provar o uma conjectura de Fermat relativa aos nmeros primos. Fermat afirmava que o primeiro tipo de nmero primo sempre era representado pela soma de nmeros ao quadrado enquanto que o segundo jamais o seria. Esta propriedade dos nmeros primos extremamente simples, porm, ao tentar provar que isto uma verdade para qualquer nmero primo, torna-se extremamente difcil. Em 1749, depois de sete anos de trabalho e quase cem anos aps a morte de Fermat, conseguiu apresentar esta prova. Ao final de vida estava completamente cego. Alis, j era cego de um olho desde os vinte anos, o que no o perturbara em nada. Certa vez disse: agora eu terei menos distraes. Aos sessenta anos foi acometido por catarata que o cegou completamente. Apesar disto, continuou a produzir Matemtica por mais sete anos. Seu imenso conhecimento permitia-lhe criar conceitos sem ter que coloc-los no papel e sua memria fenomenal permitia-lhe usar seu crebro como uma biblioteca mental. Passou os anos finais de sua vida na Rssia, ento sob a proteo de Catarina, a Grande. Definio 2: chama-se funo aritmtica toda funo f definida no conjunto dos naturais e com valores no conjunto dos inteiros, i.e., toda funo f de em (f : ) . Definio 3: uma funo aritmtica f se diz multiplicativa se f(r.s) = f(r) . f(s), para todo par de inteiros positivos r e s, tais que o MDC(r, s) = 1.

150

CAPTULO 10 FERMAT, WILSON E EULER

10.4. FUNO TOTIENT (n)


Definio 4: Chama-se Funo Totient a funo aritmtica assim definida para todo inteiro positivo n: (n) = quantidade de inteiros positivos menores que n relativamente primos a n. Em outros termos, (n) igual ao nmero de elementos do conjunto #{ x
|1

x < n, MDC(x, n) = 1 }

Observao: (1) = 1, pois MDC(1, 1) = 1. Exemplo 10.3: (30) = 8 e (12) = 4. n (n) 1 1 2 3 4 5 6 7 8 1 2 2 4 2 6 4 Tabela de (n) para os dez primeiros inteiros positivos. 9 6 10 4

Teorema 10.6: A Funo Totient uma funo aritmtica multiplicativa. Demonstrao: sejam r e s dois inteiros positivos tais que o MDC(r, s) = 1. Cumpre demonstrar que (r.s) = (r) . (s). A proposio verdadeira se r ou s igual a 1, pois, temos: Se r = 1, (r.s) = (1.s) = (s) = 1 . (s) = (1) . (s) = (r) . (s) Se s = 1, (r.s) = (r.1) = (r) = (r) . 1 = (r) . (1) = (r) . (s) Suponhamos, pois, que r > 1 e s >1. Neste caso, todos os inteiros de 1 a r.s podem ser dispostos em r colunas com s inteiros em cada uma delas, do seguinte modo: 1 2 r+1 r+2 2.r + 1 2.r + 2 ... ... (s 1).r + 1 (s 1).r + 2 ... h ... r+h ... 2.r + h ... ... ... (s 1).r + h ... r ... 2.r ... 3.r ... ... ... s.r

Como o MDC(q.r + h, r) = MDC(h, r), os inteiros da h-sima coluna so primos com r, se e somente se, h primo com r. Alm disto, como na primeira linha o nmero de inteiros que so primos com r igual a (r), segue que h somente (r) colunas formadas com inteiros que so todos os primos com r. Por outro lado, em cada uma destas (r) colunas existe precisamente (s) inteiros que so primos com s, porque na progresso aritmtica: h, r + h, 2.r + h, ..., (s 1).r + h

151

CAPTULO 10 FERMAT, WILSON E EULER

onde o MDC(r, h) = 1, o nmero de termos que so primos com s igual a (s). Assim sendo, o nmero total de inteiros que so primos com r e s, isto , que so primos a r.s, igual a (r). (s), e isto significa que (r.s) = (r) . (s).

10.5 CLCULO DE (n)


Teorema 10.7: seja p um primo, ento (p) = p 1. Demonstrao: ( ) Se n > 1 primo, ento cada um dos inteiros positivos menores que n primo com n e, portanto, (n) = n 1. ( ) Reciprocamente, se (n) = n 1, com n > 1, ento n primo, pois, se n fosse composto, teria pelo menos um divisor d tal que 1 < d < n, de modo que pelo menos dois dos inteiros 1, 2, 3,..., n no seriam primos com n, d e n, isto , (n) n 2. Logo, n primo. Teorema 10.8: se p primo e se k um inteiro positivo, ento (pk) = pk pk1 = pk . 1
1 p

Demonstrao: obviamente, o MDC(pk, n) = 1, se e somente se, p no divide n, e entre 1 e pk existem pk1 inteiros que no so primos com pk, que so todos os mltiplos de p: p, 2.p, 3.p, ..., p2, ..., p3, ..., pk segue-se que o conjunto {p, 2p, 3p, ..., p2, ..., p3, ..., pk} contm exatamente pk pk1 inteiros que so relativamente primos a pk, de modo que pela definio da funo (n) de Euler, temos: (pk) = pk pk1
k k Teorema 10.9: se n = p1 . pk . pk . ... . pr a decomposio cannica do inteiro n > 1, 2 3 ento:
1 2 3 r

k k (n) = p11 p11

. pk2 2

pk2 2

. ... . pkr r

pkr r

n. 1

1 1 1 . 1 . ... . 1 p1 p2 pr

ou seja,
r

(n) =

i 1

( piki

piki 1 )

n. (1
i 1

1 ) pi

152

CAPTULO 10 FERMAT, WILSON E EULER

Demonstrao: usaremos o Teorema da Induo Matemtica sobre r, nmero de fatores primos distintos de n. A proposio verdadeira para r = 1. Suponhamos, ento, a proposio verdadeira para r = i. k Como o MDC p11 . pk2 . ... . pkr , pkr+1 1, e (n) uma funo aritmtica multiplicativa, 2 r r+1 temos:
k k p11 . pk2 . ... . pr r 2 kr+1 . pr+1

k p11 . pk2 . ... . prkr 2

pkr+1 r+1

ou seja,
k k p11 . pk2 . ... . pr r 2 kr+1 . pr+1

k p11 . pk2 . ... . prkr 2

. pkr+1 r+1

pkr+1 r+1

ou, vista da hiptese de induo:


k p11 . pk2 . ... . pkr . pkr+1 2 r r+1 k p11 k p11 1

. pk2 2

pk2 2

. ... . pkr r

pkr r

. pkr+1 pkr+1 r+1 r+1

e isto significa que a proposio verdadeira para r = i + 1. Logo, a proposio verdadeira para todo inteiro positivo r. Teorema 10.10: Para todo inteiro positivo n > 2 , Demonstrao: Se n uma potncia de 2, isto , n 2k , com k (n) um inteiro par

2 , ento:

( n)
que um inteiro par.

(2k )

2k (1

1 ) 2

2k

Se, ao invs, n no uma potncia de 2, ento n divisvel por um primo impar p, isto : n pk m , onde k 1 e o mdc( pk , m) 1 E como (n) uma funo aritmtica multiplicativa temos:

(n)

( pk ) (m)

pk 1 ( p 1) (m)

que tambm um inteiro par, por que 2 | ( p 1) . Assim, (n) um inteiro mpar somente para n = 1 e n = 2:
(1) (2) 1

153

CAPTULO 10 FERMAT, WILSON E EULER

Teorema 10.11 (Teorema de Euler): Se n um inteiro positivo e se MDC(a, n) = 1, ento a


(n)

1 (mod n)

Provaremos, primeiramente, o seguinte lema: Sejam a e n > 1 inteiros tais que o MDC(a, n) = 1. Se a1, a2, ..., a (n) so inteiros positivos menores que n e que so relativamente primos com n, ento cada um dos inteiros a.a1, a.a2, ..., a.a (n) congruente mdulo n a um dos inteiros a1, a2, ..., a (n) (no necessariamente nesta ordem em que aparecem). Demonstrao: os inteiros a.a1, a.a2, ..., a.a (n) so mutuamente incongruentes mdulo n, pois, se a.ai a.aj (mod n), com 1 i j (n), como o MDC(a, n) = 1, podemos cancelar o fator comum a, o que d ai aj (mod n) n | (aj ai). Isto impossvel, visto que (aj ai) < n. Por outro lado, como o MDC(ai, n) = i, i = 1, 2, ..., (n) e o MDC(a, n) = 1, segue que o MDC(a.ai, n) = 1. Mas, pelo algoritmo da diviso, a.ai = n.qi + ri, 0 ri < n, que implica em a.ai ri (mod n), com 0 ri < n

portanto, MDC(ri, n) = MDC(a.ai, n) = 1, de modo que ri um dos inteiros a1, a2, ..., a (n), isto , cada um dos inteiros a.a1, a.a2, ..., a.a (n) congruente mdulo n a um nico dos inteiros a1, a2, ..., a (n), em uma certa ordem. Provemos, agora, o Teorema de Euler: A proposio verdadeira para n = 1, pois a (1) 1 (mod 1). Suponhamos, pois, n > 1, e sejam a1, a2, ..., a (n) os inteiros positivos menores que n e relativamente primos a n. Como o MDC(a, n) = 1, ento, pelo Lema acima, os inteiros a.a1, a.a2, ..., a.a (n) so congruentes mdulo n aos inteiros a1, a2, ..., a (n), em uma certa ordem: a.a1 onde a1*, a2*, ..., a
* (n)

a1*, a.a2

a2*, ..., a.a


(n)

(n)

* (n)

denotam os inteiros a1, a2, ..., a

em uma certa ordem.

Multiplicando ordenadamente todas essas (n) congruncias, obtemos: (a.a1).(a.a2). ... .(a. a ou seja, a
(n) (n))

a1* . a2* . ... . a

* (n)

(mod n)

. (a1 . a2 . ... . a

(n))

a1 . a2 . ... . a

(n)

(mod n)

Cada um dos inteiros a1, a2, ..., a (n) relativamente primo a n, de modo que podem ser sucessivamente cancelados, o que d a congruncia de Euler: a
(n)

1 (mod n)

Nota: se p um primo, (p) = p 1, e se o MDC(a, p) = 1, ento: a


(p)

ap1 (mod p)

1 (mod p)

154

CAPTULO 10 FERMAT, WILSON E EULER

que a congruncia de Fermat. Assim, o Teorema de Euler uma generalizao do teorema de Fermat. Corolrio 3: Se m > 1, k 0, n 0 e a um inteiro qualquer so tais que, MDC(a, m) = 1 e k n (mod (m)) ento, ak an (mod m). Demonstrao: basta considerar o caso em que k > n. Como k tal que k n = q . (m) e, portanto, ak = akn . an = aq.
(n)

n (mod (m)) existe q

. an = (a

(n) q

) . an

an (mod m)

Exemplo 10.4: sejam a = 5, m = 6, k = 8 e n = 2. Temos (6) = 2, e 8 2 (mod 2). Como 52 1(mod 6), ento 58 1 (mod 6) e desta forma, 58 52 (mod 6).

10.6. RESOLUO DE CONGRUNCIAS LINEARES PELO TEOREMA DE EULER


A congruncia linear a.x b (mod m) no caso em que o MDC(a, m) = 1, admite uma nica soluo mdulo m, que se pode facilmente obter usando o Teorema de Euler. Realmente, temos: ak = akn . an = aq. portanto, a.x b.a
(m) (n)

. an = (a

(n) q

) . an

an (mod m)

(mod m)

Como o MDC(a,m) = 1, podemos cancelar o fator comum a, que resulta em: x b.a
(m) 1

(mod m) 5 (mod 8), onde o MDC(3, 8) = 1, temos: 135 (mod 8) 7 (mod 8)

Exemplo 10.5: no caso da congruncia linear 3.x x 5. 3


(8)1

(mod 8)

5.341 (mod 8)

5.27 (mod 8)
(n)

Em particular, a.x 1 (mod n) x a (mod n) determina um inverso de a mdulo n. Exemplo 10.6: queremos determinar o inverso de 7 mdulo 11, ou seja, queremos resolver a congruncia linear 7.x 1 (mod 11). Queremos determinar o menor inteiro positivo em 11 que satisfaa a equao x 7
(11) 1

(mod 11)

7101 (mod 11)

79 (mod 11)

8 (mod 11)

assim, temos que x = 8 a menor soluo positiva em 11 para o problema.

155

CAPTULO 10 FERMAT, WILSON E EULER

10. 7. RESOLUO DA EQUAO (n) = m.

Vejamos agora como resolver a equao (n) = m, quando m um inteiro positivo dado. Sabemos que no existe soluo se m um mpar maior que 1. Em geral no se conhece uma frmula para se resolver essa equao, quando m qualquer inteiro positivo par. Mesmo assim, existe um mtodo com o qual podemos determinar todas as solues dessa equao.
r

Seja n
i=1

piki um inteiro qualquer, decomposto em fatores primos, que satisfaa a equao


r

(n) = m. Como
r

(n) =

i 1 r

( piki

piki 1 ) , ento 1) m.

(n) =

i 1

( piki

piki 1 )

i 1

piki 1( pi

Faamos

di

pi 1 , i 1, 2,..., r

Assim,
r i 1
r i 1

piki 1d i
piki di pi

m,
m

n.

di pi

m,
r

m
r i 1

di

i 1

pi

Todas essas equaes estabelecem trs condies para os d i , que nos permitem determinar os valores de n que satisfazem a equao (n) = m: 1) cada di 1 um primo; 2) cada d i um divisor positivo de m;

156

CAPTULO 10 FERMAT, WILSON E EULER

m
3)
r i 1

um inteiro positivo que contm somente fatores primos includos em

di
x 24 23.3

i 1

pi .

Exemplo 10.7: Resolver a equao

Os divisores positivos de 24 so 1,2,3,4,6,8,12 e 24. Sem dvida, os possveis d i so os inteiros 1,2,4,6 e 12 j que eles so os nicos tais que di 1 um primo. Como todos os produtos de quatro ou cinco dos d i so maiores que 24,podemos eliminar esses produtos 24 imediatamente; quer dizer, k no e um inteiro quando quatro o mais d i so di
i 1
k

considerados. As possveis restantes expresses para


i 1

d i so os seguintes produtos abaixo:

1, 2 , 4, 6 , 12 , 1.2 , 1.4, 2.4

2.6 , 2. 12 , 4.6 , 4.12 , 6.12 ,1.2.4, 1.2.6,

1.2.12, 1.4.6 , 1.4.12 ,1.6.12 , 2.4.6 ,

2.4.12, 2.6.12 , e 4.6.12.

Os oito ltimos produtos destacados, podem ser eliminados, j que em cada caso

24
k

di
i 1

no um inteiro ; os sete primeiros produtos destacados, podem ser eliminados ,j que em cada caso
24
k
k k

, contm fatores primos no contidos em


i 1

(di 1) , isto ,
i 1

pi .

di
i 1

157

CAPTULO 10 FERMAT, WILSON E EULER


k

Agora, os produtos restantes para


i 1

d i conduzem as solues:
24
k

24

di
i 1

di
i 1

pi
i 1

.
i 1

pi

di
i 1

x
72 56 52 45 39 35 90 84 78 70

1.2 1.6 1.12 2.4 2.12 4.6 1.2.4 1.2.6 1.2.12 1.4.6

2 .3 22 2 3 1 1 3 2 1 1

2.3 2.7 2.13 3.5 3.13 5.7 2.3.5 2.3.7 2.3.13 25.7

23.32 23.7 2 2 .13 32.5 3.13 5.7 2.32.5 22.3.7 2.3.13 2.5.7

Em conseqncia, as solues da equao ( x) 24 so 35,39,45,52,56,70,72,78,84 e 90. Exemplo 10.8: Usando o fato de que a funo Totient multiplicativa, encontrar as 10 solues da equao ( x) 24 . Consideremos ( x) (n1 ). (n2 ). (n3 )... 24 ,onde n1 , n2 , n3 so relativamente primos entre si dois a dois. Investigando uma lista de valores de (n) , podemos determinar conjuntos de fatores (n1 ), (n2 ), (n3 ) ,... onde n1 , n2 , n3 ,... so relativamente primos dois a dois cujos produtos so iguais a 24. Os produtos n1.n2 .n3... representam solues da equao

24. Como
Produto (3) . (4) . (5) . (5) . (6) . (7) . (7) . (7) . (8) . (9) . (13) = 2.12 (13) = 2.12 (7) = 4.6 (9) = 4.6 (13) = 2.12 (8) = 6.4 (10) = 6.4 (12) = 6.4 (9) = 4.6 (10) = 6.4 mdc (3,13) = 1, (4,13) = 1, (5,7) = 1, (5,9) = 1, (6,13) = 1, (7,8) = 1, (7,10) = 1, (7,12) = 1, (8,9) = 1, (9,10) = 1, Soluo x = 39; x = 52; x = 35; x = 45; x = 78; x = 56; x = 70; x = 84; x = 72; x = 90;

158

CAPTULO 10 FERMAT, WILSON E EULER

Assim, as dez solues da equao x 24 so 35, 39, 45, 52, 56, 70, 72, 78, 84 e 90. Este resultado esta de acordo com exemplo anterior. Note que para usar este mtodo e necessrio conhecer primeiramente o nmero de solues desejadas ou limitar os valores das solues desejadas com o fim de decidir quando termina a investigao. Kevin Ford provou em 1999 que para todo nmero inteiro k 2 h um nmero m para o qual a equao ( x) m tem exatamente k solues; este resultado j haviam sido conjecturado por Sierpiski. No entanto, nenhum desses m conhecido para k=1, e de acordo com a conjectura da funo totiente de Carmichael acredita-se que neste caso esse m no existe.

10.8 VALNCIA DA FUNO TOTIENTE: N (m) .

A funo N (m) , chamada de Valncia da Funo Totiente, definida como o nmero de inteiros positivos k, tal que (k) = m, tambm chamada de multiplicidade de m.

Exemplo 1: Temos que N (8) 5 , porque existem apenas 5 inteiros, k = 15, 16, 20, 24 e 30, tal que (k) = 8. A tabela abaixo mostra os valores de N (m) para n m 1 2 4 6 8 10 12
N (m)

12.

k, tal que (k) = m 1, 2 3, 4, 6 5, 8, 10, 12 7, 9, 14, 18 15, 16, 20, 24, 30 11, 22 13, 21, 26, 28, 36, 42

2 3 4 4 5 2 6

Pode-se provar que existem inteiros pares m > 1 que no so valores assumidos pela funo Totiente, como por exemplo N (14) 0 . Andrzej Schinzel provou em 1956, que para todo
t 1 , o valor 2. 7t no valor da funo Totiente. Veja que N (26)

0 e, 26 no da

forma 2. 7t . Mas, em 1976, Nathan Mendelsohn provou a existncia de uma infinidade de nmeros primos p, tais que para todo t 1 , o valor 2t. p no assumido pela funo Totiente e, 26 um nmero dessa forma.

Observe os seguintes valores da funo Totiente:

159

CAPTULO 10 FERMAT, WILSON E EULER

m 2 4 9

(m) 1! 2! 3!

35 4! 231 5! Himanshu Gupta provou em 1950, que para todo t 1 existe m tal que (m) = t!

EXERCCIOS
1. 2. Calcular (420), (1001), (5040) e (8316). Verificar que (n + 2) = (n) + 2, para n = 12, 14, 20. Verificar que (n) = (n + 1) = (n + 2), para n = 5186. Verificar que (3k . 568) = (3k . 638). Verificar que (n) uma funo aritmtica multiplicativa para n = 144. Verificar o Teorema de Euler com a = 3 e n = 10; a = 7 e n = 12. Verifique que: 11. Usando o Teorema de Euler, resolver as seguintes congruncias lineares: (a) 5x (b) 2x (c) 7x (d) 8x (e) 2x (f) 5x 7 (mod 12) 3 (mod 9) 1 (mod 10) 4 (mod 5) 1 (mod 17) 3 (mod 8)

3.

4. 5.

12. Demonstrar que se n um inteiro positivo mpar, ento:

6.

a) b)

(2.n) = (n) (4n) = 2. (n)

7.

11357 59(mod96) 31304 b) 6 84(mod101) 4205 c) 23 67(mod 77) 12101 d) 20 20(mod57) 850 e) 17 100(mod143)
a) 8. Achar os dois ltimos algarismos da direita do inteiro 3256. 9. Calcular o menor inteiro positivo n tal que n 71015 (mod 31). 10. Achar o algarismo das unidades do inteiro 3145.

13. Resolva em as seguintes equaes: a) b) c) d) e) (n) = 12 (n) = 18 (n) = 20 (n) = 30 (n) = 4!

160

CAPTULO 10 FERMAT, WILSON E EULER

10.9. TEOREMA CHINS DO RESTO (TCR)


O livro Manual Aritmtico do Mestre Sol foi escrito por Sun Zi Suanjing (ou Sun Tzu Suan Ching), provavelmente entre 280 d.C. a 483 d.C. O livro est dividido em 3 captulos. O 1 captulo, contm apenas dois problemas que dizem respeito sobretudo a mtodos para fazer multiplicaes e divises, utilizando palitinhos chineses. O segundo captulo, contm 28 problemas, apresenta mtodos para o clculo com fraes, extrao da raiz quadrada, determinao de reas e volumes, propores e regra de trs simples. O terceiro captulo contm 36 problemas aritmticos. No problema 26 (tambm conhecido como problema do Mestre Sun) do 3 captulo, Sun Tzu utiliza pela primeira vez o chamado Teorema Chins do Resto (TCR). O problema est enunciado abaixo: Temos coisas, mas no sabemos quantas; se as contarmos de trs em trs, o resto 2; se as contarmos de cinco em cinco, o resto 3; se as contarmos de sete em sete, o resto 2. Quantas coisas temos? Em notao moderna, este problema equivale a procurar as solues do seguinte sistema de congruncias:
x x x 2 (mod 3) 3 (mod 5) 2 (mod 7)

Resolver problemas como esse um dos objetivos deste tpico. Teorema 10.11 (Teorema Chins do Resto TCR): sejam m1, m2, ..., mk inteiros positivos primos entre si dois a dois, isto , tais que o MDC(mi, mj) = 1 se i j. Nestas condies, o sistema de congruncias lineares:
x x ... x ak (mod mk ) a1 (mod m1 ) a2 (mod m2 )

tem nica soluo mdulo m = m1, m2, ..., mk , dada por: x a1.M1.x1 + a2.M2.x2 + ... + ak.Mk.xk (mod m) Demonstrao: para cada k = 1, 2, 3, ..., r, seja: Seja Mk
m mk m1 . m2 . ... . mr mk

. Como os inteiros mi so todos primos entre si dois a dois, o 1 (mod Mr) tem nica

MDC(Mr, mr) = 1, de modo que a congruncia linear Mr . x soluo x xr (mod mr) Posto isto, vamos mostrar que o inteiro x soluo do sistema considerado.

a1.M1.x1 + a2.M2.x2 + ... + ak.Mk.xk (mod m) uma

161

CAPTULO 10 FERMAT, WILSON E EULER

Com efeito se i

r, ento mr | Mi e Mi x

0 (mod mr), que implica em:

a1.M1.x1 + a2.M2.x2 + ... + ak.Mk.xk (mod m)

Para demonstrar a unicidade desta soluo, suponhamos que x1 uma outra soluo qualquer do sistema considerado. Ento: x ar (mod mr) x1 (mod mr), r = 1, 2, ..., k.

e, portanto, mr | (x x1), r = 1, 2, ..., k. Mas, o MDC(mi, mj) = 1 implica em (m1 . m2 . . mk) | (x x1), isto , m | (x x1) e x (mod m), com o que termina a demonstrao do TCR. x1

Teorema 10.12: Sejam m1, m2, ..., mk inteiros positivos primos entre si dois a dois, e sejam a1, a2, ..., ak inteiros tais que MDC(ar, mr) = 1 para r = 1, 2, ..., k. Nestas condies, o sistema de congruncias lineares:
a1.x a2 .x ... ak .x bk (mod mk ) b1 (mod m1 ) b2 (mod m2 )

tem nica soluo mdulo m = m1 . m2 . ... . mk. Demonstrao: como o MDC(ar, mr) = 1, a congruncia linear ar.x soluo x ar (mod mr), de modo que: ar.ar 1 (mod mr), e ar.ar.x x (mod mr) 1 (mod mr) tem nica

e, portanto, equivalente a congruncia x

ar . br (mod mr).

Assim sendo, o sistema considerado equivalente ao seguinte sistema de congruncias lineares:


x x ... x ak .bk (mod mk ) a1.b1 (mod m1 ) a2 .b2 (mod m2 )

o qual tem, pelo TCR, uma nica soluo mdulo m = m1 . m2 . ... . mk: xk . Mk 1 (mod mk) xk Mk (m) 1 (mod mk ) onde:

162

CAPTULO 10 FERMAT, WILSON E EULER

(i) Mk

m , k = 1, 2, 3, ... mk

(ii) xk . Mk mk .

1 (mod mk)

xk

Mk (m) 1 (mod mk ) , ou seja, xk o inverso de Mk mdulo

Exemplo 10.7: utilizando o TCR, resolver o sistema de congruncias lineares:


x x x x 8 (mod 5) 5 (mod 3) 11 (mod 7) 2 (mod 4)

Resoluo: os mdulos 5, 3, 7 e 4 das congruncias lineares que formam o sistema so primos entre si dois a dois, de modo que pelo TCR este sistema tem uma nica soluo mdulo m =m1 . m2 . m3 . m4 = 5 . 3 . 7 . 4 = 420. Temos ento:
M1 m m1 420 5 84 , M2 m m2 420 3 140 , M3 m m3 420 7 60 , M4 m m4 420 4 105

Os inversos xk dos Mk so dados por: 84 x1 140 x2 60 x3 105 x4 1 (mod 5) 1 (mod 3) 1 (mod 7) 1 (mod 4)

Aplicando o mtodo de Euler nas equaes acima obtemos as solues respectivas: x1 = 4, x2 = 2, x3 = 2, e x4 = 1. Portanto, temos : x x x a1.M1.x1 + a2.M2.x2 + ... + ak.Mk.xk (mod m) 8 . 84 . 4 + 5 . 140 . 2 + 11 . 60 . 2 + 2 . 105 . 1 (mod 420) 5618 (mod 420) 158 (mod.420),

segue-se que x = 158 a menor soluo positiva mdulo 420, do sistema de congruncias lineares dado. Qualquer outra soluo da forma: x 158 (mod 420) x = 158 + 420.k, k
.

Exemplo 10.8:
5.x 3.x 11.x 11 mod 17 19 mod 32 6 mod 37

163

CAPTULO 10 FERMAT, WILSON E EULER

Resoluo: Como o MDC(17, 32) = MDC(17, 37) = MDC(32, 37) = 1, o sistema possui soluo. De 5.x 11 (mod 17), obtemos x 9 (mod 17). De 3.x 19 (mod 32), obtemos x 17 (mod 32) De 11.x 6 (mod 37), obtemos x 14 (mod 37) assim temos o seguinte sistema:
x 9 (mod 17) x 17(mod 32) x 14 (mod 37

Usando o TCR: a1 = 9, a2 = 17, a3 = 14. m1 = 17, m2 = 32, m3 = 37. m = m1 . m2 . m3 = 17 . 32 . 37 = 20128. m 20128 m 20128 629 , M3 M1 1184 , M2
m1 17

m2

32

m m3

20128 37

544

Ento 1184.b1 1 (mod 17), 629.b2 1 (mod 32) e 544.b3 1 (mod 37). De onde conclumos: b1 = 14 , b2 = 29 e b3 = 10. A soluo geral ser: x x 9 . 14 . 1184 + 17 . 29 . 629 + 14 . 10 . 544 (mod 20128) 535441 12113 (mod 20128)

EXERCCIOS 1. Resolver os seguintes sistemas de congruncias lineares utilizando o TCR:

x 3 (mod 5)
a) x 5 (mod 7) x 7 (mod11)

x 1 (mod 5)
b) x 5 (mod 7) x 7 (mod11)

x 5 (mod 6)
c) x 4 (mod11) x 3 (mod17)

x 5 (mod11)
d) x 14 (mod 29) x 15 (mod 31)

7 (mod 9)

28 (mod 29)

x
g) x x

a (mod 3) b (mod 5) c (mod 8)

2 x 1 (mod 5)
h) 4 x 1 (mod 7) 5 x 9 (mod11)

e) x 10 (mod 4) x 1 (mod 7)

f) x 30 (mod 31) x 10 (mod11)

164

CAPTULO 10 FERMAT, WILSON E EULER

10.10. POTENCIAO: UMA APLICAO DO TEOREMA DE EULER


Seja m um inteiro positivo tal que mdc(a, m) = 1 . Ento: a
(m)

1 (mod m)

Aplicamos este resultado ao clculo de potncia mdulo m. De uma maneira geral o problema o seguinte. So dados a, k e m trs inteiros positivos, dos quais sabemos que m co-primo com a. Digamos que k muito grande (o caso difcil) e queremos achar a forma reduzida de ak (mod m). Podemos simplificar as contas usando o teorema de Euler. Estamos supondo que k grande, na prtica precisamos saber apenas que k (m) . Dividindo k por (m) , obtemos k = (m) .q + r, onde o resto r satisfaz 0 r (m) 1. Temos, ento, que: ak

(m).q+r

(mod m)

(a

(m) q

) . ar (mod m)

Mas pelo teorema de Euler, a (m) 1 (mod m) . Obtemos, portanto, da equao acima que ak ar (mod p). Um exemplo numrico para convenc-lo das vantagens deste resultado to simples. Queremos calcular 25432675 (mod 13). Da maneira como vnhamos procedendo teramos que efetuar uma quantidade enorme de potenciaes mdulo 13. Usando a idia acima, obtemos o resto da diviso de k = 5432675 por (13), que r = 11, e assim: 25432675 Logo, 25432675 (2
(13) q

) . 211 (mod 13)

211 (mod 13)

7 (mod 13)

7 (mod 13).

10.11 POTENCIAO: UMA APLICAO DO TEOREMA CHINS DO RESTO (TCR)


Podemos aplicar o TCR para simplificar o clculo de potncias mdulo n em alguns casos especiais. Suponhamos que n = p1 . p2 . ... . pk, onde p1 < p2 < ... < pk so nmeros primos. Assim, estamos supondo que, na fatorao de n, cada fator primo aparece com multiplicidade 1. Neste caso fica muito fcil calcular a forma reduzida de am (mod n). Em primeiro lugar, usamos o PTF para achar a forma reduzida de am mdulo cada um dos primos p1, p2, ..., pk separadamente. Digamos que: am am am r1 (mod p1) e 0 r2 (mod p2) e 0 rk (mod pk) e 0 r1 < p1 r1 < p2 rk < pk

165

CAPTULO 10 FERMAT, WILSON E EULER

Para achar a forma reduzida de am (mod n), basta resolver o sistema: x x x r1 (mod p1) r2 (mod p2) ... rk (mod pk)

Observe que este sistema sempre tem soluo, j que os mdulos so primos distintos e, portanto, o MDC entre dois quaisquer entre eles sempre igual a 1. Alm disto, o teorema nos garante que o sistema tem uma nica soluo r (mod n), onde n = p1 . p2 . ... . pk. Logo am r (mod n), e obtivemos o que queramos. Vejamos um exemplo numrico. Digamos que queremos calcular a forma reduzida de 26754 (mod 1155) Fatorando 1155 vemos que 1155 = 3 . 5 . 7 . 11, todos primos entre si com multiplicidade 1. Aplicando o PTF a cada um destes primos, temos que: 26754 26754 26754 26754 Assim: 6754 = (3).q1 + r1 6754 = (5).q2 + r2 6754 = (7).q3 + r3 6754 = (11).q4 + r4 6754 = 2 . 3377 + 0, logo 20 6754 = 4 . 1688 + 2, logo 22 6754 = 6 . 1125 + 4, logo 24 6754 = 10 . 675 + 4, logo 24 1 (mod 3). 4 (mod 5). 2 (mod 7). 5 (mod 11).

2r1 2r2 2r3 2r4

(mod 3) (mod 5) (mod 7) (mod 11)

Precisamos, portanto, resolver o sistema: x x x x Soluo: m = m1 . m2 . m3 . m4 = 3 . 5 . 7 . 11 = 1155


M1 m m1 1155 3 385 ; M2 m m2 1155 5 231 ; M3
m m3 1155 7 165 ;

1 (mod 3) 4 (mod 5) 2 (mod 7) 5 (mod 11)

M4

m m4

1155 11

105

166

CAPTULO 10 FERMAT, WILSON E EULER

M1.x1 M2.x2 M3.x3 M4.x4

1 (mod m1) 1 (mod m2) 1 (mod m3) 1 (mod m4)

385.x1 231.x2 165.x3 105.x4

1 (mod 3) 1 (mod 5) 1 (mod 7) 1 (mod 11)

x1 385 (3)1 (mod 3) 1 (mod 3) x2 231 (5)1 (mod 5) 1 (mod 5) x3 165 (7)1 (mod 7) 2 (mod 7) x4 105 (11)1 (mod 11) 2 (mod 11)

Assim, temos: x x x a1.M1.x1 + a2.M2.x2 + a3.M3.x3 + a4.M4.x4 (mod m) 1 . 385 . 1 + 4 . 231 . 1 + 2. 165 . 2 + 5 . 105 . 2 (mod 1155) 3019 (mod 1155) 709 (mod 1155)

EXERCCIOS
1) Construa as tabelas de operaes em

8 , 9 ,

10 , 13 e 17 .
2) Encontre os dois nmeros primos, cujos produtos, geraram os nmeros abaixo, utilizando o mtodo da Fatorao de Fermat: a) 437623 b) 919199 c) 9797 d) 4061 e) 19109 3) Calcule a potncias utilizando o mtodo da exponenciao rpida. a) 33268 (mod 335) b) 1774096 (mod 277) c) 818192 (mod 92) d) 482048 (mod 20) 4) Calcule as potncias utilizando o Pequeno Teorema de Fermat a) 52349899 (mod 17) b) 4511223311 (mod 19) c) 1001002003007 (mod 281) 5) Calcule as potncias utilizando o Pequeno Teorema de Fermat e o Algoritmo Chins do Resto. a) 4753 (mod 437623) b) 51773 (mod 919199) c) 8397 (mod 2926) d) 313961 (mod 12369)

167

CAPTULO 10 FERMAT, WILSON E EULER

LEITURA COMPLEMENTAR: PSEUDOPRIMOS


De acordo com o teorema de Fermat, se p primo e a um inteiro qualquer no divisvel por p, ento ap a (mod p). Imagine ento a seguinte situao: temos um nmero mpar n, e desejamos saber se , ou no, composto. Digamos que descobrimos um inteiro b tal que bn b (mod n). Pergunta-se: n pode ser primo? A resposta no, porque isto viola o teorema de Fermat. Logo, isto nos d uma maneira indireta de verificar se um nmero composto. Observe que, na prtica, s precisamos considerar os inteiros b no intervalo 1 < b < n 1. Por qu? Em primeiro lugar, como estamos trabalhando mdulo n, qualquer inteiro congruente a um inteiro no intervalo de 0 a n 1. Alm disso, a equao bn b (mod n) sempre satisfeita quando b 0, 1 ou n 1. Este teste produz uma situao surpreendente. Com ele podemos chegar concluso de que um nmero composto mesmo que no nos seja possvel determinar seus fatores. Antes de fazer um exemplo, conveniente formularmos um teste de uma maneira mais fcil de utilizar na prtica. Para isto usaremos a segunda verso do teorema de Fermat. Eis o teste: se n > 0 e 1 < b < n 1 so nmeros inteiros, e bn1 1 (mod n), ento n um nmero composto. O nmero b conhecido como uma testemunha de fato de n ser composto. Ser que podemos inverter o teorema de Fermat para verificar se um nmero primo? Na verdade, estamos perguntando se um nmero mpar n que satisfaz bn1 1 (mod n), para algum 1 < b < n 1, primo? Leibniz, famoso pela inveno do clculo (quase ao mesmo tempo que Newton), achava que sim, e usou isto como um critrio de primalidade. Na verdade, ele tomava apenas b = 2, que o caso mais simples de calcular. Infelizmente isto no verdade. Por exemplo, 2340 1 (mod 341). Logo, segundo Leibniz, 341 seria um nmero primo. Mas, 341 = 11 . 31, composto. Estes falsos primos so conhecidos como pseudoprimos. Isto , um inteiro positivo n, mpar e composto, um pseudoprimo para a base b (onde 1 < b < n 1) se bn 1 1 (mod n). Assim, 341 um pseudoprimo para a base 2. Apesar de s vezes dar errado, o teste de Leibniz muito til. Pelo menos para nmeros pequenos, ele acerta mais do que erra. Por exemplo, entre 1 e 109 existem 50.847.534 primos, mas apenas 5597 pseudoprimos para a base 2. Logo um nmero menor que um bilho que passa no teste de Leibniz apenas com a base 2 tem uma alta probabilidade de ser primo. Alm disto, usamos apenas a base 2 at agora. Por que no testar para mais de uma base? Fazendo isto o teste fica ainda mais eficiente. Por exemplo, 3340 56 (mod 341). Logo 3 uma testemunha de fato de que 341 composto. Na verdade, h apenas 1272 pseudoprimos simultneos para as bases 2 e 3 no intervalo entre 1 e 109. Como estamos limitando nossas bases ao intervalo entre 1 e n 1, podemos considerar a possibilidade de testar se n pseudoprimo para todas estas bases. Se n grande, isto vai ser impossvel, logo o interesse prtico muito limitado. Por outro lado, este problema leva a

168

CAPTULO 10 FERMAT, WILSON E EULER

algumas questes surpreendentes, e de real interesse prtico, como veremos em seguida com os nmeros de Carmichael. Nmeros de Carmichael Primeiramente, no existem nmeros que sejam pseudoprimos para todas as bases. Isto fcil de constatar. Se n composto, ento tem um fator b, ou seja, MDC(b ,n) 1. Como tambm o MDC(bn 1, n) 1, temos pelo teorema de inverso que b no pode ser inversvel mdulo n. Em particular, bn 1 1 (mod n). Logo n no pseudoprimo para a base b. Entretanto, pode ocorrer que um nmero composto n seja pseudoprimo para todas as bases que so primas a n. este tipo de nmero que desejamos discutir. prefervel reformular o problema da seguinte maneira: seja n um nmero inteiro positivo, como caracterizar os nmeros compostos n que satisfazem equao bn b (mod n)? A vantagem desta formulao que podemos dispensar a condio MDC(b, n) = 1, o que muito conveniente. O primeiro matemtico a dar exemplos destes nmeros foi R. D. Carmichael, em um artigo publicado em 1912. Por isso so chamados nmeros de Carmichael. Portanto, um nmero composto mpar n > 0 um nmero de Carmichael se bn b (mod n) para todo 1 < b < n 1. Observe que, para um nmero ser de Carmichael preciso que seja composto. Um nmero primo p tambm satisfaz equao bp b (mod p), mas no um nmero de Carmichael. Como o prprio Carmichael determinou, o menor nmero de Carmichael 561. Em princpio podemos verificar isto usando a definio. Entretanto, mesmo para um nmero relativamente pequeno como este, isto difcil de fazer usando apenas lpis e papel. Afinal, para mostrar que 561 um nmero de Carmichael pela definio, precisamos mostrar que b561 b (mod 561) para b = 2, 3, 4, 5, ..., 558, 559; o que d um total de 558 bases a serem testadas, algumas no to pequenas. No h duvida de que esta uma tarefa para um computador. Entretanto, mesmo um computador razovel pode ter dificuldade em usar este mtodo para verificar que, por exemplo: 349407515342287435050603204719587201 um nmero de Carmichael. Felizmente h uma maneira mais simples de verificar que um dado nmero composto de Carmichael. Consideremos 561 mais uma vez. Podemos facilmente fator-lo: 561 = 3 . 11 . 17 Seja agora b um nmero inteiro entre 2 e 559. Queremos mostrar que b561 b (mod 561).

Nossa estratgia ser a seguinte: mostraremos que b561 b divisvel por 3, 11 e 17 para todos os valores de b entre 2 e 559. Como so primos distintos, segue que o produto destes primos divide b561 b. Mas este produto 561, e dizer que 561 divide b561 b equivalente a dizer que b561 b (mod 561).

169

CAPTULO 10 FERMAT, WILSON E EULER

S nos resta mostrar que b561 b divisvel por cada um dos fatores de 561, separadamente. Como estes fatores so primos, o teorema de Fermat vem em nossa ajuda. Vamos efetuar as contas para o primo 17 e deixar 3 e 11 como exerccio. Queremos verificar que b 561 b divisvel por 17, isto , b561 b (mod 17). H dois casos a considerar. Se 17 divide b ento ambos b e b561 so membros de b congruentes a zero modulo 17, logo a congruncia imediatamente verificada. Mas, digamos que 17 no divide b. Neste caso, o teorema de Fermat diz que b16 1 (mod 17). Para aplicar isto, precisamos dividir 561 por (17 1), mas isto resulta em 561 = 35 . 16 + 1. Assim, b561 (b16)35 . b (mod 17) (1)35 . b (mod 17) b (mod 17). Alguns nmeros de Carmichael (existem infinitos): 561, 1105, 1729, 2465, 2821, 6601, 8911, 10585, 15841, 29341, 41041, 46657, 52633, 62745, 63973, 75361, 101101, 115921, 126217, 162401, 172081, 188461, 252601, 278545, 294409, 314821, 334153, 340561, 399001, 410041, 449065, 488881, 512461, 530881, 552721, 656601, 658801, 670033, 748657, 825625, 838201, 852841, 997633. Provaremos que, se t tal que (6.t + 1), (12.t + 1) e (18.t + 1) so todos primos, ento o seu produto um nmero de Carmichael. No se sabe ainda (2005), se existe uma infinidade de tais nmeros t. Demonstrao: temos que n = (6t + 1).(12t + 1).(18t + 1) composto. E que (n 1) = 1296.t3 + 396.t2 + 36.t 0 (mod 36) Seja o MDC(a, n) = 1. Ento, a relativamente primo com (6.t + 1), (12.t + 1) e (18.t + 1). Pelo PTF: a6.t 1 (mod 6.t + 1) a12.t 1 (mod 12.t + 1) a18.t 1 (mod 18.t + 1) a36.t a36.t a36.t 1 (mod 6.t + 1) 1 (mod 12.t + 1) 1 (mod 18.t + 1) 1 (mod n).

Logo, a36.t 1 (mod n). Como 36.t divide (n 1), temos que an1 Como n composto, n um nmero de Carmichael.

170

Captulo 11

CIFRA DE CSAR
INTRODUO CRIPTOGRAFIA A palavra Criptografia composta por dois termos gregos kryptos (kryptos secreto,

escondido, oculto) e grapho (grapho - escrita grafia). A criptografia uma arte ou cincia de escrever ocultamente talvez to antiga quanto a prpria escrita, hoje em dia um dos mtodos mais eficientes de se transferir informao, sem que haja a possibilidade de interferncia por parte de terceiros. o estudo de tcnicas matemticas, relacionadas com os aspectos de segurana e confidencialidade de informao, a integridade de dados, a autenticao de entidades e a autenticidade de origem de dados, ou seja, consiste na converso de dados num cdigo secreto como medida de segurana para que possam existir comunicaes seguras. A criptografia lida de um modo muito estreito com termos como - encriptao e desencriptao. A encriptao a converso de dados para uma forma que no ser compreendida facilmente por pessoas autorizadas com o objectivo de assegurar a privacidade mantendo a informao escondida e ilegvel mesmo para quem v os dados encriptados. A desencriptao o processo de converter dados encriptados de volta sua forma original, para que a mensagem possa ser compreendida e para isso acontecer requer alguma informao secreta, usualmente denominada chave de desencriptao. A chave de desencriptao o algoritmo que desfaz o trabalho do algoritmo de encriptao. Um algoritmo um programa de computador que pode ser visto como um algoritmo elaborado. baseada em chaves, uma informao pode ser codificada atravs de algum algoritmo de criptografia, de modo que, tendo conhecimento do algoritmo e da chave utilizados, possvel recuperar a informao original fazendo o percurso contrrio da encriptao, a desencriptao.

171

CAPTULO 11 CIFRA DE CSAR

Embora os cdigos secretos remontem aos primrdios da comunicao escrita, tem havido um aumento recente de interesse no assunto devido necessidade de manter a privacidade da informao transmitida ao longo de linhas pblicas de comunicao. Na linguagem da criptografia, os cdigos so denominados cifras, as mensagens no codificadas, so textos comuns e as mensagens codificadas so textos cifrados ou criptogramas. O processo de converter um texto comum em cifrado chamado cifrar ou criptografar e o processo inverso de converter um texto cifrado em comum chamado decifrar. As cifras mais simples, denominadas cifras de substituio (ou Cdigo de Csar), so as que substituem cada letra do alfabeto por uma outra letra. Por exemplo, na cifra de substituio Comum A B C D E F G H I J K L M N O P Q R S T U V W X Y Z Cifra D E F G H I J K L M N O P Q R S T U V W X Y Z A B C A letra de texto comum A substituda por D, a letra de texto comum B por E e assim por diante. Com esta cifra, a mensagem de texto comum ROMA NO FOI CONSTRUDA fica URPD QDR IRL FRQVWUXLGD HP XP GLD EM UM DIA

A Cifra de Csar - Um dos sistemas criptogrficos mais antigos e simples a chamada cifra de Csar, em homenagem ao famoso imperador romano . Jlio Csar usou sua famosa cifra de substituio para cifrar mensagens governamentais. Atualmente denomina-se qualquer cifra baseada na substituio cclica do alfabeto de cdigo de Csar. Com o uso de dois discos concntricos contendo todas as letras do alfabeto, a substituio se tornava extremamente simples.

A cifra de Csar baseia-se na seguinte propriedade: Seja m>1 um inteiro. Para cada a x a (mod m) bijetiva.

Zm fixado, temos que f: Zm

Zm definida por f(x) =

172

CAPTULO 11 CIFRA DE CSAR

Demonstrao: (i) f injetiva. De fato: f(x) = f(y) x + a = y + a (mod m) x + a - a = y + a - a (mod m) x = y (mod m) x = y em m , donde f(x) = f(y) acarreta x = y (ii) f sobrejetiva Seja x um elemento qualquer de m . Ento x - a est em m e f(x - a) = x, uma vez que em m x a = x a + km, para todo inteiro k. Logo todo x em m igual a f(x - a), donde f sobrejetiva. Como f injetiva e sobrejetiva, ento f bijetiva. Imaginemos, por questo de simplicidade, as 26 letras usuais e o espao (entre duas palavras) associados aos elementos de Z 27 conforme a Tabela 1 abaixo ( adotaremos o smbolo [ ] para indicar um espao entre as palavras): A 1 N 14 [] 0 B 2 O 15 C 3 P 16 D 4 Q 17 E 5 R 18 F 6 S 19 G 7 T 20 H 8 U 21 I 9 V 22 J 10 W 23 K 11 X 24 L 12 Y 25 M 13 Z 26

Tabela 1

Fixado um elemento a Zm ( a a chave do cdigo de transmisso e de recepo), a aplicao f: x x a (mod 27) permuta os elementos de Z 27 e, conseqentemente, os elementos do conjunto formado pelo smbolo do espao e as 26 letras. Dessa forma cada mensagem se transforma em cdigo; o fato de f ser bijetiva garante que mensagens diferentes so codificadas de maneira diferente e, ainda, a possibilidade da decodificao. Exemplo 11.1: Vejamos como codificar a frase EU VOU, usando como chave a = 14, ou seja com y = x + 15 (mod 27) E U [] V O U 5 21 0 22 15 21 5 + 14 19(mod 27) 21 + 14 8 (mod 27) 0 + 14 14 (mod 27) 22 + 14 9 (mod 27) 15 + 14 2 (mod 27) 21 + 14 8 (mod 27) S H N I B H

Portanto o cdigo para a frase dada SHNIBH. Para decodificar, considerando que o simtrico aditivo de 14 mdulo 27 13 (pois, 14 + 13 0 (mod 27)), mantendo, portanto, a chave a = 14 ), procede-se assim:

173

CAPTULO 11 CIFRA DE CSAR

S H N I B H

19 8 14 9 2 8

19 + 13 8 + 13 14 + 13 9+13 2+13 8+13

5 21 0 22 15 21

(mod 27) (mod 27) (mod 27) (mod 27) (mod 27) (mod 27)

E U [] V O U

11.1. Funes Polinomiais de Codificao


No exemplo acima usamos a funo polinomial f(x) = x + 15 (mod 27) para codificar a mensagem e usamos a sua inversa f -1(x) = x + 12 (mod 27) para decodificao. A pergunta que podemos fazer podemos usar qualquer funo polinomial mdulo m para codificar uma mensagem? A resposta no! Como vimos, precisamos da inversa para decodificar a mensagem. Assim se escolhermos uma funo polinomial que no seja bijetiva no domnio trabalhado teremos problemas em decodificar em situaes normais. Alm disso, nos casos em que tivermos multiplicaes e divises da varivel x, dependendo de Zm podemos no ter os inversos de x mdulo m nos casos em que m no primo impossibilitando a decodificao. Vejamos alguns exemplos: Exp1. Use a tabela abaixo e a funo f ( x) x2 23(mod 29) para codificar a palavra DJ A B C D E F G H I J K L M 1 2 3 4 5 6 7 8 9 10 11 12 13 N O P Q R S T U V W X Y Z 14 15 16 17 18 19 20 21 22 23 24 25 26 [] 27 28 0
D J f (4) 42
2

23 10(mod 29) 23 7(mod 29)

J G

f (10) 10

Queremos agora decodificar a mensagem JG. A inversa da funo y = x 2 + 23 obtida pelo mtodo prtico de trocar x por y e colocar y em funo de x, ou seja: y x 2 23
x y2 y
2

y 2 23 x 23 ou x 6(mod 29)

Ou seja, para voltarmos estamos interessado em saber que nmero y elevado ao quadrado igual a um nmero x + 6 (mod 29) ( isto , [f(x)]2 = x + 6 ). Assim,

174

CAPTULO 11 CIFRA DE CSAR

y2

10 6 16(mod 29)

Que nmero elevado ao quadrado igual a 16 mdulo 29? Resposta: 4. Logo J D.


G y2 7 6 13(mod 7)

Que nmero elevado ao quadrado igual a 13 mdulo 29? Temos um problema! Existem dois valores que elevado ao quadrado so iguais a 13 mdulo 29: 10 e 19, letras J e S respectivamente, pois
10.10 13(mod 29) e 19.19 13(mod 29)

impedindo-nos, em situaes mais complexas, de decodificar a mensagem. Isso se deve ao fato de f(x) = x2 + 23 no ser bijetiva.

Exerccios Use a Tabela 2 abaixo para os seus clculos Z31 G 7 T 20

A 1 N 14 ? 27

B 2 O 15 28

C 3 P 16 29

D 4 Q 17 30

E 5 R 18 [] 0

F 6 S 19

H 8 U 21

I 9 V 22

J 10 W 23

K 11 X 24

L 12 Y 25

M 13 Z 26

1. Utilize as funes abaixo para codificar e sua inversa* para decodificar as mensagens dadas. a) y 2x(mod31) ; y x 7(mod31) ; O MAIS QUERIDO b) y 3x(mod31) ; y 2x 30(mod31) ; NAVEGAR PRECISO? c) y 4x(mod31) ; y 3x 2(mod31) ; SE VOU NO FICO x 1 d) y 5x(mod31) ; y (mod 31) ; ELVIS NO MORREU x 29 * Use o mtodo da troca de variveis para encontrar a funo inversa

175

CAPTULO 11 CIFRA DE CSAR

LEITURA COMPLEMENTAR: ANLISE DE FREQUNCIA


Em qualquer lngua, alguns sons so utilizados com mais frequncia do que outros. Isto significa que, na linguagem escrita, algumas letras tambm so mais utilizadas que outras. Determinar a frequncia com que ocorrem determinadas letras em determinada lngua, ou seja, fazer uma anlise da frequncia de ocorrncia de letras. Apesar de no se saber quem foi o primeiro a perceber que a variao na frequncia de letras poderia ser explorada para se quebrar cifras, a descrio mais antiga de que se tem conhecimento e que descreve esta tcnica data do sculo 9 e devida ao cientista Abu Yusuf Ya 'qub ibn Is-haq ibn as-Sabbah ibn 'omran ibn Ismail al-Kindi. Conhecido como o filsofo dos rabes, al-Kindi foi o autor de 290 livros sobre medicina, astronomia, matemtica, lingustica e msica. No entanto, seu maior tratado, o qual foi apenas redescoberto em 1987 no Arquivo Sulaimaniyyah Ottoman em Istambul, na Turquia, intitulado "Um Manuscrito sobre Decifrao de Mensagens Criptogrficas. A cifra de substituio monoalfabtica parecia inquebrvel devido ao nmero muito grande de chaves possveis. Entretanto, havia uma fraqueza que minava sua segurana. A quebra da cifra de substituio marca o nascimento da criptanlise. Tal fato ocorreu durante os anos dourados da civilizao islmica, quando muitos manuscritos estrangeiros foram levados para Bagd para integrarem as grandes bibliotecas rabes. Alguns destes manuscritos estavam encriptados, o que motivou os arrombadores de cdigos a quebrarem as cifras para revelar os segredos que continham. As letras "A" e "I" so as mais comuns em rabe. No Ingls, as letras mais comuns so o "E", o "T" e o "A". J no Portugus, as mais frequentes so "A", "E", "O" e "S". Se uma mensagem cifrada de modo que cada letra seja substituda por uma outra, ento a nova letra assumir todas os atributos da letra original, inclusive com que frequncia utilizada. Desta forma, se a longa mensagem estiver em Portugus e se a letra mais comum na mensagem cifrada for G, ento G provavelmente representa A. Se a segunda letra mais frequente na mensagem cifrada for W, ento a probabilidade de que esteja substituindo o E bastante grande, e assim por diante. Devemos considerar qual foi a lngua utilizada para redigir a mensagem. Esta uma questo essencial porque define o padro da frequncia da ocorrncia de letras que deve ser usada para fazer a comparao. bvio que SEMPRE existem outras pistas que podem ajudar: o remetente da mensagem, o destinatrio, o possvel assunto, etc. Caso estejamos considerando o Portugus, claro que se vai fazer uma anlise de frequncia usando esta lngua como base. Quanto mais longo for o texto, maior a probabilidade dos valores encontrados estarem mais prximos dos valores padro. Alm disso, no se deve esquecer que os valores padro representam a MDIA da frequncia de ocorrncia. Se, por exemplo, o padro para a letra A de 14.63%, esta frequncia pode variar, digamos, de 13% a 17%. Neste caso estamos contando com um desvio de cerca de 2%.

176

CAPTULO 11 CIFRA DE CSAR

Abaixo esto as tabelas de das frequncias relativas das letras nas lnguas Portuguesa e Inglesa.

Frequncia relativa das letras no Portugus

Frequncia relativa das letras no Ingls

177

CAPTULO 11 CIFRA DE CSAR

Exerccios 1. Suponha que voc interceptou a mensagem: JV F UZEYVZIF JFSIRJJV V JV VCR DV RDRJJV

Segundo suas fontes ela foi codificada utilizando a cifra de Csar de acordo com a tabela seguinte
A 0 N 13 B 1 O 14 C 2 P 15 D 3 Q 16 E 4 R 17 F 5 S 18 G 6 T 19 H 7 U 20 I 8 V 21 J 9 W 22 K 10 X 23 L 11 Y 24 M 12 Z 25

claro que voc no sabe a chave usada. Como bom estudante de Estatstica, voc fez uma anlise de frequncia ( isso que interesse nas coisas alheias!). Suponha que voc j havia interceptado um texto com mais de 1000 letras da mesma origem. De acordo com a anlise de frequncia, a letra A mais comum na lngua portuguesa. Voc percebeu que a letra mais comum no texto cifrado foi a R. Com mais uma suposio de que a chave ainda no foi trocada, decodifique a mensagem. Desconsidere os espaos. 2. Para as questes de 1 a 6, utilize a seguinte tabela de converso:
A B C D E F G 1 2 3 4 5 6 7 N O P Q R S T 14 15 16 17 18 19 20 *O espao em branco [ ] representado pelo valor 0. H 8 U 21 I 9 V 22 J 10 W 23 K 11 X 24 L 12 Y 25 M 13 Z 26

1. Utilizando a funo f(x) = x + 13 (mod 27), codifique a palavra SOFTWARE. 2. Utilizando a funo f(x) = 4.x + 11 (mod 27), codifique a palavra UNIVERSO. Encontre a funo inversa de f(x) e verifique se sua resposta est correta. 3. Decodifique a palavra GESIOF sabendo que a funo f(x) = x + 14 (mod 27) foi utilizada para cifragem. 4. Decodifique a palavra BVJZOL sabendo que a funo f(x) = 5.x + 17 (mod 27) foi utilizada para cifragem. 5. Dada a mensagem em texto simples FELIZ, e o seu respectivo texto cifrado YXDAR, encontre a funo afim que foi utilizada na cifragem. Determine a funo inversa 6. Dada a mensagem em texto simples TEMPO, e o seu respectivo texto cifrado ORTNG, encontre a funo afim que foi utilizada na cifragem. Determine a funo inversa.

178

Captulo 12

CIFRA DE VIGENRE
A cifra de Vigenre tem este nome em homenagem a Blaise de Vigenre, embora realmente tenha sido inventada antes por Giovan Batista Belaso. O que Vigenre fez foi modificar a cifra para torn-la mais robusta. A cifra de Vigenre um mtodo de encriptao que usa um srie de diferentes cifras de Csar baseadas em letras de uma senha. Trata-se de uma verso simplificada de uma mais geral cifra de substituio polialfabtica, inventada por Leone Battista Alberti a cerca de 1465. A cifra de Vigenre foi uma campe em segurana. Foram precisos 300 anos para que, quase que simultaneamente, ao redor de 1860, Babbage (na Inglaterra) e Kasiski (na Alemanha) quebrassem a cifra. A inveno da cifra de Vigenre erradamente atribuda a Blaise de Vigenre; encontra-se originalmente descrita por Giovan Batista Belaso no seu livro datado de 1553 com o ttulo La cifra del. Sig. Giovan Batista Belaso. Esta cifra muito conhecida porque fcil de perceber e de pr em prtica. Conseqentemente, muitos programadores implementaram esquemas de encriptao nas suas aplicaes que so no essencial cifras de Vigenre.

Descrio Em uma cifra de Csar, cada letra do alfabeto deslocada da sua posio um nmero fixo de lugares; por exemplo, se tiver uma deslocao de 3, a letra A se torna D, B se torna E, etc. A cifra de Vigenre consiste na sequncia de vrias cifras de Csar com diferentes valores de deslocamento. A cifra de Vigenre pode ser vista algebricamente. A encriptao pode ser escrita como Ci e a decriptao como Pi (Ci ai) (mod m) (Pi + ai) (mod m)

onde Pi corresponde aos valores das letras a serem cifradas, ai aos valores das letras da chave e Ci aos valores das letras cifradas.

179

CAPTULO 12 CIFRA DE VIGENRE

Exemplo 12.1: Vamos supor agora, que a palavra-chave escolhida tenha sido "GREGO" e a mensagem a ser codificada seja PERXES PREPARA UM ATAQUE. Para isso utilizaremos a seguinte tabela de valores mdulo 31: A B C D E F G H I 1 2 3 4 5 6 7 8 J K L M N O P

9 10 11 12 13 14 15 16

Q R S T U V W X Y Z ? [] 17 18 19 20 21 22 23 24 25 26 27 28 29 30 0 Para codificar a mensagem, podemos escrever as letras da palavra-chave quantas vezes for preciso acima da frase:
G R E G O G R E G O G R E G O G R E G O G R E G P E R X E S P R E P A R A U M A T A Q U E

isto equivale a fazer os seguintes clculos


7 16 18 5 5 18 7 24 15 5 7 19 18 0 5 16 7 18 15 5 7 16 18 1 5 18 7 1 15 0 7 21 18 13 5 0 7 1 15 20 7 1 18 17 5 21 7 5

Somando termo a termo mod 31 (Ci


23 W 23 W 23 W 0 20 T 26 Z 18 R 21 U 25 Y 20 T

(Pi + ai)), obtemos:


23 W 19 S 23 W 8 H 15 O 28 0 5 E 8 H 4 D 8 H 4 D 26 Z 12 L

Para decifrar a mensagem WWW TZRUYTWSWHO EHDHDZL basta fazer a operao inversa (Pi (Ci ai)).

180

CAPTULO 12 CIFRA DE VIGENRE

Exerccios: Para as questes abaixo, utilize a seguinte tabela de converso: A 1 B 2 C D E 3 S 4 5 F 6 G H 7 8 I J K L M N O ? [] P

9 10 11 12 13 14 15 16

Q R

T U V W X Y Z

17 18 19 20 21 22 23 24 25 26 27 28 29 30 0
Tabela de converso de caracteres para a Cifra de Vigenre (mod 31).

1) Decifre a mensagem abaixo, usando a palavra-chave CERA: QIOETGZTQHVIZSLARIISKE 2) Descubra a mensagem a seguir, capturada na transmisso do inimigo. TXBQMXWFRSIIMOETVOEKMENAVKYHAVT A palavra-chave utilizada foi TEBAS. Codifique a mensagem RIO DE JANEIRO utilizando a chave TRIUNFO. Codifique a mensagem FRONTEIRAS DO BRASIL utilizando a chave TIGRE. Decodifique a mensagem DYDSSX YNYFS sabendo que a chave BYTE foi utilizada. Decodifique a mensagem XIDHY OLOWFBRL sabendo que a chave MISTURA foi utilizada. 7) Dada a palavra em texto simples ILUSTRE, e o seu respectivo texto cifrado MCI FYS, encontre a chave que foi utilizada na cifragem. 8) Dada a mensagem em texto simples CRISE DOS INFERNOS, e o seu respectivo texto cifrado EEVHYHVGSBW VYZEGS, encontre a chave que foi utilizada na cifragem. 9) Desafio: dada a mensagem OSFYSRWXMFTANASWVNSSDMJMVAMVSISINQBLUDB,decodifique-a e encontre a chave que foi utilizada na cifragem. 3) 4) 5) 6)

181

Captulo 13

CIFRA DE HILL
Uma das desvantagens de cifras de substituio que elas preservam as frequncias de letras individuais, tornando relativamente fcil quebrar o cdigo por mtodos estatsticos. Uma maneira de superar este problema dividir o texto em grupos de letras e criptografar o texto comum por grupo, em vez de uma letra de cada vez. Um sistema poligrfico um sistema de criptografia no qual o texto comum dividido em conjuntos de n letras, cada um dos quais substitudo por um conjunto de n letras cifradas. Veremos uma classe de sistemas poligrficos chamados cifras de Hill (Em 1929 Lester S. Hill publica seu livro Cryptography in an Algebraic Alphabet, no qual um bloco de texto claro cifrado atravs de uma operao com matrizes). Daqui em diante, ns vamos supor que cada letra de texto comum e de texto cifrado, tem um valor numrico que especifica sua posio no alfabeto padro (Tabela 1). Utilizaremos o smbolo [ ] para indicar um espao entre as letras ou palavras. Tabela 1 F G H 6 7 8 S T U 19 20 21

A 1 N 14

B 2 O 15

C 3 P 16

D 4 Q 17

E 5 R 18

I 9 V 22

J 10 W 23

K 11 X 24

L 12 Y 25

M 13 Z 0

Iniciemos com o caso mais simples de cifras de Hill que utiliza matrizes 2x2. Usando a tabela acima em 26 transformaremos pares sucessivos de texto comum em texto cifrado pelo seguinte procedimento: Passo 1. Escolha uma matriz A, 2 x 2, com entradas inteiras para efetuar a codificao. A matriz deve ser inversvel mdulo 26 . Passo 2. Agrupe letras sucessivas de texto comum em pares, adicionando uma letra fictcia para completar o ltimo par se o texto comum tem um nmero mpar de letras; substitua cada letra de texto comum por seu valor numrico. Passo 3. Converta cada sucessivo p1 p2 de letras de texto comum em um vetor-coluna p, e forme o produto A.p. Ns chamamos p de vetor comum e Ap o correspondente vetor cifrado.

182

CAPTULO 13 CIFRA DE HILL

Passo 4. Converta cada vetor cifrado em seu equivalente alfabtico. Em todas as operaes converter os valores para congruncia mdulo 26. Exemplo 13.1: Cifra de Hill de uma Mensagem

Use a matriz A=

1 2 0 3

para obter a cifra de Hill da mensagem de texto comum

PARAFUSO
Soluo. Se ns agruparmos o texto comum em pares de letras, obteremos PA RA FU SO ou, equivalentemente, usando a Tabela 1, 16 -1 18 - 1 6 -21 19 -15 Para codificar o par PA ns efetuamos o produto matricial

1 2 0 3

16 1

18 3

(mod 26)

que fornece o texto cifrado RC pela Tabela 1. Para codificar o par RA ns efetuamos o produto matricial
1 0 2 3

18 1

20
=

(mod 26)

que fornece o texto cifrado TC. Os clculos para os demais vetores cifrados so
1 2 0 3

6 21
=

48 63 49
= =

22 11 23
= (mod 26)

1 2 0 3

19 15

45

19

(mod 26)

Estes vetores correspondem aos pares de textos cifrado VK e WS, respectivamente. Coletando os pares, obtemos a mensagem cifrada completa

183

CAPTULO 13 CIFRA DE HILL

RCTCVKWS
Como o texto comum foi agrupado em pares e criptografado por uma matriz 2 x 2, dizemos que a cifra de Hill do Exemplo 1 uma 2-cifra de Hill. Evidentemente tambm possvel agrupar o texto comum em ternos e criptografar com uma matriz 3 x 3 com entradas inteiras; isto chamado uma 3-cifra de Hill. Em geral, para uma n-cifra de Hill agrupamos o texto comum em conjunto de n letras e codificamos com uma matriz codificadora n x n de entradas inteiras. Decifrando Cada cifra til deve possuir um procedimento para decifrar. Para decifrar as cifras de Hill, usamos a inversa (mod m) da matriz codificadora. Para ser preciso, se m um inteiro positivo, dizemos que uma matriz A com entradas em Zm inversvel mdulo m se existir uma matriz B com entradas em Zm tal que A . B = B . A = I (mod m) Suponha agora que A=

a11

a12

a21 a22

inversvel mdulo m e que esta matriz usada para uma 2-cifra de Hill. Se

p=

p1 p2

um vetor comum, ento c=Ap o correspondente vetor cifrado e p = A-1 c Assim, cada vetor comum pode ser recuperado do correspondente vetor cifrado pela multiplicao esquerda por A-1 (mod m). Em criptografia importante saber quais matrizes so inversveis mdulo m e como obter suas inversas. Em seguida investigaremos estas questes. Em aritmtica comum, uma matriz quadrada A inversvel se, e somente se, det (A) 0 ou, equivalentemente, det (A) tem um inverso. O teorema seguinte o anlogo deste resultado em aritmtica modular.

184

CAPTULO 13 CIFRA DE HILL

Teorema 13.1 Uma matriz quadrada A em Zm inversvel mdulo m se, e somente se, o detA( md m) tem um inverso mdulo m. Desse modo, em particular, temos que, se

A=

a b c d

tem entradas em Zm se o det(A) = ad bc (mod m) for relativamente primo com m, ento a inversa de det(A) (mod m) dada por

A-1 = (ad-bc)-1

d c

b a

(mod m)

(I)

Onde (ad bc)-1 (mod m) o inverso de ad bc (mod m). Exemplo 13.2. Decifrando uma 2-Cifra de Hill Decifre a seguinte 2-cifra de Hill, que foi dada no Exemplo 1: RCTCVKWS Soluo: Pela Tabela 1, o equivalente numrico do texto cifrado 18-3 20-3 22-11 23-19 para obter os pares de texto comum, multiplicamos cada vetor cifrado pela inversa de A: det(A) = 1.3 2.0 = 3 temos que o inverso de 3 mdulo 26 igual 9, ou seja 3.9 1(mod 26). Assim, por ( I ), A-1 = 9 Logo

3 0

2 1

27 0

18 9

1 8 0 9

(mod 26)

42 16 1 8 18 = = (mod 26) 27 1 0 9 3

1 8 0 9

20 3
=

44 27
=

18 1
(mod 26)

185

CAPTULO 13 CIFRA DE HILL

1 8 0 9 1 8 0 9

22 11

110 99 175 171

6 21
19 15

(mod 26)

23 19

(mod 26)

Pela Tabela 1, pode-se ver que os equivalentes alfabticos destes vetores so

PARAFUSO
Matrizes Inversas Mdulo m Teorema: Se M uma matriz quadrada de ordem n e det M 0, ento a inversa de M mdulo m M-1 = (det M )-1. M (mod m) onde M a matriz adjunta de M. Exemplo 13.4:

9 0 5
Utilizando a matriz A

8 7 6 (mod 29) como chave, codifique e decodifique a mensagem 3 2 0

MATEMTICA em 29 , tal que A 1 N 14 27 B 2 O 15 28 C 3 P 16 [] 0 D 4 Q 17 E 5 R 18 F 6 S 19 G 7 T 20 H 8 U 21 I 9 V 22 J 10 W 23 K 11 X 24 L 12 Y 25 M 13 Z 26

Soluo: Convertendo a mensagem:

13 MAT 1 20 ; EM

5 13 ; TIC 27

20 9 3 ; A_ _

1 0 0

Para codificarmos a mensagem, basta multiplicar cada matriz 3x1 obtida pela matriz codificadora A:

186

CAPTULO 13 CIFRA DE HILL

9 0 5 13 8 7 6 1 3 2 0 20

217 231 41 180 293 41 195 241 78


9 3

14 28 (mod 29) 12 6 3 (mod 29) 12 21 9 20 (mod 29)

9 0 5 5 8 7 6 13 3 2 0 27

9 0 5 20 8 7 6 9 3 2 0 3
9 0 5 1 8 7 6 3 2 0 0

0 = 8 (mod 29)

A mensagem codificada : NLFCLUITIHC DECODIFICAO: Para decodificar precisamos da matriz inversa de A. Clculo da matriz inversa A 1 (mod 29): 2- Det (A) = -133, ou seja, Det (A) = 12 ( mod 29). Temos que De fato: 12 (29) 1 17(mod 29) 1227 1(mod 29) , pois 123 17 129 173 12 1227 123 17(mod 29)
12 18 -5 17 18 24 10 14 11 (mod 29) 23 15 5

1 Det ( A)

1 (mod 29) 17 . 12

3- Matriz dos Cofatores: C

10 -15 -18 35 -14 63

4- A Matriz Adjunta ( M ) a transposta da Matriz dos Cofatores:


17 10 23 M C
t

18 14 15 (mod 29) 24 11 5

5- Matriz Inversa A 1 =
17 10 23 A
1

1 .M (mod 29), temos: Det ( A)


289 170 391 306 238 255 408 187 85 28 25 14 16 6 23 (mod 29) 2 13 27

17. 18 14 15 24 11 5

Basta, ento, multiplicar a mensagem codificada pela matriz inversa para obtermos a mensagem original.

187

CAPTULO 13 CIFRA DE HILL

Exerccios: Z32 A 1 N 14 27 B 2 O 15 ? 28 C 3 P 16 29 D 4 Q 17 30 E 5 R 18 31 F 6 S 19 [] 0 G 7 T 20 H 8 U 21 I 9 V 22 J 10 W 23 K 11 X 24 L 12 Y 25 M 13 Z 26

Tabela 1

1. Utilizando a tabela 1 e as matrizes dadas como chave, codifique e decodifique as mensagens correspondentes, pelo mtodo de Hill a) A
1 3 2 7 , O PIOR CEGO AQUELE QUE NO ENXERGA O QUE V

b) A

9 15 19
7

2
8 4

, LEO AZUL

c)

1 21

12 23 14 , MATEMTICA LEGAL 22

d)

1 2 7 B 0 3 1 , CATACLISMTICO 0 5 2

Para as questes de 2 a 8, utilize a seguinte tabela de converso: A 1 N 14 B 2 O 15 C 3 P 16 D 4 Q 17 E 5 R 18 F 6 S 19 G 7 T 20 H 8 U 21 I 9 V 22


10 11 9 12

J 10 W 23

K 11 X 24

L 12 Y 25

M 13 Z 0

Tabela de converso de caracteres mdulo 26.

2. Codifique a palavra AFRODITE utilizando a matriz

14 13 11
3. Codifique a palavra ELETRICIDADE utilizando a matriz 15 17 18 19 16 12

188

CAPTULO 13 CIFRA DE HILL

4. Decodifique a palavra KJQCDYVU sabendo que a matriz cifragem.

20 21 19 22

foi utilizada na

2 6 3
5. Decodifique a palavra GEIOHSSFGQUD sabendo que a matriz 1 7 4 8 9 5 na cifragem. 6. Decodifique 2 3 1 11 13 5 16 9 14 8 7 6 a
4 12 15 10

foi utilizada

mensagem

ODNINDTYTXPWVZPQ

sabendo

que

matriz

foi utilizada na cifragem.

7. Dada a mensagem em texto simples MITO, e o seu respectivo texto cifrado UQNI, encontre a matriz 2 x 2 que foi utilizada na cifragem. 8. Dada a mensagem em texto simples ALGORITMO e o seu respectivo texto cifrado ODVMFFYWS. Encontre a matriz 3 x 3 que foi utilizada na cifragem, sabendo que todos os elementos desta matriz so inteiros positivos distintos menores ou iguais a 9. Para as questes de 9 e 10, utilize a seguinte tabela de converso: A 1 M 13 Y 25 B 2 N 14 Z 26 C 3 O 15 27 D 4 P 16 28 E 5 Q 17 29 F 6 R 18 30 G 7 S 19 31 H 8 T 20 32 I 9 U 21 33 J 10 V 22 34 K 11 W 23 35 L 12 X 24 36

*O espao em branco representado pelo valor 0 (zero). Tabela de converso extendida de caracteres mdulo 37.

9. Codifique a palavra EQUAES utilizando a matriz

31 33 29 35
26 28 29

10. Decodifique a palavra PBGEZJMJ sabendo que a matriz utilizada na cifragem.

21 25 27 22 23 24

foi

189

Captulo 14

RSA
Trs americanos desenvolveram um sistema de cdigo secreto, chamado RSA, baseado nas dificuldades existentes para descobrir os fatores primos de um nmero muito grande. Criavase um novo ramo da Criptografia, a cincia dos cdigos, fortemente baseado na Teoria dos Nmeros. Com o advento dos computadores e da computao algbrica, a Criptografia ganhou um novo impulso. Neste momento, a proliferao de senhas bancrias e de cartes de crdito, bem como a crescente necessidade de criptografar dados confidenciais que inundam a Internet, fazem da Criptografia um dos ramos mais pesquisados da Matemtica. O sistema RSA, batizado em homenagem a seus inventores Ronald Rivest, Adi Shamir e Leonard Adleman, foi o primeiro criptossistema de chave pblica e ainda o mais importante. Sua segurana est intimamente relacionada dificuldade de encontrar a fatorao de um nmero inteiro positivo composto, que o produto de dois primos gigantes.

14. 1. PR-CODIFICAO
Em primeiro lugar, devemos converter a mensagem em uma sequncia de nmeros. Essa primeira etapa chamada de pr-codificao. H vrias maneiras de se fazer isso. Aqui vamos supor que o texto no contm acentuao, pontuao, nmeros etc, apenas as letras A a Z (maisculas). Tambm vamos adicionar espaos em branco entre palavras, que ser substitudo pelo nmero 99. A letra A ser convertida no nmero 10, B ser 11 e assim por diante, at o Z correspondendo ao nmero 35. Observe que cada letra corresponde a um nmero com exatamente dois algarismos. Isso evita ambigidades.
A 10 N 23 [] 99 B 11 O 24 C 12 P 25 D 13 Q 26 E 14 R 27 F 15 S 28 G 16 T 29 H 17 U 30 I 18 V 31 J 19 W 32 K 20 X 33 L 21 Y 34 M 22 Z 35

A chave pblica um nmero n = p . q, onde p e q so primos. Antes de comear devemos, ento escolher esses nmeros. O ltimo passo da pr-codificao quebrar a mensagem em blocos. Esses blocos devem ser nmeros menores que n. A maneira de escolher os blocos no nica, mas importante evitar duas situaes: Nenhum bloco deve comear com o nmero 0 (problemas na decodificao). Os blocos no devem corresponder a nenhuma unidade lingstica (palavra, letra, etc). Assim a decodificao por contagem de freqncia fica impossvel.

190

CAPTULO 14 RSA

14.2 Codificando e decodificando


Para codificar a mensagem precisamos de n = p . q e de um inteiro positivo e (1 < e < (n)) que seja inversvel mdulo (n). Em outras palavras, m.d.c.(e, (n)) = m.d.c. (e, (p 1).(q 1)) = 1 Note que e sempre mpar, dado que p 1 par. Chamaremos o par (n, e) de chave de codificao do sistema RSA. Codificaremos cada bloco de mensagem separadamente e a mensagem codificada ser a seqncia de blocos codificados. Vamos agora mostrar como codificar cada bloco b. Chamaremos o bloco codificado de C(b). Em primeiro lugar, lembre-se que b menor que n. Ento: C(b) onde 0 be (mod n) C(b) < n.

Exemplo 14.1: Considere a frase Paraty linda. Convertendo em nmeros, 25 10 27 10 29 34 99 14 99 21 18 23 13 10 Agora devemos escolher n. Vamos comear com um nmero pequeno, por exemplo, n = 11.13 = 143. Podemos ento quebrar a mensagem acima em blocos, que devem ter valor menor que 143 e no devem iniciar com zero.: 25 102 7 102 93 49 91 49 92 118 23 13 10 Ento temos que (143) = 10.12 = 120 e portanto vamos escolher e como o menor primo que no divide 120. O valor 7. Logo, C(25) C(25) C(25) C(25) C(25) C(25) C(25) 257 (mod 143) 254 . 252 . 251 (mod 143) 254 . 53 . 25 (mod 143) 532 . 53 . 25 (mod 143) 92 . 53 . 25 (mod 143) 14 . 25 (mod 143) 64 (mod 143)

Procedendo dessa maneira com todos os blocos, obtemos a seguinte mensagem cifrada: 64 119 6 119 102 36 130 36 27 79 23 117 10 Vejamos agora como proceder para decodificar um bloco de mensagem codificada.

191

CAPTULO 14 RSA

A informao que precisamos para decodificar est contida no par (n, d), onde d (1<d< (n)) o inverso de e mdulo (n) (e.d 1 (mod (n)). Chamaremos (n, d) de chave de decodificao e D(c) o resultado do processo de decodificao. D(c) dado por: D(c) cd (mod n)

onde 0 D(c) < n. Observe que possvel calcular d, desde que (n) e e sejam conhecidos. Entretanto, se no conhecemos p e q praticamente impossvel calcular d. Voltando ao nosso exemplo, temos que n = 143 e e = 7. Para calcular d, usamos o Teorema de Euler: d.e 1 (mod (n)) d e
( (n)) 1

(mod (n))

ou seja, d d d d d 7 ( (143)) 1 (mod (143)) 7 (120) 1 (mod 120) 732 1 (mod 120) 731 (mod 120) 103 (mod 120)

Como exemplo, deciframos o primeiro bloco da mensagem cifrada: D(64) D(64) D(64) D(64) D(64) D(64) D(64) D(64) D(64) D(64) 64103 (mod 143) 64100 . 643 (mod 143) (6410)10 . 643 (mod 143) ((645)2)10 . 643 (mod 143) ((642 . 643)2)10 . 643 (mod 143) ((92 . 25)2)10 . 25 (mod 143) ((12)2)10 . 25 (mod 143) (144)10 . 25 (mod 143) (1)10 . 25 (mod 143) 25 (mod 143)

Os outros blocos ficam como exerccios. A pergunta bvia que surge agora : D(C(b)) = b? Ou seja, decodificando um bloco de mensagem codificada, encontramos um bloco da mensagem original? Porque seno todo nosso esforo foi sem sentido. Vamos mostrar nessa seo que a resposta para esta pergunta : sim! Consideremos ento n = p . q. Vamos provar que D(C(b)) b (mod n)

192

CAPTULO 14 RSA

E por que no a igualdade D(C(b)) = b (mod n)? Observe que D(C(b)) e b so menores que (n 1). Por isso escolhemos b menor que n e mantivemos os blocos separados depois da codificao. Pela definio de D e C temos que D(C(b)) (be)d bed (mod n)

Mas d o inverso de e mdulo (n). Logo, existe inteiro k tal que e.d = 1 + k. (n). Logo, bed b1 + k.
(n)

b . (b

(n) k

) (mod n)

Se m.d.c.(b, n) = 1, ento podemos usar o teorema de Euler: bed (1)k . b b (mod n)

Se b e n no so primos entre si, observe que n = p . q, p e q primos distintos. Logo, bed b1 + k.


(n)

b1 + k.(p-1).(q-1)

(b(p-1))k.(q-1). b (mod n)

Se m.d.c.(b, p) = 1, ento podemos usar o teorema de Fermat [bp-1 1 (mod p)]. Caso contrrio, temos que p | b e portanto bed Logo, bed b (mod p) b 0 (mod p)

qualquer que seja b. Fazemos o mesmo para o primo q, obtendo: bed Portanto, bed b (mod p.q) bed b (mod n) c.q.d. Exemplo 14.2:
A 10 N 23 [] 99 B 11 O 24 C 12 P 25 D 13 Q 26 E 14 R 27 F 15 S 28 G 16 T 29 H 17 U 30 I 18 V 31 J 19 W 32 K 20 X 33 L 21 Y 34 M 22 Z 35

b (mod q)

Digamos que voc est interessado em enviar a mensagem

193

CAPTULO 14 RSA

FINALMENTE Voc deve inicialmente numerar toda a palavra de acordo com a tabela acima: 15 18 23 10 21 22 14 23 29 14 Agora, suponha que a chave seja (n, e) = (7663, 17) Ento voc deve escolher blocos numricos para codificar que sejam menores que n e que no comecem com zero. Escolhamos, por exemplo, os seguintes blocos (veja que isso vai depender da mquina que voc est trabalhando): 151 823 102 122 142 329 14 Basta agora pegar cada bloco e elevar potncia e mdulo n: 15117 5137 (mod 7663) 15117 5137 (mod 7663) 82317 3345 (mod 7663) 10217 1635 (mod 7663) 12217 3009 (mod 7663) 14217 2186 (mod 7663) 32917 3890 (mod 7663) 1417 2032 (mod 7663) A mensagem codificada 5137 3345 1635 3009 2186 3890 2032 Agora vejamos como fazer se recebemos uma mensagem codificada por algum utilizando a chave pblica (n, e) = (7663, 17). Suponha que a mensagem recebida seja a mesma obtida acima: 5137 3345 1635 3009 2186 3890 2032 Para entender o que significa voc precisa da chave privada d, que inversa de e (mod ( n )). Etapas: 1. Primeiro, fatoramos o nmero n = 7623 ( veja abaixo a fatorao pelo mtodo de Fermat). Encontramos os primos 79 e 97. 2. Calculamos (n) = (7663) = (79) . (97) = 78.96 = 7488.

194

CAPTULO 14 RSA

3. Calculamos d

( (n)) 1

(mod (n))

17

(7488) 1

(mod 7488)

881 (mod 7488).

4. Agora basta pegar cada bloco que recebemos e elevar potncia d mdulo n. 5137881 3345881 1635881 3009881 2186881 3890881 2032881 5. Ento colocamos os resultados alinhados 151 823 102 122 142 329 14 depois separamos em blocos de dois nmeros 15 18 23 10 21 22 14 23 29 14 que corresponde a mensagem enviada FINALMENTE 151 (mod 7663) 823 (mod 7663) 102 (mod 7663) 122 (mod 7663) 142 (mod 7663) 329 (mod 7663) 14 (mod 7663)

14. 3. Assinatura digital utilizando a criptografia RSA


1 passo: Como a segurana do RSA reside na dificuldade de fatorar nmeros que so produtos de 2 primos; ento aconselhvel escolher primos bem grandes (10 ou mais dgitos). Mas, para fins didticos vamos escolher primos relativamente pequenos (menores que 1000), cuja idia a mesma para primos grandes. Ainda, para efeitos deste exemplo, chamaremos o remetente de Alice (A) e o destinatrio de Bob (B). Para elaborarmos as chaves pblicas e privadas do esquema, ento escolhemos adequadamente dois primos p e q para cada participante, e calculamos o produto n = p . q. Os nmeros primos de Alice so: pA = 859, qA = 547. Que resulta em nA = 469873. Os nmeros primos de Bob so: pB = 937, qB = 461. Que resulta em nB = 431957.
*Observao 1: os primos p e q no devem ser muito prximos para evitar fatoraes rpidas como a de Fermat.

2 passo: O nmero n encontrado desta maneira faz parte tanto da chave pblica como da chave privada de cada participante. Agora, escolhemos um nmero e que tambm ser pblico e que seja relativamente primo funo de Euler (n) MDC( (n), e) = 1.

195

CAPTULO 14 RSA

Para Alice, (nA) = (859) . (547) = 858 . 546 = 468468. Uma boa escolha para Alice o nmero primo eA = 5, pois o MDC(468468, 5) = 1. Para Bob, (nB) = (937) . (461) = 936 . 460 = 430560. Uma boa escolha para Bob o nmero primo eB = 7, pois o MDC(430560, 7) = 1.
*Observao 2: a chave pblica e no necessariamente um nmero primo, este exemplo foi apenas uma mera coincidncia.

3 passo: Tendo posse de (n) e da chave pblica e, podemos calcular a chave secreta d. Sendo que d o inverso multiplicativo de e, ou seja, e.d 1 (mod (n)). Pelo teorema de Euler, d e ( ( n )) 1(mod (n)) . Fazendo os devidos clculos encontramos que: Para Alice, 5. dA 1 (mod 468468) dA = 281081. Para Bob, 7. dB 1 (mod 430560) dB = 307543.
*Observao 3: uma forma computacional rpida para se encontrar a chave privada d : int d = 0; for ( d = 2; d < phi; d++ ) {if ( (e * d) % phi == 1 ) ) {System.out.println( Inverso = + d ); break; // d encontrado, sai do lao}}

Pronto, j temos posse das chaves necessrias para cifrar e decifrar mensagens. O que deve ser disponibilizado nas pginas amarelas ou qualquer diretrio pblico: Chave pblica de Alice (nA, eA) = (469873, 5). Chave pblica de Bob (nB, eB) = (431957, 7). O que no deve ser disponibilizado, pois secreto e pertence somente aos seus donos: Chave privada de Alice (nA, dA) = (469873, 281081). Chave privada de Bob (nB, dB) = (431957, 307543).
*Observao 4: note que tanto os primos p e q, assim como a funo (n), tambm devem ser mantidos secretos! Pois a partir de p e q, pode-se facilmente calcular (n) = (p 1) * (q 1). E vice-versa, a partir de (n) pode-se calcular p e q atravs de um sistema linear e uma simples equao do 2 grau. Tendo posse de (n), calcula-se a chave privada d como demonstrado.

4 passo: Finalmente, como fazer para criptografar uma mensagem? Por exemplo, tomemos a palavra BRASIL, que no cdigo ASCII corresponde aos valores: (66, 82, 65, 83, 73, 76) Concatenando esses valores, obtemos 668265837376. Reagrupando em blocos (de qualquer tamanho), obtemos 668, 265, 837, 376. O valor contido em cada bloco no deve exceder o valor da chave pblica n. Tambm se deve tomar cuidado para que nenhum bloco inicie com o valor zero, com risco de se criar um erro na cifragem. Os blocos evitam ataques por freqncia, j que se a mensagem fosse codificada caractere por caractere, o RSA se tornaria uma mera cifra de substituio. A codificao dos blocos feita, elevando-se cada valor do bloco ao expoente pblico e mdulo n do destinatrio, obtendo-se o menor valor positivo correspondente. Assim, se Alice deseja enviar a palavra BRASIL para Bob, ela deve utilizar a chave pblica (nB, eB) = (431957, 7) de Bob, e efetuar C(b) = be (mod n), da seguinte forma:

196

CAPTULO 14 RSA

C(668) = 6687 (mod 431957) C(265) = 2657 (mod 431957) C(837) = 8377 (mod 431957) C(376) = 3767 (mod 431957)

331318 (mod 431957) 387742 (mod 431957) 213180 (mod 431957) 121902 (mod 431957)

Alice, ento, deve enviar a mensagem criptografada (331318, 387742, 213180, 121902) para Bob. 5 passo: Para decifrar a mensagem, Bob deve proceder de maneira inversa, utilizando a sua chave secreta (nB, dB) = (431957, 307543), ou seja, D(c) = cd (mod n), da seguinte forma (geralmente no uma congruncia fcil de resolver...): D(331318) = 331318307543 (mod 431957) D(387742) = 387742307543 (mod 431957) D(213180) = 213180307543 (mod 431957) D(121902) = 121902307543 (mod 431957) 668 (mod 431957) 265 (mod 431957) 837 (mod 431957) 376 (mod 431957)

Bob reagrupa a mensagem decodificada 668265837376, separa em pares, (66, 82, 65, 83, 73, 76) e obtm a mensagem original BRASIL. Propriedades matemticas garantem que aplicando a funo de cifragem e depois decifragem, a mensagem original continua a mesma, isto , b = D(C(b)).

Assinatura: Assinar uma mensagem garantir que ela realmente vem de quem diz ser o remetente. Como podemos assinar uma mensagem utilizando o RSA? No exemplo, anterior, perceba que em nenhum momento chegamos a usar as chaves do remetente (Alice), portanto chegada a hora. Devido s propriedades matemticas das chaves pblicas e privadas e e d, Alice pode utilizar sua prpria chave privada dA decodificar a mensagem original ao invs de normalmente utiliz-la para decifrar uma mensagem cifrada com sua chave pblica destinada a ela. Isto faria com que a mensagem ficasse cifrada com a sua chave privada, e como isto seria til? Antes de partimos para o exemplo numrico, uma breve teoria. O que foi feito anteriormente (no exemplo de Bob): b CB(b) DB(CB(b)) b

197

CAPTULO 14 RSA

onde b a mensagem em texto puro BRASIL, CB( ) a funo de cifragem que utiliza a chave pblica de Bob, e DB( ) a funo de decifragem que utiliza a chave secreta de Bob. O esquema de assinatura digital consiste em b DA(b) CB(DA(b)) DB(CB(DA(b))) DA(b) CA(DA(b)) b Em outras palavras, Alice utiliza sua chave privada para decodificar a mensagem original (que inicialmente no uma ao muito til, pois qualquer pessoa com conhecimento da chave pblica de Alice pode desfazer esta ao). Em seguida, Alice cifra o resultado com a chave pblica de Bob. Neste ponto, o que temos? Uma mensagem-cpsula duplamente criptografada com a chave privada de Alice e a chave pblica de Bob. CB(DA(b)) = mensagem assinada O que isto garante? Que Bob pode primeiramente decodificar esta mensagem-cpsula com sua prpria chave privada, e assim obter de volta a mensagem criptografada com a chave privada de Alice. Mas aqui que surge o grande ponto: se Bob utilizar a chave pblica de Alice, ele obter a mensagem original! Como ningum mais alm de Alice possui a chave privada dA, ento realmente aquela mensagem foi codificada inicialmente por Alice. A outra garantia vem de que somente Bob sabe disto, pois Alice sabiamente tambm utilizou a chave pblica de Bob. Em resumo, no esquema tradicional (sem assinatura), b = D(C(b)). Entretanto, devido s propriedades matemticas, tambm verdade que b = C(D(b)). Ou seja, no esquema assinado temos, b = CA(DB(CB(DA(b)))), onde as funes so aplicadas de dentro para fora na ordem que aparecem. Agora o exemplo numrico. Relembrando as chaves pblicas de Alice e Bob: Chave pblica de Alice (nA, eA) = (469873, 5). Chave pblica de Bob (nB, eB) = (431957, 7). Chave privada de Alice (nA, dA) = (469873, 281081). Chave privada de Bob (nB, dB) = (431957, 307543). 1 passo: Alice utiliza sua chave privada para decodificar a palavra BRASIL separada em blocos como no exemplo anterior (as mesmas observaes de tamanho de bloco e valores menores que os mdulos tambm valem para este caso): DA(668) = 668281081 (mod 469873) DA(265) = 265281081 (mod 469873) DA(837) = 837281081 (mod 469873) DA(376) = 376281081 (mod 469873) 249014 (mod 469873) 82081 (mod 469873) 221564 (mod 469873) 231544 (mod 469873)

198

CAPTULO 14 RSA

2 passo: Em seguida, Alice pega o resultado e aplica na funo de codificao com a chave pblica de Bob: CB(249014) = 2490147 (mod 431957) 273379 (mod 431957) CB(82081) = 820817 (mod 431957) 45868 (mod 431957) CB(221564) = 2215647 (mod 431957) 414208 (mod 431957) CB(231544) = 2315447 (mod 431957) 363416 (mod 431957) Assim, Alice envia a mensagem criptografada assinada: (273379, 45868, 414208, 363416) 3 passo: Bob, ao receber a mensagem criptografada assinada, deve primeiramente aplicar sua chave privada ( uma calculeira danada, eu sei... Tente usar o computador!): DB(273379) = 273379307543 (mod 431957) 249014 (mod 431957) DB(45868) = 45868307543 (mod 431957) 82081 (mod 431957) DB(414208) = 414208307543 (mod 431957) 221564 (mod 431957) DB(363416) = 363416307543 (mod 431957) 231544 (mod 431957) J aqui neste resultado, Bob deve obter algo que Alice obteve no 1 passo. 4 passo: Finalmente, Bob aplica a chave pblica de Alice que garante que foi ela mesma que enviou esta mensagem (nenhuma outra pessoa alm de Alice pode ter a chave privada dela, lembra?): CA(249014) = 2490145 (mod 469873) 668 (mod 469873) CA(82081) = 820815 (mod 469873) 265 (mod 469873) CA(221564) = 2215645 (mod 469873) 837 (mod 469873) CA(231544) = 2315445 (mod 469873) 376 (mod 469873) Novamente, Bob reagrupa a mensagem decodificada 668265837376, separa em pares, (66, 82, 65, 83, 73, 76) e obtm a mensagem original BRASIL.

199

CAPTULO 14 RSA

Exerccios Para os exerccios seguintes utilize a tabela abaixo ou a tabela ASCII


A 10 N 23 [] 99 B 11 O 24 C 12 P 25 D 13 Q 26 E 14 R 27 F 15 S 28 G 16 T 29 H 17 U 30 I 18 V 31 J 19 W 32 K 20 X 33 L 21 Y 34 M 22 Z 35

1) Usando as chaves pblicas dadas, codifique e decodifique as frases correspondentes: a) (n, e) = (1037, 7) SONHO b) (n, e) = (2201, 11) REMO c) (n, e) = (1577, 5) ESPERANDO 2) A chave pblica utilizada pelo Banco Crash para codificar suas mensagens a seguinte: (n, e) = (4559, 5) Os computadores do banco receberam, de local indeterminado, os seguintes blocos de mensagem: 2621 2608 3594 4261 O que diz a mensagem? 3) A mensagem 96 61 751 9 foi codificada pelo mtodo RSA usando a chave (n, e) = (767, 13). Decodifique a mensagem. 4) Sabendo-se que n = 27641 igual ao produto de dois primos e que determine os fatores primos de n. 5) Sabendo-se que n = 3552377 igual ao produto de dois primos e que determine os fatores primos de n.
(n) 27300 ,

(n) 3548580 ,

6) A mensagem 6802 8728 9451 foi codificada pelo mtodo RSA usando a chave (n, e) = (16517, 5). Alm disso, sabe-se que (n) 16236 . Decodifique a mensagem. 7) (Assinatura Digital) Considere que o Banco Bandit possua a seguinte chave pblica: (nB, eB) = (20099, 7) E a Empresa Explorit possua a seguinte chave pblica: (nE , eE) = (61823, 11) Demonstre como a empresa enviaria a mensagem MONEY assinada ao banco e como o banco confirmaria a autenticidade da assinatura.

200

Captulo 15

PARTILHA DE SENHAS
O Teorema Chins do Resto utilizado em sistemas de partilha de senhas entre vrias pessoas, de modo que para cada pessoa seja dado um elemento distinto em funo da senha s a ser compartilhada. A grande vantagem do mtodo de partilha de senhas que cada pessoa contm uma chave diferente das outras chaves e uma pessoa sozinha no consegue decifrar a senha. Outra vantagem que no necessria a presena de todas as pessoas com suas respectivas chaves, bastam que k ou mais pessoas estejam presentes. A chave para cada pessoa escolhida dentro um conjunto S composto por n pares de inteiros positivos de forma que, para cada inteiro positivo k n previamente escolhido, tem-se que: Qualquer subconjunto de S com k elementos permite determinar s facilmente; muito difcil determinar s conhecendo menos que k elementos de S.

O primeiro passo escolher um conjunto apropriado L de n inteiros positivos distintos, dois a dois primos entre si. Seja N o produto dos k menores nmeros de L, e M o produto dos k 1 maiores nmeros de L, os elementos de L devem ser escolhidos com cuidado de forma que M < N, e a senha s possa ser escolhida arbitrariamente dentro do intervalo M < s < N. Os elementos at aqui mencionados so: L = {p1, p2, ..., pn}; onde p1 < p2 < ... < pn so primos distintos entre si. k = nmero de pessoas que se deseja estarem presentes para a decifragem da senha. N = p1.p2. ... .pk = produto dos k menores elementos de L. M = px.px+1. ... . pn = produto dos k 1 maiores elementos de L. s = senha aleatria que no tem relao alguma com L, mas deve ser escolhida de forma que M < s < N. Posteriormente, o conjunto gerador de chaves S ser constitudo pelos pares da forma (p, s p) onde p L e sp a forma reduzida de s (mod p). Um limite k 2 implica em s > p para qualquer p L. Logo sp < s para qualquer p L. Supe-se que sejam conhecidos, em um dado momento, t k pares de elementos de S, ou seja, existem t pessoas presentes para a decifragem da senha s. Denota-se esses pares por (p1, s1), (p2, s2), ..., (pt, st). S = {(p1, sp1), (p2, sp2), ..., (pt, spt)} Para se chegar senha s necessrio resolver o seguinte sistema de congruncias: x x s1 (mod p1) s2 (mod p2) ... x st (mod pt)

Pelo Teorema Chins do Resto, obtm-se x0 como soluo, tal que: x0 s (mod p1.p2. ... .pt)

201

CAPTULO 15 PARTILHA DE SENHAS

sabido que como t k, (p1.p2. ... .pt) soluo menor que (p1.p2. ... .pt).

N > k, e o sistema de congruncias tem uma nica

Observaes: possvel escolher os mdulos de s de modo que seja impraticvel encontrar s atravs de uma busca, conhecendo-se apenas uma das chaves; sempre possvel escolher um conjunto L que satisfaa todas as condies. A seguir, um exemplo prtico para efeito de esclarecimento. Exemplo 15.1: No banco Golden Luck h 5 funcionrios responsveis pela manuteno da senha de um cofre, e pelo menos 2 pessoas (k = 2) tm que estar presentes para a abertura do mesmo. Logo, o conjunto L deve possuir 5 elementos e seu limiar deve igual a 2. Uma possvel escolha para L envolvendo somente primos pequenos L = {11, 13, 17, 19, 23} A partir do qual se calcula os valores dos limites N e M: N = 143 = 11.13 = produto dos (k = 2) menores elementos de L. M = 23 = produto dos (k 1 = 2-1=1) maiores elementos de L. O valor da senha s pode ser escolhido aleatoriamente como qualquer inteiro no intervalo que vai de 23 a 143 (M < s < N). Por exemplo, suponha que a senha seja s = 50. Ento o conjunto S que contm os elementos da senha : S = {(11, 6), (13, 11), (17, 16), (19, 12), (23, 4)} O segundo termo de cada elemento de S, sm, o resto da diviso de s = 50 por cada termo correspondente de L, ou seja, s(mod p). Se os funcionrios que possuem as senhas (17, 16) e (23, 4), por exemplo, esto no banco, para obter a senha seria preciso resolver o sistema: x x Soluo: m = 17. 23 = 391 m m M1 = = 23 ; M 2 = = 17 m1 m2 Os inversos x1 e x2 de M1 e M2 so dados por: 23 x1 1 (mod 17) e 17x1 1 (mod 23) 16 (mod 17) 4 (mod 23)

202

CAPTULO 15 PARTILHA DE SENHAS

Aplicando o mtodo de Euler nas equaes acima obtemos as solues respectivas: x1 = 3 e x2 = 19 . Portanto, temos x a1M1x1 + a2M2x2 (mod m) x 16 . 23 . 3 + 4 . 17 . 19 (mod 391) x 2396 (mod 391) x = 50 Assim, determina-se que 50 o menor valor inteiro positivo congruente a x, que a senha correta.

Exerccios 1. Por motivo de segurana o banco Golden Luck trocou a senha do cofre. Dois funcionrios possuem as chaves (13, 5) e (19, 17). Qual a nova senha? 2. Aps uma nova troca de senhas no banco Golden Luck, 2 funcionrios tem agora as chaves (31, 6) e (41, 20). Qual a senha agora? 3. Como duas pessoas s sabem guardar um segredo se uma delas j estiver morta, o banco Golden Luck resolveu fazer uma mudana completa no esquema de segurana e novamente a senha foi trocada, alm disso ficou estabelecido que estejam presentes, no mnimo, 3 pessoas para que o cofre possa ser aberto. Trs funcionrios esto com as chaves (53, 21), (61, 9) e (71, 35). Qual a senha?

203

You might also like